Exam 1 Maternity Set

Lakukan tugas rumah & ujian kamu dengan baik sekarang menggunakan Quizwiz!

The nurse determines that a newborn has a minor genetic anomaly. What did the nurse most likely assess in this infant? Select all that apply. 1. Broad face 2. Wide-set eyes 3. Single palmar crease 4. Low anterior hairline 5. Upward-slanting eyes

1, 2, 3, 4Explanation: 1. Minor anomalies include a broad face. 2. Minor anomalies include wide-set eyes. 3. Minor anomalies include single palmar crease. 4. Minor anomalies include low anterior hairline. 5. Upward-slanting eyes are an ethnic variation.

A client's amniocentesis results indicate that the fetus is at risk for respiratory distress. What testing values support this clinical decision? Select all that apply. 1. Amniotic glucose level 50 mg/dL 2. Phosphatidylglycerol (PG) negative 3. Lecithin/sphingomyelin (L/S) ratio 1:6 4. Amniotic fluid red blood cell count 5 mg/dL 5. Lamellar body counts (LBCs) 5000/counts/mcL

A client's amniocentesis results indicate that the fetus is at risk for respiratory distress. What testing values support this clinical decision? Select all that apply. 1. Amniotic glucose level 50 mg/dL 2. Phosphatidylglycerol (PG) negative 3. Lecithin/sphingomyelin (L/S) ratio 1:6 4. Amniotic fluid red blood cell count 5 mg/dL 5. Lamellar body counts (LBCs) 5000/counts/mcL

Upon reviewing the client's chart, which piece of data would suggest to the nurse the primary healthcare provider may diagnose endometriosis? a. Back discomfort and dysphagia b. Ovarian pain and dysuria c. Pelvic cramping and dyspareunia d. Vaginal discharge and dysmenorrhea

Answer: c. Pelvic cramping and dyspareunia Feedback: The most common symptom of endometriosis is pelvic pain. Other symptoms include dysmenorrhea, dyspareunia, abnormal uterine bleeding, and infertility.

A 16-year-old girl asks, "Do I need to have a Pap smear just because I'm sexually active?" What is the nurse's correct response? 1. "No, you do not need to be screened for cervical cancer until you are 21 years old." 2. "Yes, all sexually active females should be screened for both cervical cancer and human papilloma virus (HPV)." 3. "Yes, all women under the age of 29 should be screened for both cervical cancer and human papilloma virus (HPV)." 4. "No, but you will need to begin your screenings for both cervical cancer and human papilloma virus (HPV) when you are 18 years old."

Answer: 1 Explanation: 1. 2012 guidelines issued by the U.S. Preventive Services Task Force (USPSTF) recommend initiating cervical cancer screening at age 21. 2. Engaging in sexual activity is not an indication for routine cervical cancer or for HPV screening. 3. 2012 guidelines issued by the U.S. Preventive Services Task Force (USPSTF) recommend cervical cancer screening without HPV cotesting in women ages 21 to 29. 4. 2012 guidelines issued by the U.S. Preventive Services Task Force (USPSTF) recommend initiating cervical cancer screening at age 21.

How should the nurse interpret a pregnant client's lecithin/sphingomyelin (L/S) ratio finding of 2:1 on amniotic fluid? 1. Fetal lungs are mature. 2. Fetal lungs are still immature. 3. The fetus has a congenital anomaly. 4. The fetus is small for gestational age.

Answer: 1 Explanation: 1. A 2:1 L/S ratio indicates that the risk of respiratory distress syndrome (RDS) is very low and that the fetus's lungs are mature. 2. Early in pregnancy, the sphingomyelin concentration in amniotic fluid is greater than the concentration of lecithin, and so the L/S ratio is low if the fetus's lungs are immature, which is not the case in this instance. 3. The L/S ratio is not a measurement for congenital anomalies. 4. The L/S ratio is not a measurement for size of the fetus.

A 26-year-old multigravida who is 28 weeks pregnant and follows a program of regular exercise develops gestational diabetes. What instructions should be included in a teaching plan for this client? 1. "Carry hard candy (or other simple sugar) when exercising." 2. "If your blood sugar is 120 mg/dL, eat 20 g of carbohydrate." 3. "Exercise either just before meals or wait until 2 hours after a meal." 4. "If your blood sugar is more than 120 mg/dL, drink a glass of whole milk."

Answer: 1 Explanation: 1. A client should be encouraged to continue any exercise programs in which she already is involved. She should keep hard candy (simple sugar) with her at all times, just in case the exercise induces hypoglycemia. 2. A finger-stick result of 120 mg/dL is considered to be normal. 3. It is best to exercise just after the meal in order to utilize the glucose. 4. Such clients need no additional carbohydrate or protein intake.

A pregnant client's fundal height is 26 cm at 32 weeks' gestation. Why would the healthcare provider schedule this client for sonograms every 2 weeks? 1. Evaluate fetal growth 2. Determine fetal presentation 3. Assess for congenital anomalies 4. Rule out a suspected hydatidiform mole

Answer: 1 Explanation: 1. A person who is at 32 weeks' gestation should measure 32 cm of fundal height. When a discrepancy between fundal height and measurement exists, the purpose of serial ultrasounds is to monitor fetal growth. 2. Fetal presentation would require only one ultrasound. 3. Assessment of anomalies would require only one ultrasound. 4. Ruling out a hydatidiform mole would require only one ultrasound.

1) A 7-year-old client says, "Grandpa, mommy, daddy, and my brother live at my house." In what type of family should the nurse identify that this child lives? 1. Extended 2. Binuclear 3. Traditional 4. Gay or lesbian

Answer: 1 Explanation: 1. An extended family contains a parent or a couple who share the house with their children and another adult relative. 2. A binuclear family includes the divorced parents, who have joint custody of their biologic children, who alternate spending varying amounts of time in the home of each parent. 3. The traditional nuclear family consists of an employed provider parent, a homemaking parent, and the biologic children of this union. 4. A gay or lesbian family is composed of two same-sex domestic partners; they might or might not have children.

Which statement indicates that client teaching about vaginal infections has been effective? 1. "The fishy vaginal odor I have is caused by a bacterial infection." 2. "I can use this antiyeast medication weekly to prevent another infection." 3. "My diabetes is unrelated to the frequency of my vaginal yeast infections." 4. "I should douche weekly to prevent a recurrence of my bacterial vaginosis."

Answer: 1 Explanation: 1. Bacterial vaginosis is characterized by a fishy vaginal odor and greenish discharge with a vaginal pH over 4.5. 2. Medication for vaginal yeast infections should be used as treatment, not prophylaxis. Using medication as prescribed is important client education. Medication should not be saved for future use. 3. Yeast vaginitis is more common in diabetic and pre-diabetic women. Four episodes or more per year of yeast vaginitis are an indication to screen a woman for diabetes. 4. Douching disrupts normal flora by washing out desirable bacteria; douching is not recommended.

The nurse is supervising care in the emergency department. Which situation requires immediate intervention? 1. Bright red bleeding with clots at 32 weeks' gestation; pulse = 110, blood pressure 90/50, respirations = 20. 2. Dark red bleeding at 30 weeks' gestation with normal vital signs; client reports the presence of fetal movement. 3. Spotting of pinkish brown discharge at 6 weeks' gestation and abdominal cramping; ultrasound scheduled in 1 hour. 4. Moderate vaginal bleeding at 36 weeks' gestation; client has an IV of lactated Ringer solution running at 125 mL/hour.

Answer: 1 Explanation: 1. Bleeding in the third trimester is usually associated with placenta previa or placental abruption. Blood loss can be heavy and rapid. This client has a low blood pressure with an increased pulse rate, which indicates hypovolemic shock, which can be fatal to the mother and therefore the baby. Both lives are at risk in this situation. Since there is no information given that the client has an IV started, this client is the least stable, and therefore the highest priority. 2. Occasional spotting can occur. The presence of normal vital signs and usual fetal movements reduces this client's risk of needing immediate intervention. 3. Bleeding in the first trimester can be indicative of the beginning of spontaneous abortion or of an ectopic pregnancy. An ultrasound will diagnose which situation is occurring and will determine care. Because this client is very early in the pregnancy and only experiencing spotting, it is not appropriate to have an IV at this time. 4. Bleeding in the third trimester is usually associated with placenta previa or placental abruption. Blood loss can be heavy and rapid, so having an IV stabilizes the client's vascular volume.

The nurse is leading a session on nutrition for postpartum clients. Which statement indicates that teaching has been effective? 1. "Breastfeeding requires that I eat lots of protein daily." 2. "Because I am breastfeeding, I need a low calcium intake." 3. "Since I am bottlefeeding, I don't have to eat vegetables." 4. "Bottlefeeding moms like me require a high sodium intake."

Answer: 1 Explanation: 1. Breastfeeding clients should consume 65 g of protein daily during the first 6 months of breastfeeding and 62 g daily during the second 6 months. Protein is a major ingredient in breast milk. 2. Breastfeeding requires an increase of 1000 mg per day of calcium—the same amount of calcium that is recommended during pregnancy. 3. Although vitamin intake is not directly related to bottlefeeding, good nutritional habits are important to form while bottlefeeding, because in the future, the baby will be eating what the mother eats. 4. Sodium intake is not increased during bottlefeeding.

The nurse who is counseling a group of middle school girls on pregnancy avoidance should include which statement? 1. "Although condom use is growing, there is still an increasing rate of STIs among teens." 2. "It has become far less acceptable to give birth during your teenage years than it used to be." 3. "You have learned enough from your friends and families to understand how pregnancy occurs." 4. "Although sexuality is common in the media, your peer pressure to have sex is not an important factor."

Answer: 1 Explanation: 1. Condom use is increasing, but the rate of sexually transmitted infections (STIs), including HIV, is also rising. Research indicates that young people 15 to 24 years of age make up 25% of the sexually experienced population in the United States. However, they account for nearly half of the new cases of STIs. 2. Society has become more accepting of teen pregnancy, and there are fewer stigmas attached to being a young mother. 3. Formal education on the physiology of the body and conception will decrease the myths and misunderstandings that abound among teens and undereducated adults. 4. Images of sexuality are common in American society: in music lyrics and videos, in advertising, in television shows and movies. Peer pressure to have sex is also common, and is a strong influence on when a teen becomes sexually active.

The nurse is preparing a brochure that compares and contrasts cystitis and pyelonephritis. Which information should be included in the brochure? 1. Urine culture is included in the evaluation of both cystitis and pyelonephritis. 2. Dysuria, especially at the end of urination, is often the initial symptom of both conditions. 3. Both conditions usually present with sudden onset of chills, high temperature, and flank pain. 4. Both conditions are associated with pregnancy complications including increased risk of preterm birth and of intrauterine growth restriction.

Answer: 1 Explanation: 1. Diagnosis of cystitis is made with a urine culture. Women with acute pyelonephritis should have a urine culture and sensitivity done to determine the appropriate antibiotic. 2. The initial symptom of cystitis is often dysuria, specifically at the end of urination. 3. Acute pyelonephritis has a sudden onset, with chills, high temperature, and flank pain (either unilateral or bilateral). 4. Pyelonephritis during pregnancy is associated with an increased risk of preterm birth and intrauterine growth restriction.

he nurse instructs on the importance of niacin during a preconception counseling class. Which food item selected by a participant indicates that teaching about the sources of niacin has been effective? 1. Fish 2. Milk 3. Apples 4. Broccoli

Answer: 1 Explanation: 1. Dietary sources of niacin include meats, fish, and enriched grains. 2. Milk will provide sources of other vitamins; however, it does not contain significant niacin. 3. Apples will provide sources of other vitamins; however, they do not contain significant niacin. 4. Broccoli will provide sources of other vitamins; however, it does not contain significant niacin.

The nurse has completed a presentation on reproduction. Which participant's statement indicates that teaching has been successful? 1. "Ova separate into two unequally sized cells." 2. "A male is born with all the sperm he will ever produce." 3. "Females create new ova throughout their reproductive life." 4. "Each primary spermatocyte divides into four haploid cells."

Answer: 1 Explanation: 1. Each ovum undergoes meiotic division just prior to being released from the graafian follicle. Each cell created by this meiosis has the same number of chromosomes, but the cytoplasm does not split equally. This causes a polar body to be produced along with a secondary oocyte. 2. Males begin spermatogenesis at puberty and continue throughout their life. Each sperm divides into four haploid cells. 3. Females are born with all the ova they will ever produce. The ova begin to be formed in early fetal life. One ovum is released each month during the reproductive life of a female, from menarche to menopause. 4. Primary spermatocytes contain 46 chromosomes (46XY). Each primary spermatocyte undergoes meiotic division into two haploid secondary spermatocytes (22X or 22Y).

12) Which family might find cord blood banking to be especially useful? 1. A family with a history of leukemia 2. A family with a history of infertility 3. A family that wishes to select the sex of a future child 4. A family that wishes to avoid a future intrauterine fetal surgery

Answer: 1 Explanation: 1. Families with a history of leukemia might find cord blood banking useful because cord blood, like bone marrow and embryonic tissue, contains regenerative stem cells, which can replace diseased cells in the affected individual. 2. A family with a history of infertility would not be helped by cord blood banking. 3. A family that wishes to select the sex of a future child would not be helped by cord blood banking. 4. A family that wishes to avoid a future intrauterine surgery would not be helped by cord blood banking.

The nurse is reviewing the assessment findings of a client who is at 35 weeks' gestation. Which data suggest the need for further investigation? 1. Glycosuria 2. Funic souffle 3. Pseudoanemia 4. Melasma gravidarum

Answer: 1 Explanation: 1. Glycosuria (glucose in the urine) during pregnancy may be normal or may indicate gestational diabetes, so it always warrants further testing. 2 Funic souffle is a normal assessment finding associated with the pulsing of blood through the umbilical cord. 3. Physiologic anemia of pregnancy or pseudoanemia is common during pregnancy and is an expected finding. 4. Facial chloasma or melasma gravidarum (also known as the "mask of pregnancy") is a harmless darkening of the skin over the cheeks, nose, and forehead that sometimes accompanies pregnancy.

During the first antepartal visit, a client who is at 10 weeks' gestation learns of being HIV positive. Which client statement indicates an understanding of the plan of care both during the pregnancy and postpartally? 1. "I should not breastfeed my baby." 2. "When my baby is 2 months old, he or she will be tested for HIV." 3. "If I have a cesarean section, there is an increased risk that my HIV will be passed to my baby." 4. "I am supposed to take highly active antiretroviral therapy (HAART), but only during the first trimester."

Answer: 1 Explanation: 1. HIV transmission can occur during pregnancy and through breast milk; however, it is believed that the majority of all infections occur during labor and birth. 2. Following birth, HIV infection in infants should be diagnosed using HIV virologic assays as soon as possible, with initiation of infant antiretroviral prophylaxis immediately if the test is positive. 3. Cesarean section reduces the transmission of HIV from mother to infant. 4. Longer duration therapy is preferable to shorter duration approaches, and it is best to start prophylaxis after the first trimester and no later than 28 weeks' gestation in women who do not require immediate therapy for their own health.

Which phone call should the prenatal clinic nurse return first? 1. Primipara at 32 weeks, reports headache and blurred vision 2. Primipara at 16 weeks, reports increased urinary frequency 3. Multipara at 18 weeks, reports no fetal movement this pregnancy 4. Multipara at 40 weeks, reports sudden gush of fluid and contractions

Answer: 1 Explanation: 1. Headache and blurred vision are signs of preeclampsia, which is potentially life threatening for both mother and fetus. This client has top priority. 2. Increased urinary frequency is common during pregnancy as the increased size of the uterus puts pressure on the urinary bladder. Urinary frequency is expected. If the client were reporting dysuria or hematuria, a urinary tract infection (UTI) would be suspected, but this client is only reporting increased urinary frequency. This client is a lower priority. 3. Fetal movement should be felt by 19 to 20 weeks. Multiparas sometimes feel fetal movement prior to 19 weeks, but the lack of fetal movement prior to 20 weeks is considered normal. This client is a lower priority. 4. A term client who is experiencing contractions and a sudden gush of fluid is in labor. Although laboring clients should be in contact with their healthcare provider for advice on when to go to the hospital, labor at term is an expected finding. This client is a lower priority.

The nurse is presenting an in-service to nursing staff regarding the provision of culturally competent client care. Which statement should the nurse include in the presentation? 1. "Developed countries are becoming increasingly more ethnically diverse." 2. "The rituals and customs of a group reflect the values of the dominant culture." 3. "Identification of cultural values is a task that is unrelated to providing culturally sensitive care." 4. "Many immigrants to a new country will adopt the beliefs and practices of the dominant culture."

Answer: 1 Explanation: 1. In many developed countries such as, for example, the United States, Canada, England, and Germany, populations are becoming more and more ethnically diverse as the number of immigrants continues to grow. 2. The rituals and customs of a group are a reflection of the group's values. 3. The identification of cultural values is useful in planning and providing culturally sensitive care. 4. It is not realistic or appropriate to assume that people of another culture will automatically abandon their ways and adopt the practices of the dominant culture.

The nurse reviews nutritional requirements with a breastfeeding mother who is concerned that her milk production has decreased. Which statement indicates that further teaching is required? 1. "I have started cutting back on my protein intake." 2. "At least 3 times a day, I drink a glass of milk." 3. "I am drinking a minimum of 8 to 10 glasses of liquid a day." 4. "I try to take a nap in the morning and afternoon when the baby is sleeping."

Answer: 1 Explanation: 1. It is especially important for the breastfeeding mother to consume sufficient calories because inadequate caloric intake can reduce milk volume. The decreased intake of protein represents a decrease in calories, which will decrease milk production. 2. The breastfeeding mother must increase her protein and calcium intake. 3. The breastfeeding mother must consume a minimum of 8 to 10 glasses of liquid per day. 4. It has also been found that adequate rest is necessary for the body to maintain its production of milk.

The partner of a pregnant client at 16 weeks' gestation accompanies her to the clinic. The partner tells you that the baby just does not seem real to him, and he is having a hard time relating to his partner's fatigue and food aversions. Which statement would be best for the nurse to make? 1. "Many men feel this way. Feeling the baby move will help make it real." 2. "My husband had no problem with this. What was your childhood like?" 3. "You might need professional psychologic counseling. Ask your physician." 4. "If you would concentrate harder, you would be aware of the reality of this pregnancy."

Answer: 1 Explanation: 1. Kicking and ultrasound visualization are concrete evidence of the baby's existence and often are turning points in acceptance for partners. 2. The ambivalence and disbelief occur across all socioeconomic groups, in both partners who were fathered well and those who grew up without a father. 3. This reaction is not indicative of psychologic pathology. 4. Ambivalence is common among partners, especially prior to either seeing the baby on ultrasound or feeling the baby kick and move.

Which approach to planning educational activities is best suited to a group of pregnant adolescents? 1. Primarily using visual-based presentations during teaching 2. Combining teaching for adolescent mothers of all ages as one group 3. Respecting confidentiality and building trust by avoiding the topics of drug and alcohol abuse 4. Avoiding the inclusion of handouts with bulleted items and white space as part of the teaching

Answer: 1 Explanation: 1. Many teens prefer teaching aids that are visual and that they can handle, such as realistic fetal models. 2. Teaching adolescents in groups according to their ages may be more effective for learning because younger adolescent mothers' parenting skills and emotional needs may differ from those of older adolescent mothers. 3. The nurse should review the risks associated with the use of tobacco, caffeine, drugs, and alcohol, as well as discussing the fetal effects of these substances. 4. Some pregnant teens have low reading levels and tend to prefer handouts and posters that have visual interest, short sentences, bulleted items, and white space.

A client at 18 weeks' gestation thinks she might have been exposed to a toxin at work that could affect fetal development and asks what organs might be affected at this point in the pregnancy. How should the nurse respond to this client? 1. "The brain is developing now and could be affected." 2. "Because you are in the second trimester, there is no danger." 3. "It's best to not worry about possible problems with your baby." 4. "The internal organs like the heart and lungs could be impacted."

Answer: 1 Explanation: 1. Maximum brain growth and myelination are occurring at this point in fetal development. 2. Although the greatest danger from teratogens is during the embryonic stage (the first 8 weeks of pregnancy), the fetus at 20 weeks is still vulnerable to exposure to teratogens. 3. Avoid telling clients not to worry. Doing so indicates to the client that you do not care about their concerns and will end effective communication. 4. The heart, lungs, and other internal organs form during the embryonic state, or the first 8 weeks of pregnancy. During their formation is when they are most likely to be affected by a teratogen.

The nurse is creating a teaching poster for pregnant mothers. Which description of fetal development should the nurse include? 1. Most organs are formed by 8 weeks after fertilization. 2. The embryonic stage is from fertilization until 5 months. 3. Four layers of cells form after the embryo is at the ball stage. 4. After fertilization, the cells only become larger for several weeks.

Answer: 1 Explanation: 1. Most organs are formed during the embryonic stage, which lasts from the 15th day after fertilization until the end of the 8th week after fertilization. This is also a critical period because major organs are being developed and teratogens introduced during this time can increase the risk of congenital abnormalities. 2. The embryonic stage begins on the 15th day after fertilization and ends at the completion of the 8th week after fertilization. 3. Three primary germ layers form from the ball of undifferentiated cells, the blastocyst: ectoderm, mesoderm, and endoderm. 4. After fertilization, the cells reproduce by mitosis, resulting in more cells, not larger cells.

The clinic nurse is assisting with an initial prenatal assessment. The following findings are present: spider nevi present on lower legs; dark pink, edematous nasal mucosa; mild enlargement of the thyroid gland; mottled skin and pallor on palms and nail beds; heart rate 88 with murmur present. What is the best action for the nurse to take based on these findings? 1. Have the healthcare provider see the client today. 2. Instruct the client to avoid direct sunlight. 3. Document the findings on the prenatal chart. 4. Analyze previous thyroid hormone laboratory results.

Answer: 1 Explanation: 1. Mottling of the skin is indicative of poor oxygenation and a circulation problem. Skin and nail bed pallor can indicate either hypoxia or anemia. These abnormalities must be reported to the healthcare provider immediately. 2. Spider nevi are common in pregnancy due to the increased vascular volume and high estrogen levels. Nasal passages can be inflamed during pregnancy from edema, caused by increased estrogen levels. 3. These abnormalities must be reported to the healthcare provider immediately. 4. The thyroid gland increases in size during pregnancy due to hyperplasia.

The nurse is explaining the menstrual cycle to a group of women. In which phase should the nurse instruct that the corpus luteum begins to degenerate, estrogen and progesterone levels fall, and the blood supply to the endometrium is reduced? 1. Ischemic phase 2. Secretory phase 3. Menstrual phase 4. Proliferative phase

Answer: 1 Explanation: 1. The ischemic phase is characterized by ischemia of the endometrium. 2. The secretory phase involves glycogen secretion by the endometrium after ovulation. 3. The menstrual phase is the menses. 4. The proliferative phase is characterized by proliferation of the endometrium.

The community nurse is caring for a client at 32 weeks' gestation diagnosed with preeclampsia. Which statement indicates that additional information is needed about the health problem? 1. "My urine may become darker and smaller in amount each day." 2. "Lying on my left side as much as possible is good for the baby." 3. "I should call the doctor if I develop a headache or blurred vision." 4. "Pain in the top of my abdomen is a sign my condition is worsening."

Answer: 1 Explanation: 1. Oliguria is a complication of preeclampsia caused by renal involvement and is a sign that the condition is worsening. Oliguria should be reported to the healthcare provider. 2. Left lateral position maximizes uterine and renal blood flow and therefore is the optimal position for a client with preeclampsia. 3. Headache and blurred vision or other visual disturbances are an indication of worsening preeclampsia and should be reported to the healthcare provider. 4. Epigastric pain is an indication of liver enlargement, a symptom of worsening preeclampsia, and should be reported to the healthcare provider.

While developing a conference for adolescents, the nurse prepares a handout describing socioeconomic and cultural factors that contribute to adolescent pregnancy. Which information should the nurse include in the handout? 1. Poverty is a major risk factor for teen pregnancy. 2. All cultures share an aversion to early pregnancy. 3. A child born to a teenage mother is at a lower risk for teen pregnancy. 4. The younger the teen when she first gets pregnant, the less likely she is to have another pregnancy in her teens.

Answer: 1 Explanation: 1. Poverty is a major risk factor for teen pregnancy. Adolescents who do not have access to middle-class opportunities tend to maintain their pregnancies because they see pregnancy as their only option for adult status. Teens who are on a low economic trajectory are more likely to become pregnant because of the lack of economic opportunity and the social marginalization that comes with poverty. 2. Early pregnancy is desirable in some cultures, such as where Islam is the predominant religion, where large families are desired, where social change is slow in coming, and where most childbearing occurs within marriage. 3. Daughters and sisters of a woman who had a baby in her early teens tend to have intercourse earlier and are at higher risk for teen pregnancy themselves. 4. The younger the teen when she first gets pregnant, the more likely she is to have another pregnancy in her teens.

A 63-year-old female client requests information about complementary and alternative therapies that promote wellness during menopause. Which therapy should the nurse recommend? 1. Soy for reducing insomnia symptoms 2. Non-weight-bearing exercise, such as swimming 3. Calcium intake of 600 mg per day to help prevent osteoporosis 4. Increased consumption of phytoestrogens for women with a history of endometriosis or fibroids

Answer: 1 Explanation: 1. Research suggests that isoflavones, which are found in soy, are effective in reducing symptoms of insomnia in postmenopausal women. 2. Weight-bearing exercises such as walking, jogging, tennis, and low-impact aerobics are encouraged in order to increase bone mass and decrease the risk of osteoporosis. 3. Perimenopausal and postmenopausal women are advised to have a calcium intake of at least 1200 mg per day. Most women require supplements to achieve this level. 4. Women who have endometriosis or fibroids should be cautioned about the use of phytoestrogens.

A woman is experiencing mittelschmerz and increased vaginal discharge. Her temperature has increased by 0.6°C (1.0°F) for the past 36 hours. What should these findings indicate to the nurse? 1. Ovulation has occurred. 2. Ovulation will occur soon. 3. Menstruation is about to begin. 4. She is pregnant and will not menstruate.

Answer: 1 Explanation: 1. Signs that ovulation has occurred include: pain associated with rupture of the ovum (mittelschmerz), increased vaginal discharge, and a temperature increase of 0.6°C (1.0°F) over the past 36 hours. 2. A temperature increase does not occur before ovulation has occurred. 3. A temperature increase does not occur when menstruation is about to begin. 4. Pregnancy can be detected only through testing the urine or serum for the presence of human chorionic gonadotropin hormone.

4) A young woman claims that contraception is not needed because she avoids intercourse on the day of ovulation. How should the nurse instruct this client? 1. Sperm survive 48 to 72 hours in the female reproductive tract. 2. After ovulation, ova are considered fertile for about 72 to 96 hours. 3. Sperm are believed to be healthy and highly fertile for at least 5 days. 4. Refraining from intercourse on the day of ovulation will effectively prevent pregnancy.

Answer: 1 Explanation: 1. Sperm survive 48 to 72 hours in the female reproductive tract. 2. Ova are considered fertile for about 12 to 24 hours after ovulation. 3. Sperm are believed to be most fertile for the first 24 hours following entry into the female reproductive tract. 4. Because sperm survive 48 to 72 hours in the female reproductive tract, avoidance of intercourse for 24 hours will not reliably prevent pregnancy from occurring.

The mother of a client who is 14 weeks pregnant is uncertain about how to be a good grandmother to this baby due to working full time and being so busy. How should the nurse respond in this situation? 1. "How do you envision your role as grandmother?" 2. "Don't worry. You'll be a wonderful grandmother. It will all work out fine." 3. "As long as there is another grandmother available, you do not have to worry." 4. "Grandmothers are supposed to be available. You should retire from your job."

Answer: 1 Explanation: 1. Supportive, nonjudgmental exploration of the client's concerns is one component of therapeutic communication and is appropriate. 2. Casual and/or false reassurance is not appropriate. Effective therapeutic communication requires supportive, nonjudgmental exploration of the client's concerns. 3. Minimizing a client's concern is not appropriate. Effective therapeutic communication requires supportive, nonjudgmental exploration of the client's concerns. 4. Assignment of guilt is not appropriate. Effective therapeutic communication requires supportive, nonjudgmental exploration of the client's concerns

A pregnant client asks, "What's the difference between the true pelvis and the false pelvis?" How should the nurse respond? 1. "The false pelvis helps support the weight of the pregnant uterus." 2. "The false pelvis consists of the inlet, the pelvic cavity, and the outlet." 3. "The true pelvis does not affect fetal passage during labor and childbirth." 4. "The true pelvis helps direct the presenting fetal part into the false pelvis."

Answer: 1 Explanation: 1. The false pelvis helps support the weight of the pregnant uterus. 2. The true pelvis consists of the inlet, the pelvic cavity, and the outlet. 3. The size and shape of the true pelvis must be adequate for normal fetal passage during labor and childbirth. 4. The false pelvis helps direct the presenting fetal part into the true pelvis.

6) During an assessment, the nurse notices that an African American baby has a darker, slightly bluish patch about 5 by 7 cm on the buttocks and lower back. What should the nurse do? 1. Chart the presence of a Mongolian spot. 2. Ask the mother about the cause of the bruise. 3. Confer with the physician about the possibility of a bleeding tendency. 4. Call the Department of Social Services (DSS) to report this sign of abuse.

Answer: 1 Explanation: 1. The nurse will chart the presence of a Mongolian spot, such as is observed in races with dark skin tones. 2. Asking the mother about the cause of the bruise reveals cultural ignorance in a less damaging manner than does calling DSS. 3. If choosing to confer with the physician, the nurse will reveal ignorance in culturally competent assessments. 4. The nurse who calls the DSS to report this patch as a sign of abuse will reveal ignorance in culturally competent assessments and possibly provoke harassment of the family.

During an interview the nurse asks the partner of a woman in the second trimester of pregnancy what changes they have noticed during the pregnancy. Which answer would indicate a typical response to pregnancy? 1. "She daydreams about what kind of parent she is going to be." 2. "I have not noticed anything. I just found out she is pregnant." 3. "She has been having dreams at night about misplacing the baby." 4. "She has been more tense and anxious than usual, and she is not sleeping well."

Answer: 1 Explanation: 1. The second trimester usually brings increased introspection and consideration of how she will parent. 2. In the first trimester, pregnant women usually tell their partners of the pregnancy, explore their relationship with their mother, and think about their own role as a mother. 3. The needs of the newborn typically are not considered until the third trimester, at which time dreams of misplacing the baby or being unable to get to the baby also may be common. 4. During the third trimester, the woman typically experiences more anxiety and tension, as well as increased discomfort and insomnia.

The nurse is preparing a handout on the ovarian cycle for a group of middle school girls. Which information should the nurse include? 1. There are two phases of the ovarian cycle: luteal and follicular. 2. The hormone human chorionic gonadotropin (hCG) stimulates ovulation. 3. The ovum travels from the ovary to the tube during the luteal phase. 4. Irregular menstrual cycles have varying lengths of the follicular phase.

Answer: 1 Explanation: 1. The two phases of the ovarian cycle are follicular (days 1 to 14 of the menstrual cycle) and luteal (days 15 to 28 of the menstrual cycle). 2. hCG is secreted by a fertilized ovum and does not stimulate ovulation. 3. The ovum is released from the graafian follicle of the ovary and travels to the fallopian tube during the follicular phase of the ovarian cycle. 4. Menstrual cycles that are irregular in length have a consistent follicular phase but a varying luteal phase.

A client at 30 weeks' gestation is tearful at the time of her follow-up visit. She tells the prenatal clinic nurse that she is excited to finally become a mother and that she has been thinking about what kind of parent she will be. However, she is upset because her mother has told her that she does not want to be a grandmother because she does not feel old enough. Meanwhile, the client's husband has said that the pregnancy does not feel real to him yet and that he will become excited when the baby is actually here. What is the most likely explanation for what is happening within this family? 1. Family members are adjusting to the role change at their own pace. 2. Her mother is rejecting the role of grandparent and will not help out. 3. Her husband will not attach with this child and will not be a good father. 4. The client is not progressing through the developmental tasks of pregnancy.

Answer: 1 Explanation: 1. This is a true statement. When the other family members are at different stages of adjustment to the pregnancy, conflict can ensue. 2. Adaptation to the role of grandparent is another life task that takes time. Younger grandparents often have busy and full lives and view grandparenthood as a time for elderly people who are retired and slowed down. The family will form a view of grandparenthood within this family, in a way that works for them. 3. The husband's statement is quite common. Partners often feel that a pregnancy is not real to them because they are not experiencing any of the physical changes associated with pregnancy. 4. This is a false statement. The client is at the stage of seeking acceptance of this child by others, which first will be her partner and other family members.

The pregnant client has completed the prenatal questionnaire and asks the nurse why this form had to be filled out. Which response is the most appropriate? 1. "We occasionally identify a health problem that puts the current pregnancy at higher risk." 2. "This form is designed to predict who will develop problems with their pregnancy or delivery." 3. "The doctor wants all of the pregnant clients to fill out the form so that our records are complete." 4. "Some people have things that have happened in the past that could impact their current pregnancy."

Answer: 1 Explanation: 1. This is the reason for risk assessment during pregnancy, whether it is a client-completed questionnaire or a nurse assessment form. 2. The form will identify those clients who have risk factors based on their medical history; prediction implies seeing into the future without a basis for the concern. 3. The purpose of the form is to identify which clients have risk factors; the fact that records are complete is less important than identifying at risk pregnancies. 4. Although this is true, this statement is too vague to be the best response. It is best to explain specifically that the impact on the current pregnancy might put the pregnancy at higher risk.

The nurse is admitting a client at 28 weeks' gestation to the emergency department following an episode of domestic abuse resulting in ecchymosis and lacerations. Which question is most critical to ask? 1. "Do you have a safe place where you can go?" 2. "What did you do to make your spouse so angry?" 3. "How many times has this happened in the past?" 4. "Will you be pressing charges against your spouse?"

Answer: 1 Explanation: 1. This question is the highest priority because having a safe place to go after leaving the hospital reduces the risk of a repeated attack and further injury to both mother and fetus. 2. This statement is blaming and must be avoided to establish a trusting, therapeutic relationship with an abused client. 3. Although domestic abuse tends to increase in frequency and violence during pregnancy, this is not the highest priority. 4. Legal issues are a low priority at this time. Physiologic issues such as safety in the future have more importance.

The nurse is preparing a client in her second trimester for a three-dimensional ultrasound examination. Which statement indicates that teaching had been effective? 1. "I might be able to see who the baby looks like with the ultrasound." 2. "If the ultrasound is normal, it means my baby has no abnormalities." 3. "The nuchal translucency measurement will diagnose Down syndrome." 4. "Measuring the length of my cervix will determine if I will deliver early."

Answer: 1 Explanation: 1. Ultrasounds provide a very clear photo-like image of the fetus, often providing parents the opportunity to identify a familial characteristic such as nose shape. 2. Not all fetal anomalies are detectable by ultrasound. 3. Nuchal translucency measurements are screening, not diagnostic, for trisomies 13, 18, and 21. 4. Transvaginal ultrasound is used to measure the cervical length as a screening for risk of preterm labor. However, a normal-length cervix does not preclude preterm birth.

Which statement from the mother of a pregnant 13-year-old would be an expected response? 1. "We had such high hopes for you." 2. "I told you that boy was up to no good." 3. "But she was always an easygoing child." 4. "This is just one of those things that happen."

Answer: 1 Explanation: 1. When an adolescent pregnancy is first revealed to the teen's mother, the result is often anger, shame, or disappointment. The degree of negative response will be determined by the age of the teen, the family expectations for the teen, and the presence or absence of other teen pregnancies in the family or support network. In early adolescence, the teen's mother frequently accompanies her daughter to prenatal examinations. The role of the nurse is to facilitate communication between mother and daughter and provide education for both. 2. When an adolescent pregnancy is first revealed to the teen's mother, the result is often anger, shame, or disappointment. 3. When an adolescent pregnancy is first revealed to the teen's mother, the result is often anger, shame, or disappointment. 4. When an adolescent pregnancy is first revealed to the teen's mother, the result is often anger, shame, or disappointment.

A 31-year-old woman who is at high risk for diabetes is at 18 weeks' gestation. During her first antenatal visit, which is the accurate approach to evaluate the client for diabetes? 1. Conduct screening for type 2 diabetes mellitus as soon as possible. 2. Begin serial testing of the client's serum glucose and HA1c at 24 weeks' gestation. 3. If diabetes is diagnosed, consider this condition to be gestational diabetes mellitus (GDM). 4. Recognize HA1c equal to or greater than 4.5% or a fasting plasma glucose level equal to or greater than 90 mg/dL as being diagnostic of diabetes.

Answer: 1 Explanation: 1. Women at high risk for type 2 DM should be screened for diabetes as soon as possible. 2. Women at high risk for type 2 DM should be screened for diabetes as soon as possible. 3. Women who are determined to have diabetes at this visit should be diagnosed as having overt diabetes and not GDM. 4. HA1c equal to or greater than 6.5% would be considered diagnostic, as would a fasting plasma glucose level equal to or greater than 126 mg/dL or a 2-hour plasma glucose equal to or greater than 200 mg/dL during an oral glucose tolerance test (OGTT).

A client asks, "Is it okay for me to use a vaginal douche each day when I'm on my period?" How should the nurse respond? 1. "Douching should be avoided when you're on your period." 2. "Regular douching is necessary in order to promote good hygiene." 3. "Using a douche each day will help prevent vaginal infections from occurring." 4. "During your period, douching will help promote the flow of menstrual secretions."

Answer: 1 Explanation: 1. Women should avoid douching during menstruation because the cervix is dilated to permit the downward flow of menstrual fluids from the uterine lining. 2. Douching as a hygiene practice is unnecessary because the vagina cleanses itself. 3. Douching washes away the natural mucus and upsets the vaginal flora, which can make the vagina more susceptible to infection. 4. During menstruation, douching may interfere with downward flow of menstrual fluids from the uterine lining.

The nurse is assisting an expectant couple in developing a birth plan. Which instructions should the nurse include when teaching about this plan? 1. It is a communication tool between the client and the healthcare provider. 2. It is a legally binding contract between the client and the healthcare provider. 3. It allows the client to make choices about the birth process; however, these choices cannot be altered. 4. It includes only client choices and does not take into account standard choices of the healthcare provider.

Answer: 1 Explanation: The birth plan is used as a tool for communication among the expectant parents, the healthcare provider, and the healthcare professionals at the birth setting. 2. It is not a legal document. 3. The written plan identifies options that are available; thus, it can be altered. 4. The birth plan is used as a tool for communication among the expectant parents, the healthcare provider, and the healthcare professionals at the birth setting.

A nurse provides a client with instructions regarding breast self-examination (BSE). Which client statements indicate an understanding of detecting lumps in the breast? Select all that apply. 1. "I should inspect my breasts in a circular manner." 2. "Knowing the texture and feel of my breasts is important." 3. "I should perform BSE 1 week prior to the start of my period." 4. "When I reach menopause, I will perform BSE every 2 months." 5. "I should inspect my breasts while in a supine position, with my arms at my sides."

Answer: 1, 2 Explanation: 1. Checking breasts in a circular manner, feeling all parts of the breast, provides adequate palpation and possible detection of lumps. 2. A woman who knows the texture and feel of her own breasts is far more likely to detect changes that develop. 3. BSE should be performed 1 week after the start of each menstrual period because hormonal levels are lowest and allow closer examination of softer breast tissue. 4. BSE should be performed monthly, on the same day each month, during menopause. 5. The breasts should be inspected while standing with arms at sides.

The nurse is preparing a presentation about causes of fetal organ malformation in the first trimester. What prenatal influences on the intrauterine environment should be included in this teaching? Select all that apply. 1. The use of drugs 2. Maternal nutrition 3. The use of saunas or hot tubs 4. Age of the mother at conception 5. The quality of the sperm or ovum

Answer: 1, 2, 3 Explanation: 1. Many drugs can have teratogenic effects. 2. Maternal nutrition, if deficient, can cause damage to the fetus. Vitamins and folic acid taken prior to and during the pregnancy can have beneficial effects. 3. The use of saunas or hot tubs is associated with maternal hyperthermia and neural tube defects. 4. A maternal age of 35 or older is associated with genetic defects that occur at conception, not with first-trimester organ malformation. 5. The quality of the sperm or ovum can affect fertility but not organ formation.

20) The maternity nurse is planning to incorporate complementary and alternative therapies when providing care to clients in labor. What should the nurse do before implementing these therapies? Select all that apply. 1. Find evidence-based research for the use of the therapies. 2. Identify the therapies that are within the scope of nursing practice. 3. Document the use of therapies within the context of nursing practice. 4. List the therapies that are permitted according to the nurse practice act. 5. Determine which therapies are appropriate for a client's health problem.

Answer: 1, 2, 3, 4 Explanation: 1. The use of complementary and alternative therapies should also be supported by evidence-based research. 2. Nurses should use complementary modalities that are in the scope of their nursing practice. 3. Nurses who use complementary modalities should document their use within the context of nursing practice. This is most effective when the modality is identified as an intervention to address a specific nursing diagnosis or an identified client need. 4. Nurses should use complementary modalities that are in the nursing practice act in their state. 5. Determining which therapies are appropriate for a client's health problem is beyond the nurse's scope of practice.

A pregnant client diagnosed with Chlamydia trachomatis infection is refusing treatment. What effects on the fetus should the nurse explain might occur if treatment is waived? Select all that apply. 1. Fetal death 2. Premature labor 3. Newborn conjunctivitis 4. Chlamydial pneumonia 5. Ophthalmia neonatorum

Answer: 1, 2, 3, 4Explanation: 1. Fetal death is a potential adverse effect of maternal untreated Chlamydia trachomatis infection. 2. Premature labor is a potential adverse effect of maternal untreated Chlamydia trachomatis infection. 3. An infant of a woman with untreated chlamydial infection may develop newborn conjunctivitis. 4. An infant of a woman with untreated chlamydial infection may develop chlamydial pneumonia. 5. Ophthalmia neonatorum is associated with gonorrhea.

Healthcare providers strongly suggest that a client undergo genetic testing before becoming pregnant. What should the nurse do to ensure this client's informed consent and confidentiality? Select all that apply. 1. Explain potential societal impact. 2. Discuss risks and benefits of the test. 3. Explain that all genetic testing is voluntary. 4. Review potential physical or psychologic harm. 5. Expect that results are to be shared with family.

Answer: 1, 2, 3, 4Explanation: 1. It is the nurse's responsibility to ensure that the consent process includes discussion of the potential societal injury due to stigmatization or discrimination. 2. It is the nurse's responsibility to ensure that the consent process includes discussion of the risks and benefits of the test. 3. All genetic testing should be voluntary. 4. It is the nurse's responsibility to ensure that the consent process includes discussion of any physical or psychologic harm. 5. Discussing the sharing of results with family does not ensure the client's informed consent or confidentiality.

20) A female client is considering ovulation-inducing medications to achieve pregnancy. What should the nurse explain as potential adverse effects of this type of assisted reproductive technology (ART)? Select all that apply. 1. Miscarriage 2. Preterm birth 3. Neonatal morbidity 4. Multifetal pregnancy 5. Pelvic inflammatory disease

Answer: 1, 2, 3, 4Explanation: 1. Multifetal pregnancy, which can occur through the use of ovulation-inducing medications, increases the risk of miscarriage. 2. Multifetal pregnancy, which can occur through the use of ovulation-inducing medications, increases the risk of preterm birth. 3. Multifetal pregnancy, which can occur through the use of ovulation-inducing medications, increases the risk of neonatal morbidity. 4. Multifetal pregnancy can occur through the use of ovulation-inducing medications. 5. Pelvic inflammatory disease is not an adverse effect of ovulation-inducing medications.

A pregnant client is reviewing the tentative buffet menu for her upcoming baby shower. Which items should the nurse suggest be substituted to ensure the health of the client and developing fetus? Select all that apply. 1. Baked brie with crackers 2. Greek salad with feta cheese 3. Caesar salad with sourdough croutons 4. Salmon pate with toasted bread rounds 5. Grilled orange roughy with summer vegetables

Answer: 1, 2, 3, 4Explanation: 1. Soft cheese such as brie should be avoided because of the potential for harboring Listeria. 2. Soft cheese such as feta should be avoided because of the potential for harboring Listeria. 3. Salad dressings such as caesar are made with raw eggs, which should be avoided because of the potential for a Salmonella infection. 4. Pates should be avoided because of the potential for harboring Listeria. 5. There are no known bacterial infection issues associated with eating grilled orange roughy and summer vegetables while pregnant.

A pregnant client is diagnosed with a cardiac problem. What should the nurse prepare to instruct this client to do, to ensure a safe pregnancy? Select all that apply. 1. Restrict activities. 2. Follow a diet high in iron and protein. 3. Restrict the intake of sodium. 4. Obtain 8 to 10 hours of sleep. 5. Obtain pneumococcal vaccination.

Answer: 1, 2, 3, 4Explanation: 1. To help preserve her cardiac reserves, the woman may need to restrict her activities. 2. For the pregnant client with cardiac problems, the client should be instructed in the importance of a diet high in iron and protein. 3. For the pregnant client with cardiac problems, the client should be instructed in the importance of a diet low in sodium. 4. For the pregnant client with cardiac problems, 8 to 10 hours of sleep are essential. 5. Because upper respiratory infections may tax the heart and lead to decompensation, the woman must avoid contact with sources of infection. A pneumococcal vaccination is not indicated.

12) The nurse notes a general increase in clients' use of complementary and alternative therapies. What should the nurse identify as reasons for the increase in these therapies? Select all that apply. 1. Increased media attention 2. The advent of the Internet 3. Increased international travel 4. The use of traditional Western medicine for treatment 5. Increased consumer awareness of the limitations of conventional medicine

Answer: 1, 2, 3, 5 Explanation: 1. Increased media attention has spotlighted complementary and alternative therapies. 2. The advent of the Internet has made obtaining complementary and alternative therapies easier. 3. Increased international travel has increased the awareness of complementary and alternative therapies. 4. The use of traditional Western medicine for treatment often has stopped the use of complementary therapies and forced clients to hide the fact they use them from their healthcare providers. 5. Increased consumer awareness of the limitations of current conventional medicine has increased the awareness of complementary and alternative therapies.

14) Which practices characterize the basic competencies related to evidence-based practice? Select all that apply. 1. Clinical practice supported by data 2. Clinical practice that promotes quality 3. Clinical practice supported by good evidence 4. Clinical practice supported by intuitive evidence 5. Clinical practice that provides a useful approach to problem solving

Answer: 1, 2, 3, 5 Explanation: 1. Supported by data is a hallmark characteristic of the basic competencies related to evidence-based practice. 2. Promoting quality is a hallmark characteristic of the basic competencies related to evidence-based practice. 3. Supported by good evidence is a hallmark characteristic of the basic competencies related to evidence-based practice. 4. Clinical practice supported by intuitive evidence does not provide valid evidence and data for the proper actions. 5. Providing a useful approach to problem solving is a hallmark characteristic of the basic competencies related to evidence-based practice.

The nurse is reviewing clients who would benefit from a biophysical profile (BPP). Which clients should the nurse identify as a priority? Select all that apply. 1. A gravida who is postterm 2. A gravida with intrauterine growth restriction 3. A gravida with mild hypertension of pregnancy 4. A gravida who is experiencing nausea and vomiting 5. A gravida who complains of decreased fetal movement for 2 days

Answer: 1, 2, 3, 5 Explanation: 1. The infant who is postterm might be compromised due to placental insufficiency. 2. The infant who has intrauterine growth problems might be compromised due to placental insufficiency. 3. The BPP is indicated when there is risk of placental insufficiency or fetal compromise because of maternal preeclampsia or eclampsia. 4. Maternal nausea and vomiting is not a criterion for a BPP. 5. The gravida who is experiencing decreased fetal movement for 2 days needs assessment of the placenta and the fetus.

15) The maternal-child nurse is caring for a teenager who is 16 weeks pregnant. What actions should the nurse perform when advocating for this client? Select all that apply. 1. Understand what the client needs. 2. Know the needs of the client's family. 3. Compile a list of community resources. 4. Coordinate services to meet quality measures. 5. Examine policies to ensure meeting the client's needs.

Answer: 1, 2, 3, 5 Explanation: 1. To be an effective advocate, the nurse must be aware of the individual's needs. 2. To be an effective advocate, the nurse must be aware of the family's needs. 3. To be an effective advocate, the nurse must be aware of the healthcare services available in the hospital and the community. The nurse can then assist the family to make informed choices about these services and to act in their best interests. 4. Case management is a process of coordinating the delivery of healthcare services in a manner that focuses on quality outcomes. 5. To be an effective advocate, nurses must also ensure that the policies and resources of healthcare agencies meet the psychosocial needs of childbearing women and of children and their families.

The nurse is preparing to perform an initial prenatal assessment of a pregnant client who recently immigrated to the United States. Which cultural aspects should the nurse include during the assessment of this client? Select all that apply. 1. Birth rituals 2. Nutritional practices 3. Use of home remedies 4. Expectations to return to work 5. Beliefs about exercise and activity

Answer: 1, 2, 3, 5Explanation: 1. Birth rituals may vary according to cultural group and should be assessed. 2. Nutritional practices may vary according to cultural group and should be assessed. 3. Use of home remedies may vary according to cultural group and should be assessed. 4. Expectations to return to work are part of anticipatory guidance, from which every pregnant client and family would benefit. 5. Beliefs about exercise and activity may vary according to cultural group and should be assessed.

During a phone call to the clinic the nurse suspects that a pregnant client is experiencing preeclampsia. What manifestations did this client report that caused the nurse to make this clinical determination? Select all that apply. 1. Double vision 2. Epigastric pain 3. Facial swelling 4. Painful urination 5. Severe headache

Answer: 1, 2, 3, 5Explanation: 1. Double vision is a manifestation of preeclampsia. 2. Epigastric pain is a manifestation of preeclampsia. 3. Facial edema is a manifestation of preeclampsia. 4. Dysuria is a manifestation of a urinary tract infection. 5. Severe headache is a manifestation of preeclampsia.

A pregnant client's quadruple screen shows a risk for the fetus to have Down syndrome. Which test results within the screen were used to make this determination? Select all that apply. 1. Inhibin-A 2. Alpha-fetoprotein (AFP) 3. Unconjugated estriol (UE) 4. Hemoglobin electrophoresis 5. Human chorionic gonadotropin (hCG)

Answer: 1, 2, 3, 5Explanation: 1. Higher than normal levels of inhibin-A may indicate that a woman is at increased risk of having a baby with Down syndrome. 2. Lower than normal AFP could indicate that the woman's child is at risk for Down syndrome or trisomy 18. 3. Lower than normal UE may indicate that a woman is at increased risk of having a baby with Down syndrome. 4. Hemoglobin electrophoresis is not a test within the quadruple screen. 5. Higher than normal levels of hCG may indicate that a woman is at increased risk of having a baby with Down syndrome.

17) The nurse is preparing a teaching session for staff nurses on cultural influences of childbearing practices. Which topics should the nurse include? Select all that apply. 1. Gender of children 2. Number of children 3. Use of contraception 4. Achievement of developmental milestones 5. Pregnancy as an illness or expected condition

Answer: 1, 2, 3, 5Explanation: 1. In some cultures, a woman who gives birth achieves a higher status, especially if the child is male. 2. In many cultures throughout the world, it is common to have as many children as possible. 3. Culture may also influence attitudes and beliefs about contraception. In some cultures, contraception is appropriate but sterilization is not. 4. Achievement of developmental milestones would be included with content about childrearing practices. 5. Certain behaviors can be expected if a culture views pregnancy as a sickness, whereas other behaviors can be expected if the culture views pregnancy as a natural occurrence.

The nurse is reviewing the process of mitosis with a group of young adults attending a family planning seminar. What should the nurse include when explaining the role of mitosis? Select all that apply. 1. Replaces lost skin cells 2. Ensures rapid cell growth in early life 3. Results in the formation of sperm and ova 4. Maintains cells for respiratory functioning 5. Ensures continuity of cells in the gastrointestinal tract

Answer: 1, 2, 4, 5 Explanation: 1. Mitosis replaces cells lost daily from skin surfaces. 2. Mitosis is responsible for rapid human growth in early life. 3. Meiosis occurs only in the reproductive cells of the testes and ovaries and results in the formation of sperm and ova. 4. Mitosis is responsible for replacing cells lost daily from the lining of the respiratory tract. 5. Mitosis replaces cells lost daily from the lining of the gastrointestinal tract.

The nurse is preparing a presentation on psychosocial issues of older pregnant clients. What should the nurse include in this presentation? Select all that apply. 1. Social isolation 2. Financial issues 3. Pending mortality 4. Managing adverse effects 5. Family's and friends' attitudes

Answer: 1, 2, 3, 5Explanation: 1. Older couples facing pregnancy may feel isolated socially. They may feel different because they are often the only couple in their peer group expecting their first baby. 2. The older couple is generally more financially secure, but when their "baby" is ready for college, the older couple may be close to retirement and might not have the means to provide for their child. 3. The older couple may also be forced to face their own mortality. Older expectant parents may confront the issue earlier as they consider what will happen as their child grows. 4. Managing adverse effects would be a part of a presentation on physiologic issues of older pregnant clients. 5. The family's and friends' responses to the pregnancy may be mixed since it will affect relationships and lifestyle.

The nurse notes that the majority of clients who signed up to attend prenatal classes are over the age of 35. What should the nurse consider as reasons why these clients are this age? Select all that apply. 1. Age of marriage was later. 2. More effective birth control methods. 3. Waited until financially secure before having a family. 4. Needed to care for aging parents before having a family. 5. Wanted to become established in a career before having a family.

Answer: 1, 2, 3, 5Explanation: 1. Reasons for women choosing to have their first baby after age 35 include marrying later in life. 2. Reasons for women choosing to have their first baby after age 35 include more effective birth control methods. 3. Reasons for women choosing to have their first baby after age 35 include waiting until financially secure before having a family. 4. Caring for aging parents is not identified as a reason for women choosing to have their first baby after age 35. 5. Reasons for women choosing to have their first baby after age 35 include wanting to become established in a career before having a family.

A 38-year-old client is thrilled to learn of being pregnant with her first child. What should the nurse identify as advantages for the client having a child at this age? Select all that apply. 1. Most likely well-educated. 2. Ready to make a life change. 3. Decision was made deliberately. 4. Child care will be easier at this age. 5. Able to take on the responsibilities of a child.

Answer: 1, 2, 3, 5Explanation: 1. Single women or couples who delay childbearing until they are older tend to be well educated and financially secure. 2. Some women are ready to make a change in their lives, wanting to stay home with a new baby. 3. Usually, the decision to have a baby at an older age was deliberately and thoughtfully made. 4. Because of their greater life experiences, they also are more aware of the realities of having a child and what it means to have a baby at this age. Child care will not be easier. 5. Many of the women have experienced fulfillment in their careers and feel secure enough to take on the added responsibility of a child.

A client in labor is demonstrating acute manifestations of schizophrenia. What should the nurse identify as a priority for this client? Select all that apply. 1. Ensuring fetal well-being 2. Ensuring maternal well-being 3. Maintaining a safe environment 4. Medicating for pain as necessary 5. Considering pharmacologic intervention

Answer: 1, 2, 3, 5Explanation: 1. Some women with severe psychologic disorders may have excessive symptoms during their labor and birth. Care of these women should focus on ensuring fetal well-being. 2. Some women with severe psychologic disorders may have excessive symptoms during their labor and birth. Care of these women should focus on ensuring maternal well-being. 3. Some women with severe psychologic disorders may have excessive symptoms during their labor and birth. Care of these women should focus on maintaining a safe environment. 4. Medicating for pain would be provided for all laboring clients, not just those experiencing acute manifestations of schizophrenia. 5. Pharmacologic interventions may be necessary for excessive symptoms.

The staff development trainer is preparing content on the psychologic implications of genetic testing for a group of neonatal intensive care nurses. What should the trainer include in this instruction? Select all that apply. 1. Survivor guilt of other nonaffected children 2. Feelings of unworthiness and altered self-image 3. Carrier status interfering with relationships 4. Formation of higher expectations for the child 5. Stress of uncertainty while waiting for test results

Answer: 1, 2, 3, 5Explanation: 1. Survivor guilt may affect children with negative results if their siblings are positive. 2. A positive test result may lead to feelings of unworthiness and may disturb self-image. 3. Concerns about carrier status may interfere with development of intimacy and interpersonal relationships. 4. The parent and other family members may unconsciously form lowered expectations for the child or adolescent. 5. Uncertainty and stress associated with making a decision to undertake genetic testing may extend into weeks or even months before results are available.

A 40-year-old client, pregnant with her first child, arrives for a prenatal visit. What perinatal risk factors should the nurse keep in mind when planning this client's care? Select all that apply. 1. Miscarriage 2. Maternal death 3. Perinatal morbidity 4. Potential for multiple births 5. Maternal chronic health conditions

Answer: 1, 2, 3, 5Explanation: 1. The rate of miscarriage is higher in pregnant women over age 35. 2. The risk of maternal death is higher for women over age 35 and even higher for women age 40 and older. 3. The rate of perinatal morbidity is higher in pregnant women over age 35. 4. There is no evidence to support that pregnant women over the age of 35 or 40 have a greater potential for multiple births. 5. Women over the age of 40 and older are more likely to have chronic medical conditions that can complicate a pregnancy.

19) The nurse manager is considering adopting clinical practice guidelines to care for clients experiencing abruptio placentae. What advantages of these guidelines should the manager discuss with the nursing staff? Select all that apply. 1. Limit the cost of care. 2. Help evaluate the effectiveness of care. 3. Reduce the number of nurses needed to provide care. 4. Reduce variations when caring for clients with the same health problem. 5. Provide sequence and timing of interventions to help achieve expected client outcomes.

Answer: 1, 2, 4, 5 Explanation: 1. Clinical practice guidelines are adopted within a healthcare setting to limit costs of care. 2. Clinical practice guidelines are adopted within a healthcare setting to evaluate the effectiveness of care. 3. Clinical practice guidelines are not used for evidence to reduce the number of nurses needed to provide care. 4. Clinical practice guidelines are adopted within a healthcare setting to reduce variation in care management. 5. Clinical practice guidelines are comprehensive interdisciplinary care plans for a specific condition that describe the sequence and timing of interventions that should result in expected client outcomes.

A pregnant client is scheduled for fetal genetic testing to determine the presence of multifactorial disorders. Which health problems can occur in the fetus from this type of genetic anomaly? Select all that apply. 1. Autism 2. Asthma 3. Cleft lip 4. Scoliosis 5. Cleft palate

Answer: 1, 2, 3, 5Explanation: 1. The term multifactorial implies genetic effect of the environment on genetic expression. Most diseases and health conditions are polygenic, and the expression of those altered genes is often modified by environmental influences. Such conditions are said to be multifactorial and include the pediatric condition autism. 2. The term multifactorial implies genetic effect of the environment on genetic expression. Most diseases and health conditions are polygenic, and the expression of those altered genes is often modified by environmental influences. Such conditions are said to be multifactorial and include the pediatric condition asthma. 3. The term multifactorial implies genetic effect of the environment on genetic expression. Most diseases and health conditions are polygenic, and the expression of those altered genes is often modified by environmental influences. Such conditions are said to be multifactorial and include cleft lip. 4. Scoliosis is not identified as being a multifactorial genetic disorder. 5. The term multifactorial implies genetic effect of the environment on genetic expression. Most diseases and health conditions are polygenic, and the expression of those altered genes is often modified by environmental influences. Such conditions are said to be multifactorial and include cleft palate.

During an assessment the nurse suspects that a client is at risk for developing a cystocele. What findings did the nurse use to come to this conclusion? Select all that apply. 1. Age 68 2. Body mass index (BMI) 32 3. Pregnant seven times 4. History of diabetes 5. Takes hormone replacement therapy

Answer: 1, 2, 3Explanation: 1. Advanced age is a risk factor for the development of a cystocele. 2. Obesity is a risk factor for the development of a cystocele. 3. Childbearing is a risk factor for the development of a cystocele. 4. History of diabetes is not a risk factor for the development of a cystocele. 5. Hormone replacement therapy is not a risk factor for the development of a cystocele.

A 25-year-old client with a learning disability arrives for her first prenatal visit at week 24 of gestation. What should the nurse emphasize when caring for this client? Select all that apply. 1. Ask if the client has any questions. 2. Avoid rushing through the examination. 3. Provide information in small increments. 4. Ask who is going to financially assist the client. 5. Suggest discussing the pregnancy with social services.

Answer: 1, 2, 3Explanation: 1. The client may have a lower reading and comprehension level. Asking if the client has any questions will help with retention. 2. The client will be anxious, which can be helped by not rushing through the examination. 3. Providing information in small increments will help with comprehension. 4. The client's financial situation is inappropriate for the nurse to assess. 5. Women with intellectual disabilities are at increased risk of intervention by social services, which may result in the removal of the newborns from their custody. Discussing the pregnancy with social services will add to this client's anxiety.

The nurse is reviewing the structures of the developing fetus with a group of pregnant clients. What should the nurse emphasize about the function of the placenta? Select all that apply. 1. It produces hormones. 2. It produces glycogen, cholesterol, and fatty acids. 3. It maintains fetal respiration, nutrition, and excretion. 4. It provides a circulatory pathway from the chorionic villi to the embryo. 5. It contains a specialized mucoid connective tissue known as Wharton jelly.

Answer: 1, 2, 3Explanation: 1. The placenta produces human chorionic gonadotropin (hCG); human placental lactogen (hPL), also referred to as human chorionic somatomammotropin (hCS); relaxin; inhibin; and estrogen and progesterone. 2. The placenta produces glycogen, cholesterol, and fatty acids continuously for fetal use and hormone production. 3. The placental functions include fetal respiration, nutrition, and excretion. To carry out these functions, the placenta is involved in metabolic and transfer activities. 4. The umbilical cord provides a circulatory pathway from the chorionic villi to the embryo. 5. The umbilical cord contains a specialized mucoid connective tissue known as Wharton jelly, which prevents compression of the umbilical cord in utero.

The nurse is explaining the difference between meiosis and mitosis. Which statements should the nurse include? Select all that apply. 1. Mitosis occurs in most of the cells of the body. 2. Meiotic division leads to cells that halve the original genetic material. 3. Meiosis is the division of a cell into two exact copies of the original cell. 4. Meiosis is the process by which gametes, or the sperm and ova, are formed. 5. Mitosis is splitting one cell into two, each with half the chromosomes of the original cell.

Answer: 1, 2, 4 Explanation: 1. Mitosis is how the majority of cells reproduce so that the new cells have the same structure and function as the original. Meiosis only occurs in gametes. 2. Meiosis creates two cells that have half of the chromosomes of the original cell. 3. Meiosis creates two cells that have half of the chromosomes of the original cell. 4. Both sperm and ova are created through meiosis. 5. Mitosis creates two cells that are exact copies of the original cell.

14) A pregnant client is interested in the use of herbs during her pregnancy. How should the nurse counsel this client? Select all that apply. 1. Do not take any herbs with other medication. 2. Refer to the list to learn which herbs to avoid during pregnancy. 3. Most herbs are harmless and can be safely taken while pregnant. 4. Refer to the list to learn which herbs to avoid during breastfeeding. 5. Consult with your healthcare provider before taking any herbs, even as teas.

Answer: 1, 2, 4, 5 Explanation: 1. Certain herbs may interact with prescribed medication, and should not be used. 2. Lists identifying common herbs that women are advised to avoid or use with caution during pregnancy are available. 3. Most herbs are not harmless. They are not routinely regulated and should be treated with caution. 4. Lists identifying common herbs that women are advised to avoid or use with caution during lactation are available. 5. Pregnant and lactating women interested in using herbs are best advised to consult with their healthcare provider before taking any herbs, even as teas.

During a prenatal visit the nurse notes that a client entering the third trimester has gained a total of 8 lb. What action should the nurse take at this time? Select all that apply. 1. Assess for nausea. 2. Refer to a dietitian. 3. Suggest amniocentesis. 4. Assess nutritional intake. 5. Discuss importance of adequate weight gain.

Answer: 1, 2, 4, 5Explanation: 1. The nurse should assess for reasons that may restrict the client's intake, such as nausea. 2. The nurse should refer the client to a dietitian for nutritional teaching. 3. An amniocentesis is not indicated at this time. 4. The nurse should discuss the importance of adequate nutritional intake. 5. The nurse should discuss the importance of adequate weight gain to support the developing fetus.

16) The nurse is establishing criteria for a medical or healthcare home for children. What should the nurse include when planning this approach to care? Select all that apply. 1. Providers partner with the family. 2. Children are known by the provider. 3. Home visits are made when necessary. 4. Specialty care can be accessed if necessary. 5. Communication with family occurs routinely.

Answer: 1, 2, 4, 5 Explanation: 1. Criteria for a medical or healthcare home for children include partnering with the family in the child's care. 2. Criteria for a medical or healthcare home for children include being well known by a physician or nurse who provides the usual source of sick care. 3. Home visits are a part of home care. 4. Criteria for a medical or healthcare home for children include having access to specialty care. 5. Criteria for a medical or healthcare home for children include spending adequate time communicating clearly with the family.

A middle school student comes to the nurse's office concerned about her menstrual flow. What should the nurse include when discussing menstruation with this student? Select all that apply. 1. A tub bath is helpful to promote blood flow and reduce cramping. 2. Show a variety of pads and tampons available for the student to use. 3. A fishy odor experienced during menstruation is expected and normal. 4. Demonstrate that the volume of blood lost during menstruation is about 1 to 2 ounces. 5. Wash the hands before and after using any feminine hygiene products for menstruation.

Answer: 1, 2, 4, 5Explanation: 1. A long, leisurely soak in a warm tub promotes menstrual blood flow and relieves cramps by relaxing the muscles. 2. If working with teens and preteens, keeping a variety of pads and tampons on hand helps these young girls become familiar with the options available for dealing with menstruation. 3. Conditions such as vaginitis produce a foul-smelling discharge that women often describe as having a "fishy" odor. This is not expected or normal. 4. The average flow is approximately 25 to 60 mL per period. 5. Because of the potential for developing an infection, a woman should wash her hands before inserting a fresh tampon and should avoid touching the tip of the tampon when unwrapping it or before insertion. Hand washing should also occur after using the commode.

The nurse is reviewing with a pregnant client the skin changes that she might experience during gestation. What should the nurse include in this discussion? Select all that apply. 1. Linea nigra 2. Spider nevi 3. Psoriatic lesions 4. Striae gravidarum 5. Melasma gravidarum

Answer: 1, 2, 4, 5Explanation: 1. Changes in skin pigmentation occurring during pregnancy are thought to be stimulated by increased estrogen, progesterone, and α-melanocytic-stimulating hormone levels. The skin in the middle of the abdomen may develop a pigmented line, the linea nigra, which usually extends from the pubic area to the umbilicus or higher. 2. Vascular spider nevi, small, bright-red elevations of the skin radiating from a central body, may develop on the chest, neck, face, arms, and legs. They may be caused by increased subcutaneous blood flow in response to elevated estrogen levels. 3. Psoriatic lesions are not typically associated with pregnancy. 4. Striae gravidarum, or stretch marks, may appear on the abdomen, thighs, buttocks, and breasts. They result from reduced connective tissue strength because of elevated adrenal steroid levels. 5. Changes in skin pigmentation occurring during pregnancy are thought to be stimulated by increased estrogen, progesterone, and α-melanocytic-stimulating hormone levels. Melasma gravidarum, also known as the "mask of pregnancy," a darkening of the skin over the forehead and around the eyes, may develop. Melasma is more prominent in dark-haired women and is aggravated by exposure to the sun.

A client is suspected of having a hydatidiform mole. What should the nurse expect to assess in this client? Select all that apply. 1. Elevated blood pressure 2. Absence of fetal heart tones 3. Frequent urination and thirst 4. Dark brown vaginal drainage 5. Larger than gestational age fundal height

Answer: 1, 2, 4, 5Explanation: 1. Manifestations of preeclampsia are associated with a hydatidiform mole, which would include an elevated blood pressure. 2. Fetal heart sounds are absent with a hydatidiform mole because a fetus is not developing in the uterus. 3. Frequent urination and thirst are not manifestations of hydatidiform mole. 4. Dark brown vaginal discharge, similar to prune juice, occurs because of liquefaction of the uterine clot. 5. Uterine enlargement greater than expected for gestational age is a classic sign of a complete mole, which is present in about half of cases. Enlargement is due to the proliferating trophoblastic tissue and to a large amount of clotted blood.

During a health interview the nurse determines that a client contemplating pregnancy would benefit from teaching on preconception health measures. What information did the nurse use to make this clinical determination? Select all that apply. 1. Smokes one half of a pack per day of cigarettes 2. Works in an industrial plant 3. Drinks decaffeinated coffee 4. Drinks 4 ounces red wine every evening 5. Takes over-the-counter antacids as needed

Answer: 1, 2, 4, 5Explanation: 1. One preconception health measure is smoking cessation. 2. One preconception health measure is identifying environmental hazards. 3. Avoiding caffeine several months before conception is a preconception health measure. 4. One preconception health measure is alcohol intake. 5. One preconception health measure is the use of over-the-counter medications.

A pregnant client is in a motor vehicle crash and needs surgery to repair a fractured lower leg. What special precautions will this client need during and after the surgery? Select all that apply. 1. Prepare for intubation. 2. Insert a nasogastric tube. 3. Maintain on strict bed rest. 4. Insert an indwelling urinary catheter. 5. Apply sequential compression devices (SCDs).

Answer: 1, 2, 4, 5Explanation: 1. Pregnancy causes increased secretions of the respiratory tract and engorgement of the nasal mucous membrane, often making breathing through the nose difficult. Consequently, pregnant women often need an endotracheal tube to maintain an airway during surgery. 2. The decreased intestinal motility and delayed gastric emptying that occur in pregnancy increase the risk of vomiting when anesthetics are given and during the postoperative period. A nasogastric tube may be recommended before major surgery. 3. Exercises in bed should be encouraged along with early ambulation after surgery. 4. An indwelling urinary catheter prevents bladder distention, decreases risk of injury to the bladder, and permits monitoring of output. 5. SCDs during and after surgery help prevent venous stasis and the development of thrombophlebitis.

A 34-year-old female client has been diagnosed with recurrent pregnancy loss (RPL). Which of the following statements accurately represents this condition? a. RPL is not related to infertility. b. Research has demonstrated that there is no link between RPL and autoimmune disorders. c. RPL is defined by two or more failed pregnancies. d. The cause for RPL can be diagnosed in 90% of cases

Answer: c. RPL is defined as having two or more failed pregnancies. Feedback: RPL is a form of infertility. It may be related to maternal autoimmune disorders. The etiology cannot be determined in 50% of cases.

A client who is at 28 weeks' gestation arrives for her first prenatal examination in a wheelchair. What should the nurse include when assessing this client? Select all that apply. 1. Signs of physical abuse 2. Evidence of mental abuse 3. Teaching about cesarean birth 4. Reason why prenatal care was delayed 5. Language that would hint toward financial abuse

Answer: 1, 2, 4, 5Explanation: 1. Women with disabilities are at greater risk of being victimized and of sustaining intimate partner violence. Women who rely on their partner for assistance with activities of daily living are at risk for physical abuse. 2. Women with disabilities are at greater risk of being victimized and of sustaining intimate partner violence. Women who rely on their partner for assistance with activities of daily living are at risk for mental abuse. 3. Pregnant women who use a wheelchair will not necessarily need a cesarean birth. 4. Women with disabilities are at greater risk of being victimized and of sustaining intimate partner violence. Women who rely on their partner for assistance with activities of daily living are at risk for having care withheld. 5. Women with disabilities are at greater risk of being victimized and of sustaining intimate partner violence. They are also at risk for financial abuse.

The nurse notes that a client who is 10 weeks pregnant is experiencing changes to the upper respiratory system. What should the nurse explain as being the reason for these changes? Select all that apply. 1. Estrogen-induced edema 2. Hypersecretion of mucus 3. Decreased white blood cell production 4. Vascular congestion of the nasal mucosa 5. Amniotic fluid reducing total fluid volume

Answer: 1, 2, 4Explanation: 1. Upper respiratory changes in the form of nasal stuffiness and epistaxis may occur because of estrogen-induced edema. 2. Upper respiratory changes in the form of nasal stuffiness and epistaxis may occur because of hypersecretion of mucus. 3. There is no change in white blood cell production. 4. Upper respiratory changes in the form of nasal stuffiness and epistaxis may occur because of vascular congestion of the nasal mucosa. 5. Amniotic fluid is not reducing the client's total fluid volume.

What is the role of the certified nurse-midwife (CNM)? Select all that apply. 1. Give primary care for healthy newborns. 2. Be educated in two disciplines of nursing. 3. Give primary care for high-risk clients who are in hospital settings. 4. Obtain a physician consultation for all technical procedures at delivery. 5. Be prepared to manage independently the care of women at low risk for complications during pregnancy and birth.

Answer: 1, 2, 5 Explanation: 1. The CNM is prepared to manage independently the care of women at low risk for complications during pregnancy and birth and the care of healthy newborns. 2. The CNM is educated in the disciplines of nursing and midwifery. 3. CNMs cannot give primary care for high-risk clients who are in hospital settings. The physician provides the primary care. 4. The CNM does not need to obtain a physician consultation for all technical procedures at delivery. Situations in which the client is at risk, such as for a 4th-degree laceration or forceps delivery, would need physician consultation. 5. The CNM is prepared to manage independently the care of women at low risk for complications during pregnancy and birth and the care of healthy newborns.

Through genetic testing a homosexual male learns of having two copies of a gene deleted in gene CCR5. What should the nurse prepare to discuss with this client? Select all that apply. 1. Safe sexual practices 2. Reduced risk of contracting HIV 3. Increased risk of contracting HIV 4. Expectation to develop AIDs quickly 5. Delayed conversion of HIV to AIDS

Answer: 1, 2Explanation: 1. Even though persons who have two copies of the altered CCR5 gene are almost completely resistant to infection with HIV type 1, the client should still be reminded of safe sexual practices. 2. Persons who have two copies of the altered CCR5 gene are almost completely resistant to infection with HIV type 1. 3. The client does not have an increased risk for contracting HIV. 4. Two copies of the altered CCR5 gene will not accelerate the development of AIDS from HIV. 5. Clients who have one copy of the altered CCR5 gene will have delayed conversion of HIV to AIDS.

A 38-year-old pregnant client is reluctant to attend prenatal classes because the other participants will be much younger. What should the nurse do to encourage this client's attendance at classes? Select all that apply. 1. Support the client's strengths. 2. Find a class with participants that are older. 3. Encourage the client to read prenatal training material online. 4. Nothing, since the client most likely has already researched the birthing process. 5. Prepare a list of reasons for the late pregnancy to use as responses when others ask.

Answer: 1, 2Explanation: 1. Even though women who are over age 35 and having their first baby tend to be better educated than other healthcare consumers, it should not be assumed that anticipatory guidance and support are not needed. Instead, support the client's strengths and be sensitive to her individual needs. 2. Older expectant parents often feel uncomfortable in classes in which most of the participants are much younger. Because of this, classes for expectant parents over age 35 are now available in many communities. 3. The client needs support and not just information that can be read over the Internet. 4. The older client may be better informed. However, she still needs support. 5. There is no reason for the client to have a list of excuses for delaying pregnancy.

The nurse is preparing teaching for a pregnant client with a history of preterm labor. What information should the nurse specifically provide regarding sexual activity? Select all that apply. 1. Avoid intercourse. 2. Avoid nipple stimulation. 3. Avoid intercourse if vaginal bleeding occurs. 4. Avoid intercourse after the membranes rupture. 5. There are no restrictions to intercourse during pregnancy.

Answer: 1, 2Explanation: 1. Women with a history of preterm labor may be advised to avoid intercourse because the oxytocin that is released with orgasm stimulates uterine contractions and may trigger preterm labor. 2. Because oxytocin is also released with nipple stimulation, fondling the breasts may also be contraindicated in women with a history of preterm labor. 3. All pregnant women should be instructed to avoid intercourse if vaginal bleeding occurs. 4. All pregnant women should be instructed to avoid intercourse after the membranes rupture. 5. Women with a history of preterm labor may be advised to avoid intercourse because the oxytocin that is released with orgasm stimulates uterine contractions and may trigger preterm labor.

The nurse is reviewing a list of families scheduled for clinic visits. Which family would benefit from genetic counseling? 1. Family who has a child with cystic fibrosis 2. Family whose youngest child is overweight 3. Family whose oldest child wears eyeglasses 4. Family who has a neighbor with Down syndrome

Answer: 1Explanation: 1. Any couple who has had a child with a chromosomal abnormality may be at increased risk of having another child similarly affected. 2. There is no evidence to support that weight issues are caused by genetic anomalies. 3. Wearing eyeglasses is not a genetic anomaly. 4. Having a neighbor with Down syndrome has no effect on the family's genetic background.

The nurse suspects that a pregnant client will be scheduled for chorionic villi sampling (CVS). What assessment data did the nurse use to make this clinical determination? Select all that apply. 1. Client age 39 years 2. Client carrying twins 3. Fetus at 10 weeks' gestation 4. Client has an X-linked disorder 5. Client has a history of smoking

Answer: 1, 3, 4 Explanation: 1. Invasive diagnostic testing is frequently targeted to women who are over the age of 35. 2. Number of fetuses is not an indication for invasive diagnostic testing. 3. Chorionic villi sampling can be performed after 9 weeks' gestation. 4. Invasive diagnostic testing is frequently targeted to women who have an X-linked disorder. 5. Smoking history is not an indication for invasive diagnostic testing.

he nurse is preparing an in-service presentation about the role of culture in adolescent pregnancy. What should the nurse include in this presentation? Select all that apply. 1. "Teens with future goals tend to use birth control more consistently." 2. "Most pregnant teens do not have any relatives who had their first child as teens." 3. "Young teens who have a child are more likely to have another while still a teen." 4. "Although the rate has dropped, non-Caucasian teens are more likely to become pregnant." 5. "Eighty-five percent of teen mothers are middle class, and give birth to gain adult status."

Answer: 1, 3, 4 Explanation: 1. Teens with future goals such as college or a job tend to use birth control more consistently compared with other teens; if they become pregnant, they are also more likely to have abortions. 2. Having a mother or a sister who had her first child during adolescence is a risk for a teen to become pregnant. 3. When the first birth occurs in the early teen years, the next birth also is likely to occur prior to adulthood. 4. In the United States, the adolescent birth rate is higher among African American teens and Hispanic teens than among Caucasian teens. 5. When teens in poverty become pregnant, they are more likely to maintain the pregnancy and view the birth as a way to be seen as an adult. Middle-class teens are more likely to have future education and career goals, use contraception, and seek therapeutic abortion if they become pregnant.

8) The nurse is coordinating a clinical research trial with pediatric clients. From which clients should the nurse seek assent to participate in the research? Select all that apply. 1. A 9-year-old client who qualifies to test a medication for muscular dystrophy 2. A precocious 4-year-old starting as a participant in a cystic fibrosis research study 3. A 10-year-old starting in an investigative study for clients with precocious puberty 4. A 7-year-old client with leukemia who has elected to receive a newly developed trial medication 5. A 13-year-old client beginning participation in a research program for attention deficit/hyperactivity disorder (ADHD) treatments

Answer: 1, 3, 4, 5 Explanation: 1. Federal guidelines mandate that research participants 7 years old and older must receive developmentally appropriate information about healthcare procedures and treatments, and give assent. 2. The 4-year-old patient would qualify since the age of assent is 7 years old. 3. Federal guidelines mandate that research participants 7 years old and older must receive developmentally appropriate information about healthcare procedures and treatments, and give assent. 4. Federal guidelines mandate that research participants 7 years old and older must receive developmentally appropriate information about healthcare procedures and treatments, and give assent. 5. Federal guidelines mandate that research participants 7 years old and older must receive developmentally appropriate information about healthcare procedures and treatments, and give assent.

6) The nurse is evaluating telephone calls made by the mothers of newborns in a community clinic. Which calls should the nurse make a priority? Select all that apply. 1. Mother who is 16 years old 2. Mother who is breastfeeding 3. Mother who is a single parent 4. Mother who gave birth to twins 2 weeks ago 5. Mother whose baby was born at 30 weeks' gestation

Answer: 1, 3, 4, 5 Explanation: 1. Infant mortality rates are higher for infants of teen mothers. 2. There are no data to support the mortality rate of infants who are being breastfed. 3. Infant mortality rates are higher among unmarried mothers. 4. Infant mortality rates are higher among infants born in multiple births. 5. Infant mortality rates are higher among infants born prematurely.

18) The nurse is caring for clients in a major urban community health clinic. For which cultural groups should the nurse make assessing for lactose intolerance a priority? Select all that apply. 1. Asians 2. Mediterranean descent 3. American Indians 4. African Americans 5. Mexican Americans

Answer: 1, 3, 4, 5 Explanation: 1. Lactose intolerance is common in Asians. 2. People of Mediterranean descent are not identified as having lactose intolerance. 3. Lactose intolerance is common in American Indians. 4. Lactose intolerance is common in African Americans. 5. Lactose intolerance is common in Mexican Americans.

A client in the second trimester of pregnancy is experiencing severe heartburn. What should the nurse explain about the reasons for this health problem? Select all that apply. 1. Decreased gastrointestinal motility 2. Changes in carbohydrate metabolism 3. Increased production of progesterone 4. Relaxation of the esophageal sphincter 5. Displacement of the stomach by the enlarging uterus

Answer: 1, 3, 4, 5Explanation: 1. Decreased gastrointestinal motility contributes to heartburn. 2. Changes in carbohydrate metabolism contribute to the development of nausea and vomiting. 3. The increased production of progesterone in pregnancy contributes to heartburn. 4. Relaxation of the esophageal sphincter contributes to heartburn. 5. Heartburn during pregnancy appears to be primarily a result of the displacement of the stomach by the enlarging uterus.

A female client with infertility is suspected as having a problem with patent fallopian tubes and ovum motility. For which health problems should the nurse expect that this client will be evaluated? Select all that apply. 1. Endometriosis 2. Cervical stenosis 3. Ectopic pregnancy 4. Peritubal adhesions 5. Pelvic inflammatory disease

Answer: 1, 3, 4, 5Explanation: 1. Fallopian tube function and motility can be affected by endometriosis. 2. Cervical stenosis affects cervical mucus. 3. Fallopian tube function and motility can be affected by an ectopic pregnancy. 4. Fallopian tube function and motility can be affected by peritubal adhesions. 5. Fallopian tube function and motility can be affected by pelvic inflammatory disease.

From genetic testing a client learns of having a Y-linked genetic disorder. Which health problem would be explained by this finding? 1. Infertility 2. Hemophilia A 3. Beta-thalassemia 4. Tay-Sachs disease

Answer: 1Explanation: 1. Because the Y chromosome has very few genes, alterations on the Y chromosome are not often associated with health problems. The Y chromosome does contain genes associated with spermatogenesis, and alterations in those genes can cause male infertility. 2. Hemophilia A is an X-linked recessive disorder. 3. Beta-thalassemia is an autosomal recessive disorder. 4. Tay-Sachs disease is an autosomal recessive disorder.

The nurse is preparing prenatal teaching material to support the learning needs of a pregnant client with a learning disability. What should the nurse keep in mind when preparing these materials? Select all that apply. 1. Provide with videotapes. 2. Provide web site addresses. 3. Allow for extra teaching time. 4. Select an easy-to-understand format. 5. Audiotape information from prenatal classes

Answer: 1, 3, 4, 5Explanation: 1. Good communication aids such as videotapes should be used. 2. The client has a learning disability and most likely will not be using a computer. 3. Extra teaching time should be provided to ensure all learning needs are met. 4. Easy-to-understand information will facilitate learning. 5. Good communication aids such as audiotapes from prenatal classes should be used.

The community nurse is providing prenatal care to a client in the home who has been unable to receive traditional prenatal care. Which barriers should the nurse consider when caring for this client? Select all that apply. 1. Location of healthcare facilities 2. View that prenatal care is insignificant 3. Lack of transportation to healthcare facilities 4. Appointment schedule conflicting with work hours 5. Lack of support to care for other children while attending prenatal appointments

Answer: 1, 3, 4, 5Explanation: 1. Home care is especially effective in removing barriers for women who have difficulty accessing health care. A lack of locally available healthcare facilities is a barrier. 2. Viewing that prenatal care is insignificant is not identified as a barrier to a woman having difficulty accessing health care. 3. Home care is especially effective in removing barriers for women who have difficulty accessing health care. Lack of transportation to healthcare facilities is a barrier. 4. Home care is especially effective in removing barriers for women who have difficulty accessing health care. Appointment schedule that conflicts with work hours is a barrier. 5. Home care is especially effective in removing barriers for women who have difficulty accessing health care. Lack of support to care for other children while attending prenatal appointments is a barrier.

The nurse is preparing teaching material on chorionic villus sampling (CVS) for a client who is entering the 10th week of gestation. What risks should the nurse include with this material? Select all that apply. 1. Bleeding 2. Embryonic puncture 3. Intrauterine infection 4. Inability to obtain a tissue sample 5. Inadvertent rupture of the membranes

Answer: 1, 3, 4, 5Explanation: 1. Risks of CVS include bleeding. 2. Embryonic puncture is not a risk associated with CVS. 3. Risks of CVS include intrauterine infection. 4. Risks of CVS include failure to obtain tissue. 5. Risks of CVS include rupture of the membranes.

The nurse is instructing a newly pregnant client who follows a vegan eating plan about folic acid. What should the nurse include when teaching the client about this nutrient? Select all that apply. 1. The best sources are fresh green leafy vegetables. 2. Overcooking foods high in folic acid is preferred. 3. Foods high in folic acid should be protected from light. 4. Cook foods high in folic acid with small amounts of water. 5. Peanuts and whole-grain breads and cereals are good sources.

Answer: 1, 3, 4, 5Explanation: 1. Sources for folic acid include fresh green leafy vegetables. 2. To prevent unnecessary loss, foods with folic acid should not be overcooked. 3. To prevent unnecessary loss, foods with folic acid should be stored covered to protect them from light. 4. To prevent unnecessary loss, foods with folic acid should be cooked with only a small amount of water. 5. Sources of folic acid include peanuts and whole-grain breads and cereals, which are all appropriate for the vegan eating plan.

A client is diagnosed with a uterine prolapse. Which treatments should the nurse prepare to discuss with this client? Select all that apply. 1. Hysterectomy 2. Kegel exercises 3. Vaginal pessary 4. Topical estrogen 5. Systemic estrogen

Answer: 1, 3, 4, 5Explanation: 1. Surgery for uterine prolapse often involves hysterectomy and repair of the prolapsed vaginal walls. 2. Kegel exercises are indicated in the treatment of a cystocele. 3. Conservative treatment for a uterine prolapse includes a vaginal pessary. 4. Conservative treatment for a uterine prolapse includes topical estrogen. 5. Conservative treatment for a uterine prolapse includes systemic estrogen.

A client tells the nurse that she is unable to have routine health care because of the lack of health insurance. How should the nurse respond regarding the Affordable Care Act? Select all that apply. 1. There is no annual limit on insurance coverage. 2. Requires enrollment to occur when a specific age is reached. 3. Health insurance can be obtained with pre-existing medical conditions. 4. Provides a tax credit for middle- and low-income families to cover a part of the cost. 5. Young adults can be covered under parents' health insurance for longer periods of time.

Answer: 1, 3, 4, 5Explanation: 1. The Affordable Care Act eliminates annual limits on insurance coverage. 2. Enrollment when a specific age is reached is a characteristic of Medicare. 3. The Affordable Care Act ends pre-existing condition exclusions for children. 4. The Affordable Care Act provides more affordable health insurance options including tax credits for middle- and low-income families. These credits cover a major portion of the cost. 5. The Affordable Care Act keeps young adults covered by their parents' healthcare insurance for a longer period.

16) The nurse is preparing to assess a client who is from a non-English-speaking culture. What should the nurse review about the client's culture before beginning the assessment? Select all that apply. 1. Touch 2. Employment 3. Personal space 4. Physical differences 5. Use of nonverbal communication

Answer: 1, 3, 4, 5Explanation: 1. The appropriateness of touch varies with each culture. 2. Employment is not culturally determined. 3. An individual's sense of personal space differs by culture. 4. Genetic and physical differences occur among cultural groups and can lead to disparity in needs and care. 5. Depending upon the culture, gestures and body language may be misunderstood or misinterpreted.

After genetic testing, a client is suspected of having mitochondrial gene alteration. When reviewing the test result with the client, which body systems are noted as being affected by this alteration? Select all that apply. 1. Brain 2. Renal 3. Cardiac 4. Genitourinary 5. Musculoskeletal

Answer: 1, 3, 5 Explanation: 1. Clinical manifestations occurring as a result of mitochondrial gene alterations primarily affect high-energy tissues such as the brain. 2. Clinical manifestations occurring as a result of mitochondrial gene alterations primarily affect high-energy tissues. The renal system is not identified as being affected. 3. Clinical manifestations occurring as a result of mitochondrial gene alterations primarily affect high-energy tissues such as cardiac muscle. 4. Clinical manifestations occurring as a result of mitochondrial gene alterations primarily affect high-energy tissues. The genitourinary system is not identified as being affected. 5. Clinical manifestations occurring as a result of mitochondrial gene alterations primarily affect high-energy tissues such as skeletal muscle.

13) The nurse is reviewing a client's use of complementary and alternative therapies. Which situations should the nurse identify as being risky for the client? Select all that apply. 1. Trying out a homeopathic medicine obtained from a friend to reduce swelling in the legs 2. Joining a group that practices tai chi weekly to help with physical fitness and movement 3. Taking an herbal preparation suggested by a health food store worker for treatment of leg pain 4. Getting a massage from a licensed massage therapist for back pain, when such treatment has been prescribed by the primary healthcare giver 5. Getting a chiropractic treatment for lower back pain due to discomforts of pregnancy without telling the primary healthcare provider

Answer: 1, 3, 5 Explanation: 1. Lack of standardization, lack of regulation and research to substantiate their safety and effectiveness, and inadequate training and certification of some healers make some therapies risky. Trying out a homeopathic medicine obtained from a friend to reduce swelling in the legs is a risk factor when considering these therapies. 2. Joining a group that practices tai chi weekly to help with physical fitness and movement is a perfectly good use of complementary therapies. 3. Lack of standardization, lack of regulation and research to substantiate their safety and effectiveness, and inadequate training and certification of some healers make some therapies risky. Taking an herbal preparation suggested by a health food store worker for treatment of leg pain is a risk factor when considering these therapies. 4. Getting a massage from a licensed massage therapist for back pain, when such treatment has been prescribed by the primary healthcare giver, is a perfectly good use of complementary therapies. 5. Lack of standardization, lack of regulation and research to substantiate their safety and effectiveness, and inadequate training and certification of some healers make some therapies risky. Getting a chiropractic treatment for lower back pain due to discomforts of pregnancy without telling the primary healthcare provider is a risk factor when considering these therapies.

A client asks why a genetic pedigree is being prepared. What responses would be appropriate? Select all that apply. 1. Illustrates patterns of inheritance 2. Reduces the cost for genetic diagnostic testing 3. Provides guidance for reproductive risk teaching 4. Identifies appropriate diagnostic tests for the client 5. Identifies family members who would benefit from genetic counseling

Answer: 1, 3, 5Explanation: 1. A pedigree can illustrate patterns of inheritance. 2. A pedigree does not affect the cost of diagnostic testing. 3. On the basis of the pedigree, reproductive risk teaching for the individual and family can occur. 4. A pedigree will not identify appropriate diagnostic tests for the client. 5. A pedigree identifies family members who might benefit from a genetic consultation.

The nurse provides teaching on the signs of impending labor with a client at week 37 of gestation. Which client statements indicate that additional teaching would be beneficial? Select all that apply. 1. "A bloody show means labor will begin within 2 days." 2. "Having uterine contractions that increase over time is a sign of labor." 3. "Uterine contractions that do not radiate to the back are a sign of labor." 4. "Having a spontaneous flow of water from my vagina is a sign of labor." 5. "Expulsion of a plug of mucus means the baby has dropped into the pelvis."

Answer: 1, 3, 5Explanation: 1. Bloody show is a sign of impending labor, which will begin before 2 days. 2. Uterine contractions that increase over time are a sign of impending labor. 3. Uterine contractions that do not radiate to the back describe false labor. 4. Spontaneous rupture of membranes is a sign of impending labor. 5. Expulsion of a mucous plug is a sign of impending labor and not an indication that the fetus has dropped into the pelvis.

The manager notes that a neonatal intensive care unit (NICU) nurse is practicing culturally competent care. What did the manager observe to make this decision? Select all that apply. 1. Respects the rituals of the ethnic group of a new mother 2. Explains the processes that are followed in an American hospital 3. Speaks a few phrases in the language of a non-English-speaking client 4. Discusses the odd practices that a client from Europe wants to have done 5. Contacts an interpreter to facilitate communication with a Spanish-speaking client

Answer: 1, 3, 5Explanation: 1. Evidence of cultural competence includes respecting the rituals of the client's ethnic group. 2. Explaining the processes to be followed in an American hospital can be perceived as being ethnocentric behavior. 3. Evidence of cultural competence includes learning the language, or at least several key phrases, of at least one of the cultural groups with whom the nurse interacts. 4. Discussing a non-American client's odd practices could be perceived as being ethnocentric behavior. 5. Evidence of cultural competence includes providing for the services of an interpreter if a language barrier exists.

The nurse is evaluating the effectiveness of prenatal education provided to the spouse of a pregnant client. Which observations indicate that this education was effective? Select all that apply. 1. Client is relaxed. 2. Client and spouse are arguing. 3. Spouse is looking forward to the birth. 4. Spouse expresses fear of being a father. 5. Client is following prenatal recommendations.

Answer: 1, 3, 5Explanation: 1. Research indicates that increased focus on the father's needs during prenatal care improves the mother's stress levels. 2. Arguing could indicate that the spouse is not transitioning to fatherhood and will affect the client's stress levels. 3. Research indicates that increased focus on the father's needs during prenatal care aids his transition to fatherhood. 4. Fear of being a father indicates that prenatal education was not effective in helping the spouse transition to fatherhood. 5. Research indicates that increased focus on the father's needs during prenatal care improves the mother's prenatal health behavior.

A client at 20 weeks' gestation is scheduled for a transabdominal ultrasound. What should the nurse instruct this client about the examination? Select all that apply. 1. "The entire procedure takes between 20 and 30 minutes." 2. "Arrive 30 minutes before the examination so pain medication will be effective." 3. "Transmission gel will be spread over the abdomen during the examination." 4. "Use an over-the-counter enema to empty the colon before the examination." 5. "Drink 1.5 quarts of water 2 hours before the exam and refrain from voiding."

Answer: 1, 3, 5Explanation: 1. Ultrasound testing takes 20 to 30 minutes. 2. Ultrasound is a painless noninvasive diagnostic test. No anesthesia or pain medication is required prior to the procedure. 3. Transmission gel is generously spread over the client's abdomen, and the sonographer slowly moves a transducer over the abdomen to obtain a picture of the uterine contents. 4. It is not necessary to have an empty colon for an ultrasound. 5. The bladder must be full for an ultrasound. The client should be instructed to drink 1 to 1.5 quarts of water 2 hours before the examination and refrain from voiding to ensure a full bladder.

A genetic screening test completed on a newborn is found to be positive. What should the nurse anticipate the next action will be for this infant? 1. Scheduling for diagnostic testing 2. Discussing the need for major intervention 3. Nothing, since further action is not necessary 4. Instructing the parents on expression of the genetic disorder

Answer: 1Explanation: 1. Each positive screening test must be followed by a diagnostic test. 2. Major intervention cannot be identified until a diagnosis is made. 3. Even if a false-positive occurs, the newborn will need diagnostic testing. 4. Until a diagnosis is made, teaching about genetic expression of the disorder cannot be completed.

A pregnant client with rheumatoid arthritis arrives for a prenatal examination. How should the nurse support this client's needs? Select all that apply. 1. Assist with positioning on the examination table. 2. Repeat teaching instructions slowly and succinctly. 3. Assist with changing clothing for the examination. 4. Permit the client privacy while preparing for the examination. 5. Recommend delaying an examination until later in the pregnancy.

Answer: 1, 3Explanation: 1. A client with arthritis might find examination positions uncomfortable. The nurse should assist this client with positioning. 2. There is no need to repeat teaching instructions slowly and succinctly because the client does not have a learning disability. 3. A client with arthritis might need assistance changing into an examination gown. 4. An adolescent client would want privacy, more so than the client with a physical disability. 5. The pregnant client with a disability should have the same amount and degree of prenatal care. Delaying examinations is not appropriate to suggest.

The screening results of a quadruple screening completed on a 37-year-old pregnant client were not within the normal range. For which additional testing should the nurse prepare this client? Select all that apply. 1. Ultrasound 2. Serum glucose 3. Amniocentesis 4. Serum ferritin levels 5. Fetal heart monitoring

Answer: 1, 3Explanation: 1. If the screening results are not in the normal range, follow-up testing using ultrasound is often indicated. 2. Serum glucose level is not used to detect fetal genetic abnormalities. 3. If the screening results are not in the normal range, follow-up testing using amniocentesis is often indicated. 4. Serum ferritin levels would measure iron level and will not detect fetal genetic abnormalities. 5. Fetal heart monitoring will not detect fetal genetic abnormalities.

The nurse is reviewing amniocentesis with a pregnant client. In which order should the nurse explain the steps that will occur during this procedure? 1. Conduct an ultrasound 2. Fetal heart rate assessed 3. Local anesthetic provided 4. Skin cleansed with antiseptic solution 5. Insertion site observed for fluid streaming 6. 22-gauge needle inserted to withdraw amniotic fluid

Answer: 1, 4, 3, 6, 5, 2 Explanation: 1. An ultrasound is performed first to identify amniotic fluid pockets. 2. Fetal heart rate is assessed last. 3. A local anesthetic is provided after the skin is cleansed. 4. Skin is cleansed after the ultrasound. 5. Fluid streaming occurs after the 22-gauge needle is removed. 6. A 22-gauge needle is inserted after the local anesthetic.

The nurse is caring for a pregnant client who has scoliosis that has affected sensation below the level of the umbilicus. What should the nurse instruct the client to do, to reduce the risk of adverse effects during pregnancy? Select all that apply. 1. Walk slowly and deliberately. 2. Limit the amount of daily exercise. 3. Eat fewer calories to restrict weight gain. 4. Ingest adequate amounts of fruits, vegetables, and water. 5. Review signs of pending labor other than uterine contractions.

Answer: 1, 4, 5Explanation: 1. Pregnancy may shift the center of gravity. For the client with lumbar scoliosis, this could cause the client to have difficulty maintaining balance with walking. Walking slowly and deliberately will reduce the risk of falling. 2. There is no reason for the client to restrict the amount of daily exercise. 3. Restricting weight gain while pregnant is not healthy for the client or fetus. 4. Ingesting adequate amounts of fruits, vegetables, and water will prevent the development of constipation, which can occur when nervous innervation to the lower abdomen is affected. 5. The client has reduced sensation below the level of the umbilicus and may not recognize or feel uterine contractions associated with pending labor. The nurse needs to review the other signs of pending labor, such as spontaneous rupture of membranes and bloody show.

The nurse is preparing a teaching guide to be used for prenatal, pregnancy, and postpartum nutritional classes. Which nutrients should the nurse emphasize that remain consistent throughout these gestational periods? Select all that apply. 1. Fat 2. Iron 3. Protein 4. Vitamin K 5. Pyridoxine

Answer: 1, 4Explanation: 1. Fat requirements are unchanged during pregnancy and should account for about 20% to 35% of daily caloric intake, of which 10% or less should be saturated fat. 2. Iron requirements increase during pregnancy because of the growth of the fetus and placenta and the increased maternal blood volume. 3. The protein requirement for a pregnant woman is 60 g/day, an increase of 14 g over nonpregnant levels. 4. The recommended daily allowance (RDA) for vitamin K, 90 mcg per day, does not increase during pregnancy. 5. The recommended daily allowance (RDA) during pregnancy for pyridoxine is 1.9 mg/day, an increase of 0.6 mg over the allowance for nonpregnant women.

The nurse is planning a group session for parents who are beginning infertility evaluation. Which statements should be included in this session? Select all that apply. 1. "Infertility can be stressful for a marriage." 2. "Taking a vacation usually results in pregnancy." 3. "Your insurance will pay for the infertility treatments." 4. "The doctor will be able to tell why you have not conceived." 5. "Keep communicating with one another through this process."

Answer: 1, 5 Explanation: 1. Infertility is often stressful on a marriage, as a result of the need to schedule intercourse and pay for treatments and the societal expectation to have children. 2. A common myth is that taking a vacation or just relaxing will result in conception. 3. Insurance often does not pay for infertility treatment. 4. Some infertility cannot be explained, despite extensive treatments. 5. Communication is important to help cope with stress. A nurse should always encourage clients to ask questions.

A client who became pregnant at a weight of 60 kg and a BMI of 16 has gained 4 kg at the end of the 5th gestational month. How many more pounds of weight should the nurse counsel this client to gain to achieve the least amount of weight that an underweight person should gain while pregnant?

Answer: 19.2 lb Explanation: For a client with a BMI less than 18.5, the weight gain for the total pregnancy should be between 28 and 40 lb. The client has gained 4 kg, or 8.8 lb. To achieve the minimal amount of weight that the client should gain during the entire pregnancy, subtract the amount of weight gained from the minimal amount, or 28 lb - 8.8 lb = 19.2 lb.

A newly pregnant client weighs 75.0 kg and has a body mass index of 28.5. What is the maximum amount of weight in pounds that this client should weigh during the pregnancy?

Answer: 190 lb Explanation: For a client with a BMI between 25.0 and 29.9, the weight gain for the total pregnancy should be between 15 and 25 lb. This client's weight is 75 kg. To determine the maximum amount of weight to gain, first determine the client's weight in pounds by multiplying the weight in kilograms by 2.2, or 75.0 × 2.2 = 165 lb. Then add the maximum amount of weight of 25 lb to this total, or 165 + 25 = 190 lb.

Two family members are diagnosed with the same genetic disorder but have distinctly different manifestations. What should the nurse consider as being the reason for this occurrence? 1. Imprinting 2. Penetrance 3. New mutation 4. Variable expression

Answer: 1Explanation: 1. The expression of a few genetic conditions varies depending on whether the altered gene is inherited from the mother or the father. This differential gene expression is due to genomic imprinting. Imprinting takes place before gametes are formed, when certain genes are chemically marked as having maternal or paternal origin. After conception, the imprint controls gene expression so that only one allele, either maternal or paternal, is expressed. If the unsilenced (active) allele carries a mutation, disease may result and the diseases may be different. 2. Penetrance is the probability that a gene will be expressed phenotypically. 3. A new mutation is when there is no previous family history of a condition. 4. The term expressivity is used to describe the degree to which a phenotype is expressed. When people with the same genetic makeup (genotype) exhibit signs or symptoms with varying degrees of severity, the phenotype is described as showing variable expression.

5) The nurse instructs a client about conception and fetal development. Which client statement indicates understanding about transportation time of the zygote through the fallopian tube and into the cavity of the uterus? 1. "It will take 8 days for the egg to reach the uterus." 2. "It will take at least 3 days for the egg to reach the uterus." 3. "It will take 18 hours for the fertilized egg to implant in the uterus." 4. "It will only take 12 hours for the egg to go through the fallopian tube."

Answer: 2 Explanation: 1. "It will take 8 days for the egg to reach the uterus" is an incorrect interpretation of the information on conception. 2. It takes at least 3 days for the egg to reach the uterus. 3. "It will take 18 hours for the fertilized egg to implant in the uterus" is an incorrect interpretation of the information on conception. 4. "It will only take 12 hours for the egg to go through the fallopian tube" is an incorrect interpretation of the information on conception.

Which client statement on cultural or religious influences on nutrition requires intervention? 1. "I avoid milk and meat at meals because I am Jewish." 2. "My auntie sent me clay from the south to eat every day." 3. "Because I am Muslim, I do not ever eat any pork products." 4. "My grandmother makes sure I eat a serving of greens each day."

Answer: 2 Explanation: 1. A kosher diet involves avoiding pork and shellfish and not eating dairy and meat at the same meal. 2. Eating clay is pica. The clay, being a type of soil, can be contaminated with hazardous substances and should be avoided. Some African Americans, especially those from the South, practice clay-eating pica. 3. Dietary restrictions in the Muslim tradition include avoidance of pork. Because other meats are eaten, the client is not at risk for protein or iron deficiency. 4. Greens, such as collard greens and spinach, have high amounts of folic acid and are healthy foods to eat during pregnancy. Women from the southern United States often eat greens.

The nurse is teaching a couple pregnant for the first time. Which statement made by the couple about prenatal classes indicates that additional information is necessary? 1. "Facilitate better communication between both partners." 2. "Eliminate the risk of needing a vacuum extraction or cesarean birth." 3. "Help parents cope with the discomforts and unknowns of childbirth." 4. "Encourage participants to write a list of their requests for labor and birth."

Answer: 2 Explanation: 1. A secondary goal of prenatal classes is to facilitate communication between the pregnant woman and her partner. The primary goals are to inform participants of the birth process and teach them skills to cope with labor and birth. 2. This is false reassurance. There is no guarantee that an operative birth can be avoided, even if a couple attends prenatal classes. 3. This is one of the main goals of prenatal classes: learning skills to help get through the discomforts of childbirth. Participants also learn how the birthing process progresses. Participants will learn new skills that will facilitate the birthing process. 4. One of the strategies used in prenatal classes is to have participants write a birth plan that lists their requests for how they want their birthing experience to be. But putting their desires down in writing is less important than learning about the birth process and learning skills to cope with labor and delivery.

The nurse is assessing a 25-year-old primigravida who is 20 weeks pregnant. Which vital signs finding should the nurse report immediately to the physician? 1. Pulse 88/min 2. Respirations 30/min 3. Blood pressure 134/82 4. Temperature 37.4°C (99.3°F)

Answer: 2 Explanation: 1. A slight increase in pulse is an expected finding during pregnancy due to the increased oxygen consumption to support fetal metabolism. 2. Tachypnea is not a normal finding and requires medical care. 3. The blood pressure is within normal limits. 4. A slightly elevated temperature is an expected finding during pregnancy due to the increased oxygen consumption to support fetal metabolism.

A 16-year-old is making her first prenatal visit to the clinic in her fourth month of pregnancy. What is the nurse's first responsibility? 1. Contact the social worker. 2. Develop a trusting relationship. 3. Teach the client about proper nutrition. 4. Schedule the client for prenatal classes.

Answer: 2 Explanation: 1. A social worker might be able to provide assistance with financial program eligibility, support groups, or obtaining baby items such as furniture and car seats. 2. The most important goal for the nurse caring for a pregnant adolescent is to be open minded and nonjudgmental in order to foster trust between the adolescent and the nurse. Through a trusting relationship, the nurse can provide counseling and education to the mother-to-be, both about her body and the fetus. Developing a trusting relationship with the pregnant adolescent is essential. Honesty, respect, and a caring attitude promote self-esteem. 3. Although nutrition is an important physiologic need, without a trusting relationship, little teaching will occur because the teen will often "tune out" adults that she does not trust. 4. Prenatal classes specifically designed for teen mothers and attended by only teen mothers facilitate both learning and support for the teens.

What should the nurse do when assisting a pregnant client who is having an abdominal ultrasound to determine fetal age? 1. Has the woman empty her bladder before the test begins 2. Assists the woman into a supine position on the examining table 3. Asks the woman to sign an operative consent form prior to the procedure 4. Instructs the woman to eat a fat-free meal 2 hours before the scheduled test time

Answer: 2 Explanation: 1. The recommendation is that the client should have a full bladder to help elevate the uterus out of the pelvic cavity for better visualization. 2. Clients are placed in a supine position on the table. 3. Abdominal ultrasounds are not invasive procedures and do not require a consent form. 4. Dietary intake is not relevant to the ultrasound.

A client who is 32 weeks pregnant is HIV positive, but asymptomatic. What would be important in managing her pregnancy and delivery? 1. An amniocentesis at 30 and 36 weeks 2. Weekly non-stress testing beginning at 32 weeks' gestation 3. Administration of intravenous antibiotics during labor and delivery 4. Application of a fetal scalp electrode as soon as her membranes rupture in labor

Answer: 2 Explanation: 1. All invasive procedures that would expose the uninfected infant to the HIV virus are avoided. 2. Clients who are HIV positive are considered high-risk pregnancies. Therefore, beginning at about 32 weeks, these clients have weekly non-stress tests to assess for placental function and an ultrasound every 2 to 3 weeks to assess for intrauterine growth retardation (IUGR). 3. Antibiotics would be ineffective for either the mother or the infant who was HIV positive. 4. All invasive procedures that would expose the uninfected infant to the HIV virus are avoided.

The nurse is presenting a session on intimate partner violence. Which statement indicates a need for further education? 1. "The 'honeymoon period' follows an episode of violence." 2. "Everyone experiences anger and hitting in a relationship." 3. "Abusers can be either husbands or boyfriends or wives or girlfriends." 4. "My daughter is not to blame for the violence in her marriage."

Answer: 2 Explanation: 1. An acute episode of battering is followed by the tranquil phase, or honeymoon period, when the abuser is often repentant and promising never to abuse the victim again. In some cases, the honeymoon period is the only time there is a lack of building tension. 2. Violence is not a normal part of intimate relationships. This statement indicates that the client has likely been a victim of domestic violence. 3. Abusers can be spouses or boyfriends or girlfriends. Intimate partner violence can be experienced in any intimate relationship, regardless of whether the couple is straight, gay, or lesbian, and both within marriage and outside of marriage. 4. The victims of violence are not the cause of the violence. Abusers are responsible for their violent behavior. Avoiding blaming and shaming of victims of domestic violence is important to establish a therapeutic relationship.

The nurse is providing nutritional counseling for a postpartum client with a hemoglobin of 8. Which statement indicates that additional teaching is necessary? 1. "I need to increase food sources that contain iron." 2. "If I drink lots of milk, I will increase my iron level faster." 3. "My iron is low, but it will increase as I take iron supplements." 4. "I might feel less energetic and tire more easily while my iron is low."

Answer: 2 Explanation: 1. Anemia requires additional iron. Many foods, such as red meat, will provide iron. Increasing iron-rich foods will improve anemia. 2. Milk does not contain iron; it contains calcium. Increased calcium intake will not increase hemoglobin levels. Further, iron should not be taken with milk, as the iron will not be absorbed. 3. Iron supplements are indicated with anemia. This client's hemoglobin level is 8; lower than 10 is considered anemia during pregnancy. Taking iron will increase hemoglobin. 4. Hemoglobin carries oxygen; when the hemoglobin level is low, the muscles are not adequately oxygenated, especially during activity, and fatigue results.

The nurse has completed a presentation for newly pregnant women about the changes of pregnancy. Which participant's statement reflects accurate comprehension of the information? 1. "Uterine souffle is a positive change of pregnancy." 2. "A positive Goodell sign is a probable change of pregnancy." 3. "Changes in the pelvic organs are presumptive signs of pregnancy." 4. "Three positive pregnancy tests in a 1-week period is considered to be a positive change of pregnancy."

Answer: 2 Explanation: 1. Because uterine souffle can be objectively identified but may be caused by conditions other than pregnancy, it is considered to be a probable change of pregnancy. 2. A positive Goodell sign can be objectively identified but may also be caused by conditions other than pregnancy; therefore, it is considered to be a probable change of pregnancy. 3. Changes in the pelvic organs can be objectively identified; however, because some pelvic organ changes may be associated with conditions other than pregnancy, they are considered to be probable changes of pregnancy. 4. Because other conditions may cause elevated hCG, pregnancy tests are considered probable changes of pregnancy.

13) A nurse is providing guidance to a group of parents of children in the infant-to-preschool age group. After reviewing statistics on the most common cause of death in this age group, what information should the nurse include about prevention? 1. Reduce the use of pesticides in the home to prevent cancer. 2. Review swimming pool and traffic accidents to prevent accidental injury. 3. Incorporate heart-healthy foods into the child's diet to prevent heart disease. 4. Provide a diet high in vitamin C from fruits and vegetables to prevent pneumonia.

Answer: 2 Explanation: 1. Cancer is not a common cause of death for this age group. 2. Unintentional injuries are the most common cause of death for children between 1 and 19 years old. In children 1 to 4 years old, this is followed by drowning; fire and burns; suffocation; and pedestrian-related injuries. 3. Heart disease is not a common cause of death for this age group. 4. Pneumonia is not a common cause of death for this age group.

An expectant couple is determining their compatibility with a healthcare provider. Which question should the nurse encourage the couple to ask first? 1. "Can my children attend the birth?" 2. "What is your philosophy of birth?" 3. "If I have a cesarean birth, can my husband attend?" 4. "What percentage of your clients have episiotomies?"

Answer: 2 Explanation: 1. Children's attendance is a complement to the healthcare provider's philosophy. 2. A thorough understanding of the healthcare provider's philosophy is essential to determining compatibility. 3. A husband's presence at a cesarean birth is a complement to the healthcare provider's philosophy. 4. Episiotomy percentages are a complement to the healthcare provider's philosophy.

When a woman who has been raped is admitted to the emergency department, which nursing intervention has priority? 1. Contact family members. 2. Create a safe, secure atmosphere for the woman. 3. Assure the woman that everything will be all right. 4. Explain exactly what will need to be done to preserve legal evidence.

Answer: 2 Explanation: 1. Contacting family members is not the top priority and can wait until a safe environment is established. 2. The first priority in caring for a survivor of a sexual assault is to create a safe, secure atmosphere that will allow the woman to process what has happened. 3. Assuring the woman that everything will be all right is not the top priority and is giving false promise. 4. Explaining exactly what will need to be done to preserve legal evidence is not the top priority.

What should the gynecology clinic nurse recommend for the client experiencing premenstrual syndrome (PMS)? 1. "Increase your consumption of red meat when you feel symptoms, and eat three large meals per day." 2. "Engage in aerobic activity often throughout the month, and continue exercising when your symptoms begin." 3. "Decrease your dietary intake of dairy and soy slightly during the month, and especially during your days of bleeding." 4. "Eat more chocolate and drink more caffeine beginning a week prior to when your menstrual cycle bleeding should begin."

Answer: 2 Explanation: 1. Decreased red meat consumption can be beneficial to reduce PMS symptoms, as will eating several small meals per day rather than three large meals. 2. Regular aerobic activity helps to decrease PMS symptoms. 3. 1200 mg of calcium per day can help decrease PMS symptoms. The calcium can either come from supplements or be obtained through dietary intake of dairy and soy products. 4. Chocolate and caffeine contain methylxanthines; therefore, intake of chocolate, coffee, and colas should be limited throughout the month.

The nurse is presenting a community education session on female hormones. Which participant statement indicates the need for further information? 1. "Estrogen is what causes females to look female." 2. "Prostaglandin is responsible for achieving conception." 3. "Progesterone is present at the end of the menstrual cycle." 4. "The presence of some hormones causes others to be secreted."

Answer: 2 Explanation: 1. Estrogen causes secondary sex characteristics, such as enlarged breasts and widened hips. 2. Prostaglandin is not related to conception. Prostaglandin is called the hormone of pregnancy because it maintains pregnancy. 3. Progesterone is present in large quantities during the secretory phase of the menstrual cycle. 4. An example is that the production of gonadotropin-releasing hormone (GnRH) causes the secretion of luteinizing hormone (LH) and follicle-stimulating hormone (FSH).

A woman has been unable to complete a full-term pregnancy because the fertilized ovum failed to implant in the uterus. Which hormone is most likely causing this client's issues with pregnancy? 1. Estrogen 2. Progesterone 3. LH (luteinizing hormone) 4. FSH (follicle-stimulating hormone)

Answer: 2 Explanation: 1. Estrogen primarily assists in maturation of the ovarian follicles and causes endometrial mucosa to proliferate. 2. Progesterone is the likely cause because it decreases uterine motility and contractibility caused by estrogens, thereby preparing the uterus for implantation. 3. LH is a hormone secreted by the pituitary gland. 4. FSH is a hormone secreted by the pituitary gland.

Which situation in the high-risk antepartal unit requires immediate intervention? 1. Fetal monitoring is being performed on a client in her third trimester who is scheduled for a cholecystectomy tomorrow. 2. A third-trimester client pregnant with twins who required an appendectomy yesterday is positioned in a supine position. 3. Oxygen is being administered at 2 L via nasal cannula to a client in her third trimester who underwent a urolithotomy today. 4. The client in her third trimester who returned from bowel resection surgery has a nasogastric tube attached to intermittent suction.

Answer: 2 Explanation: 1. Fetal monitoring prior to, during, and after surgery on pregnant clients is important to assess the fetal condition. 2. A client undergoing surgery in the third trimester should be positioned in a left lateral position or with a hip wedge placed under the right hip. Being supine will cause vena cava syndrome and hypotension, which in turn will decrease fetal oxygenation. Twin gestation, with the larger uterus and heavier uterine contents, makes vena cava syndrome more problematic. 3. Oxygen is required during and after surgery during pregnancy to maintain adequate fetal oxygenation. 4. Due to the decreased peristalsis of pregnancy, pregnant clients who undergo abdominal surgery are at risk for vomiting. A nasogastric tube is placed to prevent vomiting.

The nurse is preparing a presentation on the menstrual cycle for a group of high school students. Which statement should the nurse include in this presentation? 1. "One hormone controls the phases of the menstrual cycle." 2. "The secretory phase occurs when a woman is most fertile." 3. "Menstrual cycle phases vary in order from one woman to another." 4. "The menstrual cycle has five distinct phases that occur during the month."

Answer: 2 Explanation: 1. Four hormones control ovulation and therefore the menstrual cycle: progesterone, estrogen, follicle-stimulating hormone (FSH), and luteinizing hormone (LH). 2. During the secretory phase, the endometrium is thickest, and glycogen is produced to nourish a fertilized ovum. 3. Although the length of the menstrual cycle might vary, the phases of the menstrual cycle always occur in the same order. 4. There are four phases of the menstrual cycle: menstrual, proliferative, secretory, and ischemic phases.

The nurse is teaching an early pregnancy class for clients in the first trimester of pregnancy. Which statement requires immediate intervention by the nurse? 1. "When my nausea is bad, I will drink some ginger tea." 2. "It is normal for my vaginal discharge to get green colored." 3. "I will urinate less often during the middle of my pregnancy." 4. "The fatigue I am experiencing will improve in the second trimester."

Answer: 2 Explanation: 1. Ginger helps nausea, and is safe for use during pregnancy. 2. Leukorrhea is an increase in white vaginal discharge and is an expected finding during pregnancy. Green discharge is not a normal finding and could indicate an infection. Further assessment is required for a client with green vaginal discharge. 3. As the uterus rises in the pelvis during the second trimester, urinary frequency decreases. Urinary frequency increases again during the end of the third trimester as the fetal head descends into the pelvis. 4. First-trimester fatigue is common; fatigue usually improves during the second trimester.

The nurse is conducting a postpartum visit to a client who is formula-feeding her infant. Which client statement indicates that teaching about weight maintenance has been effective? 1. "I have increased my caloric intake by 600 calories per day." 2. "My dietician has set my weight loss goal at 1 to 2 pounds per week." 3. "Instead of making another doctor's appointment, I started a diet that my best friend recommended." 4. "My daily regimen includes taking extra vitamin A, vitamin C, and thiamine in order to meet my body's increased need for nutrients after pregnancy."

Answer: 2 Explanation: 1. If the mother has a good understanding of nutritional principles, it is sufficient to advise her to reduce her daily caloric intake by about 300 kcal and to return to prepregnancy levels for other nutrients. 2. Weight loss goals of 1 to 2 pounds (0.45 to 0.9 kg)/week are usually suggested for mothers who formula-feed. 3. The woman should diet only under the guidance of her primary healthcare provider. 4. After birth, the formula-feeding mother's dietary requirements return to prepregnancy levels.

The nurse is preparing teaching on maternal-fetal ABO incompatibility for antepartum clients. Which statement should the nurse include in the teaching information? 1. In most cases, ABO incompatibility is limited to type A mothers with a type B or O fetus. 2. In most cases, ABO incompatibility is limited to type O mothers with a type A or B fetus. 3. ABO incompatibility occurs as a result of the fetal serum antibodies present and interaction between the antigen sites on the maternal red blood cells (RBCs). 4. Group A infants, because they have no antigenic sites on the red blood cells (RBCs), are never affected regardless of the mother's blood type.

Answer: 2 Explanation: 1. In most cases, ABO incompatibility is limited to type O mothers with a type A or B fetus. The group B fetus of a group A mother and the group A fetus of a group B mother are only occasionally affected. 2. In most cases, ABO incompatibility is limited to type O mothers with a type A or B fetus. The group B fetus of a group A mother and the group A fetus of a group B mother are only occasionally affected. 3. The incompatibility occurs as a result of the maternal antibodies present in her serum and interaction between the antigen sites on the fetal red blood cells (RBCs). 4. Group O infants, because they have no antigenic sites on the red blood cells (RBCs), are never affected regardless of the mother's blood type.

8) The nurse wants to teach a child newly enrolled in English as a second language class about the importance of handwashing before meals and of not eating food dropped on the examination room floor. What is the best way to assimilate the nurse's cultural values about hygienic nutrition? 1. Schedule a medical interpreter to accompany the client to his or her next visit. 2. Have the child repeat her or his interpretation of the information that was taught. 3. Provide written materials in English about hygiene and diet for the client to take home. 4. Have the nurse model proper handwashing before examining the child and throw out the dropped cookie.

Answer: 2 Explanation: 1. In working with families with limited English proficiency, it is optimal to have a medical interpreter present for the entire visit. When teaching has been done, the nurse has a responsibility to assess client understanding; thus, an interpreter at the next visit will not help the nurse or the client now. 2. When an interpreter is not available, asking the client to repeat his or her understanding of what was taught reveals how concepts were understood. 3. Written materials in English hold minimal value for clients with limited understanding. 4. Assimilation is described as adopting and incorporating traits of the new culture within one's practices. Information must be understood before it is assimilated. The purpose of modeled behavior might be misunderstood if it is not accompanied by an explanation.

The nurse is responding to phone calls. Which call should the nurse return first? 1. 29 weeks' gestation, reports increased fetal movement 2. 37 weeks' gestation, reports no fetal movement for 24 hours 3. 32 weeks' gestation, reports decreased fetal movement for 2 days 4. 35 weeks' gestation, reports decreased fetal movement for 4 hours

Answer: 2 Explanation: 1. Increased fetal movement is not indicative of a problem. 2. A lack of fetal movement in a fetus in the third trimester can indicate fetal hypoxia or fetal death. This client is the highest priority. 3. Although decreased fetal movement can indicate intrauterine growth restriction or fetal hypoxia, this client is not the highest priority. 4. Although decreased fetal movement can indicate intrauterine growth restriction or fetal hypoxia, 4 hours is a very short amount of time to assess decreased fetal movement.

A client who is experiencing her first pregnancy has just completed the initial prenatal examination with a certified nurse-midwife. Which statement indicates that the client has a correct understanding of her condition? 1. "The increased size of my uterus means that I am finally pregnant." 2. "Because we heard the baby's heartbeat, I am undoubtedly pregnant." 3. "Since I haven't felt the baby move yet, we don't know if I'm pregnant." 4. "My last period was 2 months ago, which means I'm 2 months along."

Answer: 2 Explanation: 1. Increased uterine size is a probable, or objective, change and does not conclusively verify pregnancy status. 2. Hearing the fetal heart rate is a positive, or diagnostic, change of pregnancy. 3. Fetal movement is a presumptive, or subjective, change of pregnancy. Absence or presence of the sensation of fetal movement is not a conclusive indicator of pregnancy status. 4. Amenorrhea is a presumptive, or subjective, change and does not conclusively verify pregnancy status.

The nurse seeks to involve the adolescent father in the prenatal care of his partner. What is the reason for this strategy? 1. Improves the long-term outcome of the relationship 2. Increases the self-care behaviors of the pregnant teen 3. Avoids legal action by the adolescent father's family 4. Avoids conflict between the adolescent father and pregnant teen

Answer: 2 Explanation: 1. Involving the client's partner in prenatal care will not decrease the likelihood that this relationship will be short term. 2. Involving the partner of a pregnant adolescent helps the mother-to-be feel more confident in her decision making and improves her self-confidence and self-esteem, which in turn will improve positive self-care behaviors. 3. The nurse first must explore what the relationship is between the pregnant teen and the father. Relationships between adolescents tend to be short lived, and pregnancy is an added stressor for the couple. 4. If the client desires the participation of her partner, the nurse should provide education and support appropriate to the age, knowledge, and developmental level of the adolescent father.

A client in the third trimester of pregnancy reports frequent leg cramps. What strategy would be most appropriate for the nurse to suggest? 1. Limit activity for several days. 2. Flex the foot to stretch the calf. 3. Point the toes of the affected leg. 4. Increase intake of protein-rich foods.

Answer: 2 Explanation: 1. Limiting activity is not appropriate. 2. Leg cramps are a common problem in pregnancy, resulting from an imbalance in the calcium-phosphorus ratio; pressure on nerves or decreased circulation in the legs from the enlarged uterus; or fatigue. Dorsiflexing the foot will stretch the calf muscles and will help relieve the cramps. 3. Pointing the toes will exacerbate leg cramps. 4. Protein intake does not affect leg cramps.

During a prenatal examination the healthcare provider is going to estimate the adequacy of the client's pelvis for birth. Which measurement will the physician perform vaginally? 1. True conjugate 2. Diagonal conjugate 3. Obstetric conjugate 4. Transverse outlet diameter

Answer: 2 Explanation: 1. The true conjugate is a measurement of the pelvic inlet and cannot be directly measured. 2. The diagonal conjugate is measured from the lower edge of the symphysis to the sacral promontory. 3. The obstetric conjugate is a measurement of the pelvic inlet and cannot be directly measured. 4. The transverse outlet diameter is measured externally.

The nurse is caring for a client who is pregnant with twins. Which statement indicates the client needs additional information? 1. "It is rare for twins to both be within the same amniotic sac." 2. "Because both of my twins are boys, I know that they are identical." 3. "If I have one boy and one girl, I will know they came from two eggs." 4. "If my twins came from one fertilized egg that split, they are identical."

Answer: 2 Explanation: 1. Monoamnionic-monochorionic twins are very rare and occur as a result of the zygote splitting 7 or more days after fertilization. 2. Not all same-sex twins are identical or monozygotic, because fraternal, or dizygotic, twins can be the same gender or different genders. 3. The only way to have twins of different genders is if they came from two separate fertilized ova. Monozygotic twins are identical and are the same gender. 4. When the zygote splits, identical twins share the same genotype result.

A client calls his urologist's office to clarify instructions about semen analysis. What should the nurse instruct the client to do? 1. Use a lubricant while obtaining the semen specimen. 2. Remain abstinent for 3 days prior to collecting the specimen. 3. Immediately refrigerate the specimen for a maximum of 8 hours. 4. Deliver the specimen to the laboratory within 1 hour of collection.

Answer: 2 Explanation: 1. Most lubricants also are spermicidal and should not be used unless approved by the andrology laboratory. 2. To obtain accurate results of a semen analysis, the specimen is collected after 3 days of abstinence. 3. If the specimen is obtained at home, it needs to be kept at body temperature and delivered to the laboratory within 1 hour so as not to impair motility. 4. If the specimen is obtained at home, it needs to be delivered to the laboratory within 1 hour so as not to impair motility.

15) A client says that she is taking a preparation that makes symptoms of a disease worse. Which type of complementary and alternative therapy is this client using? 1. Naturopathy 2. Homeopathy 3. Herbal therapy 4. Chinese medicine

Answer: 2 Explanation: 1. Naturopathy is a form of medicine that utilizes the healing forces of nature and is commonly referred to as natural medicine. It is more precisely defined as a healing system that combines safe and effective traditional means of preventing and treating human disease with the most current advances in modern medicine. 2. Homeopathy is a healing approach in which a sick person is treated with small doses of medicines that would cause illness when given to someone who is healthy. 3. Herbs do not usually cause symptoms when taken. 4. Chinese medicine uses a variety of techniques including acupuncture, acupressure, and herbal therapy.

A 20-year-old client who is at 10 weeks' gestation confides that the pregnancy was unplanned and is unsure about continuing it or sharing news about it with her partner. How should the nurse respond to this client? 1. "You should go to a pregnancy support group to be a good mother." 2. "It's common to feel ambiguous about pregnancy in the first trimester." 3. "These thoughts are because your mother died when you were 4 years old." 4. "It's really unusual for a pregnant woman to feel this way early in the pregnancy."

Answer: 2 Explanation: 1. No psychopathology is present, so a support group is not indicated. 2. Ambivalence toward the pregnancy is very common in the first trimester. 3. Loss of the client's own mother at a young age would not affect the occurrence of ambivalence in the first trimester. 4. Fathers might not be told immediately about the diagnosis of pregnancy.

A pregnant client asks what kinds of medications cause birth defects. Which statement would best answer this question? 1. "Almost all medications will cause birth defects in the first trimester." 2. "To be safe, do not take any medication without talking to your doctor." 3. "Too much vitamin C is one of the most common issues but is avoidable." 4. "Birth defects are very rare. Do not worry; your healthcare provider will watch for problems."

Answer: 2 Explanation: 1. Not all medications are teratogenic. 2. Pregnant women need to avoid all medications—prescribed, homeopathic, or over-the-counter—if possible. 3. Vitamin C can cause rebound scurvy but is not teratogenic. 4. The nurse should avoid a "do not worry" answer to ensure therapeutic communication, but it is appropriate to instruct the client to talk to the doctor about medications.

A client with a normal prepregnancy weight asks why she has been told to gain 25 to 35 pounds during her pregnancy, but her underweight friend was told to gain 28 to 40 pounds. What should the nurse explain as being the recommended weight gain during pregnancy? 1. More than 25 to 35 pounds for an overweight client 2. More than 25 to 35 pounds for an underweight woman 3. 25 to 35 pounds, regardless of a client's prepregnancy weight 4. The same for a normal-weight woman as for an overweight woman

Answer: 2 Explanation: 1. Overweight women should gain 15 to 25 pounds during pregnancy. 2. Underweight women are encouraged to gain 28 to 40 pounds during pregnancy 3. Prepregnancy weight determines the recommended weight gain during pregnancy. Women of normal weight should gain 25 to 35 pounds during pregnancy for optimal fetal outcome. 4. Overweight women should gain 15 to 25 pounds during pregnancy.

The nurse has been asked by a community organization to give a presentation on prevention of teen pregnancy. Which statement indicates appropriate steps toward reduction of the local teen pregnancy rate? 1. Classes on how to parent will be mandatory in high school. 2. Plans are made to create a low-cost reproductive health clinic. 3. Parents will be encouraged to avoid discussing sexual activity. 4. Abstinence-only education will be offered in the school and clinics.

Answer: 2 Explanation: 1. Parenting classes for teens who are neither pregnant nor parents do not address reducing teen pregnancy. 2. Confidential, low-cost contraceptive education and services are most likely to increase contraceptive use by teens who are sexually active and therefore decrease teen pregnancy rates. 3. Parents are the biggest influence on teens' decisions to begin or avoid sexual activity. Parents should be encouraged to talk openly and frankly with their teens about their views on sex, contraception, and abstinence. 4. Abstinence first with information on contraception is most effective in reducing teen pregnancy rates.

11) The nurse is reviewing the 1973 Supreme Court decision in Roe v. Wade for a patient asking about an abortion. What should the nurse explain about the induction of a legal abortion? 1. It must be performed at a federally funded clinic. 2. It must be performed before the period of viability. 3. It must be performed at a military hospital overseas. 4. It must be performed to provide tissue for therapeutic research.

Answer: 2 Explanation: 1. Whether conducted at a federally funded clinic, abortion can be provided legally if under U.S. laws. 2. Abortion can be performed legally until the period of viability; after viability, the rights of the fetus take precedence. 3. Whether or not conducted at a military hospital overseas, abortion can be provided legally if under U.S. laws. 4. Abortion cannot be used to provide tissue for therapeutic research.

The nurse is planning a community adolescent pregnancy prevention program aimed toward parents. Which recommendation should be included in the program in order to be effective? 1. Parents should encourage steady dating. 2. Parents should not allow their son to develop an intense relationship with a girl who is much younger. 3. Rather than embarrassing an adolescent by addressing specific topics related to sex, parents should speak in broad, general terms. 4. Instead of bringing up the topic of sex, parents should allow their children to reach a point where the children initiate the discussion.

Answer: 2 Explanation: 1. Parents need to clearly discourage early dating as well as frequent and steady dating. 2. Parents should take a strong stand against allowing a daughter to date a much older boy; nor should they allow a son to develop an intense relationship with a much younger girl. 3. Parents should be specific in their discussions about sex. 4. Parents need to talk with their children about sex early and often and be specific in the discussions.

The nurse is creating a care plan for a client who is unable to conceive as a consequence of endometriosis. Which statement accurately reflects a nursing diagnosis that may apply to the care of this client? 1. Acute pain related to dysuria and renal pain secondary to endometriosis 2. Compromised family coping related to depression secondary to infertility 3. Infertility related to endometrial inflammation and adhesions secondary to endometriosis 4. Hyperandrogenism related to elevated serum androgen levels secondary to endometriosis

Answer: 2 Explanation: 1. Pelvic pain is a frequent symptom of endometriosis, while dysuria and renal pain are more commonly associated with conditions such as upper urinary tract infections (UTIs). 2. Infertility may lead to depression and subsequent compromised family coping, which is a nursing diagnosis. 3. Although associated with the medical condition of endometriosis, infertility is a medical diagnosis. 4. Hyperandrogenism is a medical diagnosis that pertains to elevated serum androgen levels. Hyperandrogenism is associated with polycystic ovarian syndrome (PCOS).

A newly diagnosed type 1, insulin-dependent diabetic with good blood sugar control at 20 weeks' gestation asks how the diabetes will affect the baby. How should the nurse respond? 1. "Your baby may be smaller than average at birth." 2. "Your baby will probably be larger than average at birth." 3 "Your baby might have high blood sugar for several days." 4. "As long as you control your blood sugar, your baby will not be affected at all."

Answer: 2 Explanation: 1. Poorly controlled type 1 diabetics who have developed vascular problems will have infants who are small-for-gestational-age (SGA) due to placental insufficiency. 2. The infant of a diabetic mother produces excessive amounts of insulin in response to the high blood sugar. This hyperinsulinism stimulates growth (or macrosomia) in the infant because the infant utilizes the glucose in the bloodstream. 3. Within minutes of delivery, the baby of an insulin-dependent diabetic can begin to develop low blood sugar. 4. The demands of pregnancy will make it difficult for the best of clients to control blood sugar on a regular basis.

Which diagnostic test should the nurse question if prescribed for a client with pelvic inflammatory disease (PID)? 1. RPR (rapid plasma reagin) 2. Throat culture for streptococcus A 3. Vaginal culture for Neisseria gonorrhoeae 4. CBC (complete blood count) with differential

Answer: 2 Explanation: 1. RPR is a test for syphilis, another cause of PID. 2. Streptococcal infection of the throat is not associated with PID. 3. Gonorrhea is a common cause of PID, and the client should be tested for this. 4. CBC with differential will give an indication of the severity of the infection.

The nurse is preparing a class for expectant fathers. Which information should the nurse include? 1. Siblings adjust readily to the new baby. 2. Sexual activity is safe for normal pregnancy. 3. The expectant mother decides the feeding method. 4. Fathers are expected to be involved in labor and birth.

Answer: 2 Explanation: 1. Siblings often have difficulty adapting to the arrival of a new baby. Regression is often seen in siblings' behaviors. 2. During a normal pregnancy, sexual activity is safe for both mother and baby. 3. Often, the father wants input into the feeding method. 4. In some cultures, labor and birth are only for women, and it is inappropriate for fathers to be involved with the labor and birth.

3) Through genetic testing a client is found to have a germline mutation of a specific gene. What information about this finding should be reviewed with the client? 1. No children will have the mutation. 2. All future children will have the mutation. 3. Only male children will have the mutation. 4. Only female children will have the mutation.

Answer: 2 Explanation: 1. Somatic mutations are not passed from one generation to another. 2. Hereditary mutations are passed to offspring from one or both parents and are also known as germline mutations because the mutation exists in the reproductive cells or gametes. The DNA in every cell of that offspring will have the mutation, which can then be transmitted to following generations. 3. This mutation exists in the reproductive cells and will affect both genders. 4. This mutation exists in the reproductive cells and will affect both genders.

4) A client of Hmong descent who immigrated to the United States 5 years ago asks or the regular hospital menu because she likes American food. To which cultural concept should the nurse attribute this client's request? 1. Stereotyping 2. Acculturation 3. Enculturation 4. Ethnocentrism

Answer: 2 Explanation: 1. Stereotyping is assuming that all members of a group have the same characteristics. 2. Acculturation (assimilation) is the correct assessment because the client adapted to a new cultural norm in terms of food choices. 3. Enculturation is when culture is learned and passed on from generation to generation, and often happens when a group is isolated. 4. Ethnocentrism refers to a social identity that is associated with shared behaviors and patterns.

Which client should be treated with ceftriaxone (Rocephin) IM and doxycycline (Vibramycin) by mouth? 1. A pregnant client with syphilis 2. A nonpregnant client with gonorrhea and chlamydial infection 3. A pregnant client with gonorrhea and a yeast infection 4. A nonpregnant client with chlamydial infection and trichomoniasis

Answer: 2 Explanation: 1. Syphilis is treated with penicillin. 2. This combined treatment provides dual treatment for gonorrhea and chlamydial infection because the two infections frequently occur together. 3. Doxycycline is contraindicated during pregnancy. 4. Trichomoniasis is treated with metronidazole.

The nurse is conducting an early pregnancy class for a group of pregnant women. Which statement indicates a need for further education about the placenta? 1. "It develops and grows larger until about 20 weeks' gestation." 2. "It floats in the amniotic sac and filters waste products from the fetus." 3. "It creates hormones and enzymes that are necessary during pregnancy." 4. "It ages and becomes less permeable during the last month of pregnancy."

Answer: 2 Explanation: 1. The chorionic villi of the placenta become more differentiated with time. The placenta grows in size until about 20 weeks. After this point, the placenta thickens but does not increase in size. 2. The placenta is attached to the uterine wall, and does not float in the amniotic sac. One function of the placenta is to filter metabolic waste products from the baby's blood so that they can be excreted by the mother. 3. The placenta creates glycogen, cholesterol, hormones such as human chorionic gonadotropin (hCG), and enzymes such as sulfatase and insulinase. 4. The placenta is designed to last for 40 weeks, the average length of human gestation. The permeability to nutrients and oxygen begins to decrease starting at about 36 weeks as a part of the aging of the placenta.

A nurse is teaching a classroom of teenage girls about the female reproductive system. After teaching, the nurse asks the students to describe the release of an ovum during ovulation. Which response indicates that teaching has been effective? 1. "Ovulation is when the uterus releases an unfertilized egg or ovum." 2. "During ovulation, an egg is released from the ovary and enters the fallopian tube." 3. "The endometrium is where the eggs are formed and released into the fallopian tube." 4. "Around the middle of the menstrual cycle, one of the fallopian tubes releases an egg."

Answer: 2 Explanation: 1. The egg is formed in the ovary and travels by way of the fallopian tube to the uterus. 2. The egg is formed in the ovary and once released, it enters the fallopian tube. 3. The uterine endometrium is the site of implantation of a fertilized egg. 4. The egg is formed in the ovary and then released near the fimbria of the fallopian tube.

A pregnant woman is having a nipple-stimulated contraction stress test. Which result indicates hyperstimulation? 1. There are more than five fetal movements in a 10-minute period. 2. There are more than three uterine contractions in a 6-minute period. 3. The fetal heart rate accelerates when contractions last up to 60 seconds. 4. The fetal heart rate decelerates when three contractions occur within a 10-minute period.

Answer: 2 Explanation: 1. The fetal movement is considered a negative contraction stress test. 2. An equivocal or suspicious test has nonpersistent late decelerations or decelerations associated with hyperstimulation (contraction frequency of every 2 minutes or duration lasting longer than 90 seconds). When this test result occurs, more information is needed. 3. The acceleration of the heart rate is considered a negative contraction stress test. 4. Decelerations are considered a positive contraction stress test.

A client who has experienced a fetal death in utero at 20 weeks asks what her baby will look like when it is delivered. How should the nurse respond to this client? 1. "The genitals of the baby will be ambiguous." 2. "Your baby will be covered in fine hair called lanugo." 3. "Your child will have arm and leg buds, but not fully formed limbs." 4. "A white, cheesy substance called vernix caseosa will be on the skin."

Answer: 2 Explanation: 1. The genitals are apparent by 12 weeks after fertilization. This fetus would have had specifically male or female genitals 8 weeks ago. 2. Downy fine hair called lanugo covers a 20-week fetus. 3. Limb buds have developed by 35 days postfertilization. At 20 weeks, the fetus will have well-developed well-differentiated arms and legs. 4. Vernix caseosa forms at about 24 weeks. This fetus is only 20 weeks and will not have vernix.

The school nurse is teaching a health class to middle school children. Which structure should the nurse explain as secreting follicle-stimulating hormone (FSH) and luteinizing hormone (LH)? 1. Hypothalamus 2. Anterior pituitary 3. Posterior pituitary 4. Ovaries and testes

Answer: 2 Explanation: 1. The hypothalamus secretes gonadotropin-releasing hormone to the pituitary gland in response to signals from the central nervous system. 2. The anterior pituitary secretes FSH and LH, which are primarily responsible for maturation of the ovarian follicle. 3. The posterior pituitary gland secretes oxytocin and antidiuretic hormone. 4. The ovaries secrete the female hormones estrogen and progesterone, and the testes secrete testosterone.

A pregnant adolescent asks about fundal height. What should the nurse say that incorporates the location of the uterine fundus? 1. It is the lower third area of the uterus. 2. It is at the uppermost (dome-shaped top) portion of the uterus. 3. It is the elongated portion of the uterus where the fallopian tubes enter. 4. It is the portion of the uterus that lies between the internal cervical os and the endometrial cavity

Answer: 2 Explanation: 1. The lower third of the uterus is called the cervix or neck. 2. The rounded, uppermost (dome-shaped top) portion of the uterus that extends above the points of attachment of the fallopian tubes is called the fundus. 3. The elongated portion where the fallopian tubes enter the uterus is called the cornua. 4. The portion of the uterus between the internal cervical os and the endometrial cavity is called the isthmus.

A client in the first trimester of pregnancy is experiencing nausea. What should the nurse suggest the client do to relieve the nausea? 1. Eat spicy foods. 2. Eat small, frequent meals. 3. Avoid carbonated beverages. 4. Wait to eat for 2 hours in the morning.

Answer: 2 Explanation: 1. The nausea of pregnancy can be exacerbated by ketosis, fatigue, and certain foods, such as those containing caffeine or spices. 2. Avoiding severe hunger by eating small, frequent meals throughout the day can help to prevent or decrease the severity of the nausea. 3. Carbonated beverages might be helpful in decreasing nausea. 4. Eating dry carbohydrates prior to rising each day can help to prevent or decrease the severity of the nausea.

9) A 12-year-old pediatric client is in need of surgery. Which healthcare member is legally responsible for obtaining informed consent for an invasive procedure? 1. The nurse 2. The physician 3. The social worker 4. The unit secretary

Answer: 2 Explanation: 1. The nurse is not legally responsible for obtaining informed consent for an invasive procedure. 2. Informed consent is legal preauthorization for an invasive procedure. It is the physician's legal responsibility to obtain this because it consists of an explanation about the medical condition, a detailed description of treatment plans, the expected benefits and risks related to the proposed treatment plan, alternative treatment options, the client's questions, and the client's or guardian's right to refuse treatment. 3. It is beyond the social worker's scope of practice to obtain informed consent for an invasive procedure. 4. It is beyond the unit secretary's scope of practice to obtain informed consent for an invasive procedure.

A newly admitted client at 32 weeks' gestation is experiencing a sudden onset of intense nausea and a frontal headache for the past 2 days. The client's initial blood pressure is 158/98, and she reports scant urination over the past 24 hours. Which intervention should the nurse anticipate implementing? 1. Ordering a low-protein diet for the client 2. Conducting a urine dipstick test to assess for proteinuria 3. Placing a wedge under the client's left hip so that she is in a right lateral tilt position 4. Administering diuretics and facilitating a dietary regimen of strict sodium restriction

Answer: 2 Explanation: 1. This client's signs and symptoms are consistent with preeclampsia. Dietary interventions include moderate to high protein intake (80 to 100 g/day, or 1.5 g/kg/day) to replace protein lost in the urine. 2. This client's signs and symptoms are consistent with preeclampsia. Treatment includes daily urine dipstick testing to assess for proteinuria. 3. This client's signs and symptoms are consistent with preeclampsia. Appropriate interventions include instituting bed rest with the client positioned primarily on her left side, to decrease pressure on the vena cava, thereby increasing venous return, circulatory volume, and placental and renal perfusion. 4. This client's signs and symptoms are consistent with preeclampsia. Treatment includes avoidance of excessively salty foods, but sodium restriction and diuretics are no longer used in treating preeclampsia.

The nurse is working with a prenatal client. Which statement indicates that additional teaching about prenatal screening tests is necessary? 1. "My blood will be checked for hemoglobin level." 2. "Because I am married, I won't have the STI screening." 3. "My vagina will be cultured at 36 weeks for group B strep." 4. "I will have Rh testing, even though this is my first pregnancy."

Answer: 2 Explanation: 1. This is a true statement. All women will have their hemoglobin assessed. 2. All women should be screened for syphilis, gonorrhea, and hepatitis B. 3. This is a true statement. Women are tested for group B strep to prevent neonatal infection. 4. This is a true statement. All clients are screened for blood type, Rh factor, and Rh antibodies, regardless of how many previous pregnancies (if any) they have had.

The nurse is completing an assessment for a prenatal visit. Which client statement indicates that further teaching is necessary? 1. "Now that I've felt fetal movement, I should feel movement regularly." 2. "Because I'm in my third trimester, I should return to the clinic in a month." 3. "Alcohol is possibly harmful to my baby, even at the end of my pregnancy." 4. "Before I take any over-the-counter medications, I should contact my doctor."

Answer: 2 Explanation: 1. This is a true statement. Once fetal movement is perceived, it should be felt regularly. Initially, this might not be every day, but in the third trimester, fetal movement should be noticeable several times per day. 2. This statement is incorrect because prenatal visits during the third trimester are every 2 weeks from 26 to 36 weeks, and every week from 36 weeks to delivery. 3. This is a true statement. Alcohol should be avoided throughout pregnancy and lactation. 4. This is a true statement. Regardless of the gestational age, over-the-counter medications can have deleterious effects on the mother or baby; thus, it is important for a pregnant woman to consult her healthcare provider prior to taking any over-the-counter medications throughout the pregnancy.

11) The nurse is working with a child whose religious beliefs differ from those of the general population. What should the nurse do to meet the specific spiritual needs of this child and family? 1. Ask, "What do you think caused the child's illness?" 2. Show respect while allowing time and privacy for religious rituals. 3. Identify healthcare practices forbidden by religious or spiritual beliefs. 4. Ask, "How do the child's and family's religious/spiritual beliefs impact their practices for health and illness?"

Answer: 2 Explanation: 1. This may be part of the spiritual assessment process. 2. Whenever possible the nurse should attempt to accommodate religious rituals and practices requested by the family. 3. This may be part of the spiritual assessment process. 4. This may be part of the spiritual assessment process.

A pregnant client in the 21st week of pregnancy is planning a vacation with family and asks which method of travel she should use. How should the nurse respond? 1. "Travel by bus." 2. "Fly on an airplane." 3. "Take an automobile." 4. "Do not travel this late in the pregnancy."

Answer: 2 Explanation: 1. Traveling by bus is similar to traveling by automobile, which does not allow for frequent enough ambulation. 2. As pregnancy progresses, long-distance trips are best taken by plane. 3. Automobile travel does not allow for frequent enough ambulation. 4. It is not necessary to cease travel altogether.

10) The nurse is working in a clinic where children from several cultures are seen. What should the nurse do as a first step toward the goal of personal cultural competence? 1. Enhance cultural skills. 2. Gain cultural awareness. 3. Seek cultural encounters. 4. Acquire cultural knowledge.

Answer: 2 Explanation: 1. Ways to enhance cultural skill include learning a prevalent language or learning how to recognize health-manifesting skin color variations in different races. 2. Without cultural awareness, healthcare givers tend to project their own cultural responses onto foreign-born clients; clients from different socioeconomic, religious, or educational groups; or clients from different regions of the country. 3. During daily interactions with clients from diverse backgrounds, these cultural encounters allow the nurse to appreciate the uniqueness of individuals from varying backgrounds. 4. Acquiring cultural knowledge includes studying information about the beliefs, biologic variations, and favored treatments of specific cultural groups. This would be important; however, it is not the first step toward the goal of personal cultural competence.

The nurse suspects that a pregnant client is a substance user. Which approach should the nurse take during the health history? 1. Explaining how harmful drugs can be for her baby. 2. Asking the woman directly, "Do you use any street drugs?" 3. Asking the woman if she would like to talk to a counselor. 4. Asking some questions about over-the-counter medications and avoiding the mention of illicit drugs.

Answer: 2 Explanation: 1. When talking to clients in a therapeutic manner, it is important not to be threatening or judgmental. Explaining how harmful drugs can be for her baby is an example of being judgmental. 2. The best method of finding out if a client is using substances is to be direct and ask the question in a direct fashion without prejudice, bias, or negative body language. Lack of judgmental attitudes/body language typically results in honest answers. 3. It is the responsibility of the nurse to question the client. 4. It is the responsibility of the nurse not to avoid the issue.

A woman who is at 12 weeks' gestation asks the nurse if she can undergo chorionic villus sampling (CVS) testing in order to determine whether her baby has a neural tube defect. Which response is best? 1. "No, because CVS testing is not performed until after 20 weeks' gestation." 2. "No, because CVS testing alone at any stage cannot detect neural tube defects." 3. "Yes, at 12 weeks' gestation, CVS can be used to diagnose a neural tube defect." 4. "Yes, at 12 weeks' gestation, CVS is combined with amniocentesis to diagnose neural tube defects."

Answer: 2 Explanation: 1. While CVS is typically performed between 10 and 13 weeks' gestation, this test cannot detect neural tube defects. 2. Because CVS testing is performed so early in the pregnancy, it cannot detect neural tube defects. 3. CVS is typically performed between 10 and 13 weeks' gestation; however, CVS does not detect neural tube defects. 4. CVS is typically performed between 10 and 13 weeks' gestation; however, amniocentesis is not performed until 15 weeks' gestation.

A woman at 30 weeks' gestation and a history of sickle cell anemia is experiencing fever, chills, and diarrhea for 3 days. What are the most serious potential complications that this client faces? 1. Severe lethargy 2. Sickle cell crisis 3. Electrolyte imbalance 4. Fetal neural tube defects

Answer: 2 Explanation: 1. While the client may develop severe lethargy, her greatest risk concerns development of sickle cell crisis. 2. Dehydration and fever can trigger sickling and crisis; for this reason, maternal infections are treated promptly. 3. While the client may experience electrolyte imbalance, sickle cell crisis is the most serious potential complication of dehydration and fever. 4. Fever, chills, and dehydration in the client with sickle cell anemia are not associated with an increased incidence of neural tube defects.

A woman who is at 15 weeks' gestation received normal chorionic villus sampling (CVS) results and abnormal quadruple screen test results. For detection of congenital anomalies, which test should the nurse expect the woman to be offered next? 1. Ultrasound 2. Amniocentesis 3. Non-stress test (NST) 4. Contraction stress test (CST)

Answer: 2 Explanation: 1. While ultrasound has many uses, it is not useful in the diagnosis of congenital anomalies. 2. Women who have a normal CVS and an abnormal quadruple screen test would be offered amniocentesis to screen for congenital anomalies. 3. The non-stress test is used to assess fetal status. 4. The contraction stress test is used to assess fetal status.

The nurse receives a phone call from a client who thinks she is pregnant. The client reports that she has regular menses that occur every 28 days and last 5 days. The first day of her last menses was April 10. What is the client's estimated date of delivery (EDD)? 1. January 10 2. January 17 3. December 3 4. November 13

Answer: 2 Explanation: Naegele's rule is to add 7 days to the last menstrual period (LMP) and subtract 3 months. The LMP is April 10; therefore, January 17 is the EDD.

A pregnant woman is being excavated from the back seat of a motor vehicle after a crash. In which order should this victim receive emergency care? 1. Apply oxygen. 2. Establish an airway. 3. Monitor fetal activity. 4. Position on the left side. 5. Administer intravenous fluids.

Answer: 2, 1, 5, 4, 3Explanation: 1. Applying oxygen occurs after an airway is established. 2. The first action is to establish an airway. 3. Monitoring fetal activity occurs after the victim is stabilized. 4. Positioning on the left side helps prevent hypotension. 5. Intravenous fluids are provided to prevent shock and maintain circulation.

A family is scheduled for a genetic consultation. What should the nurse explain to the family about this appointment? Select all that apply. 1. A diagnosis will be made. 2. Photographs may be taken. 3. An interview will be conducted. 4. A geneticist will examine the child. 5. The appointment can last several hours.

Answer: 2, 3, 4, 5Explanation: 1. A definitive diagnosis may not be possible at the initial visit. 2. Photos may be taken. 3. An interview will be done with the parents and the child. 4. A geneticist will examine the child and possibly the parents. 5. The initial visit can last several hours.

A pregnant client is hesitant to have nuchal translucency testing. What should the nurse explain as being the advantages of having this test? Select all that apply. 1. It has a high false-positive rate. 2. It is performed early in the pregnancy. 3. There is no risk of spontaneous abortion. 4. It provides reassurance of the fetus's development. 5. It accurately detects 90% of Down syndrome fetuses.

Answer: 2, 3, 4, 5Explanation: 1. A high false-positive rate would be a disadvantage of this test. 2. Nuchal translucency testing can be performed in the first trimester to determine if a fetus is at risk for chromosomal disorders. 3. Since it is noninvasive, there is no risk of spontaneous abortion. 4. A normal result can provide reassurance to the woman that her baby most likely does not have a chromosomal disorder. 5. Nuchal translucency testing accurately detects 90% of fetuses with Down syndrome.

The nurse is preparing information about reproduction for high school students. What should the nurse include about the production of sperm? Select all that apply. 1. Diploid cells are formed. 2. Sperm cells develop through meiotic division. 3. Four spermatids are formed during the second meiotic division. 4. Spermatocytes contain either a double-structured X or Y sex chromosome. 5. Two secondary spermatocytes contain 22 double-structured autosomal chromosomes.

Answer: 2, 3, 4, 5Explanation: 1. The formation of diploid cells occurs during mitosis. 2. Meiosis occurs during gametogenesis, the process by which sperm are produced. 3. During the second meiotic division, they divide to form four spermatids, each with the haploid number of chromosomes. 4. Spermatocytes contain either a double-structured X sex chromosome or a double-structured Y sex chromosome. 5. During the first meiotic division, the spermatogonium replicates and forms two cells called secondary spermatocytes, each of which contains 22 double-structured autosomal chromosomes.

18) The nurse is assisting with the design of a study to analyze changes in maternal mortality. What should the nurse keep in mind when planning the design? Select all that apply. 1. The marital status of maternity clients 2. The use of hospitals by maternity clients 3. The prevention of infection with antibiotics 4. The availability of blood products for transfusions 5. The establishment of care centers for high-risk mothers

Answer: 2, 3, 4, 5Explanation: 1. The marital status of maternity clients would contribute to a study about infant mortality. 2. Factors influencing maternal mortality include the increased use of hospitals by maternity clients. 3. Factors influencing maternal mortality include the prevention infection with antibiotics. 4. Factors influencing maternal mortality include the availability of blood products for transfusions. 5. Factors influencing maternal mortality include the establishment of care centers for high-risk mothers.

The nurse is concerned that a pregnant client is experiencing depression. Which potential health issues should the nurse include when planning care for this client? Select all that apply. 1. Alcohol use 2. Preterm birth 3. Poor appetite 4. Poor weight gain 5. Antenatal hemorrhage

Answer: 2, 3, 4Explanation: 1. A pregnant client with bipolar disorder is at risk for alcohol use. 2. A pregnant client with depression is at risk for preterm birth. 3. A pregnant client with depression is at risk for poor appetite. 4. A pregnant client with depression is at risk for poor weight gain. 5. A pregnant client with schizophrenia is at risk for antenatal hemorrhage.

The nurse suspects that a home care client is experiencing the tension-building phase within the cycle of violence. What observations caused the nurse to make this clinical determination? Select all that apply. 1. Spouse ignoring the client 2. Spouse yelling at the client 3. Client asking the nurse to leave 4. Client apologizing to the spouse 5. Spouse throwing items at the client

Answer: 2, 3, 4Explanation: 1. During the tension-building phase of the cycle of violence the batterer demonstrates power and control. Ignoring the client could occur during the tranquil loving phase. 2. During the tension-building phase of the cycle of violence the batterer demonstrates power and control which is characterized by anger and arguing. 3. During the tension-building phase of the cycle of violence the batterer demonstrates power and control; however, the woman may believe the escalation of anger can be controlled by her own actions. Asking the nurse to leave would be a controllable action. 4. During the tension-building phase of the cycle of violence the batterer demonstrates power and control; however, the woman may blame herself and apologize for her actions. 5. Throwing things at the client would occur during the acute battering incident.

Which issues should the nurse consider when counseling a client on contraceptive methods? Select all that apply. 1. Age at menarche 2. Efficacy of the method 3. Future childbearing plans 4. Whether the client is a vegetarian 5. Cultural perspectives on menstruation and pregnancy

Answer: 2, 3, 5 Explanation: 1. Age at menarche has no impact on contraceptive method use. 2. Efficacy of contraceptive methods varies and must be considered when discussing contraception with clients. When pregnancy is medically contraindicated, high-efficacy methods (such as an IUD, hormonal methods, or sterilization) should be discussed with the client. When the client would like to avoid pregnancy at this time, but pregnancy is not medically contraindicated, lower-efficacy methods (such as diaphragm, cervical cap, or Today sponge) could be discussed. 3. If a client desires children in the future, sterilization methods would be inappropriate to discuss. 4. Vegetarianism has no impact on contraceptive method use. 5. Cultural and religious beliefs, practices, and sanctions must be considered when discussing contraception with clients in order to avoid insulting a client for whom a particular type of contraceptive method is prohibited by her background.

A pregnant client who restricted the intake of fat, protein, and sugar to prevent a large weight gain delivers a small-for-gestational-age fetus. What long-term health problems is this child at risk for developing as an adult? Select all that apply. 1. Arthritis 2. Diabetes 3. Hypertension 4. Cystic fibrosis 5. Coronary heart disease

Answer: 2, 3, 5Explanation: 1. Arthritis is not specifically linked to nutritional deficiencies in utero. 2. Poor maternal nutrition may also predispose babies who were small or disproportionate at birth to the development of adult diabetes. 3. Poor maternal nutrition may also predispose babies who were small or disproportionate at birth to the development of adult hypertension. 4. Cystic fibrosis is a genetic disease seen in pediatric clients. 5. Poor maternal nutrition may also predispose babies who were small or disproportionate at birth to the development of adult coronary heart disease.

During a home visit the nurse observes a pregnant client assess fetal activity. Which observations indicate that the client understands the correct process for this count? Select all that apply. 1. Sits in a chair 2. Assumes a side-lying position 3. Counts the same time every day 4. Watches television while counting 5. Begins counting 1 hour after a meal

Answer: 2, 3, 5Explanation: 1. The client should be in a side-lying position when assessing fetal activity. 2. A side-lying position is the position for assessing fetal activity. 3. The count should be conducted the same time every day. 4. The environment should be quiet during the count. 5. The count should occur about 1 hour after a meal.

The nurse learns that a client is being considered for a total abdominal hysterectomy. Which health problem should the nurse suspect this client is experiencing? Select all that apply. 1. Small fibroids 2. Ovarian cancer 3. Cervical cancer 4. Pelvic relaxation 5. Abnormal uterine bleeding

Answer: 2, 3Explanation: 1. Vaginal hysterectomy is generally done for small fibroids. 2. Abdominal hysterectomy is the usual treatment for cancer of the ovary. 3. Abdominal hysterectomy is the usual treatment for cancer of the cervix. 4. Vaginal hysterectomy is generally done for pelvic relaxation. 5. Vaginal hysterectomy is generally done for abnormal uterine bleeding.

A 32-year-old pregnant client is diagnosed with active tuberculosis (TB). What fetal health issues is this client at risk for developing? Select all that apply. 1. Cleft palate 2. Preterm labor 3. Microcephaly 4. Spontaneous abortion 5. Suboptimal weight gain

Answer: 2, 4, 5Explanation: 1. Infants of women taking prednisone or prednisolone for rheumatoid arthritis during the first trimester have a slightly increased risk of cleft palate. 2. Women with TB have a higher rate of preterm labor. 3. Women with untreated hyperphenylalaninemia have an increased incidence of microcephaly. 4. Women with TB have a higher rate of spontaneous abortion. 5. Women with TB have a higher rate of suboptimal weight gain.

The emergency room nurse is caring for a victim of sexual assault. What pharmacologic intervention should the nurse prepare to discuss with the victim? Select all that apply. 1. Prophylactic analgesics 2. Postcoital contraceptive therapy 3. Prophylactic immunizations for tetanus 4. Postexposure prophylaxis with HIV antiviral medications 5. Prophylactic treatment for sexually transmitted infections (STIs)

Answer: 2, 4, 5Explanation: 1. Pain medications are not routinely required for this situation. 2. Postcoital contraceptive therapy should be offered to the victim. 3. Prophylactic tetanus immunization is not routinely required for this situation. 4. Postexposure prophylaxis with HIV antiviral medications should be offered to the victim. 5. Prophylactic treatment for STIs should be offered to the victim.

A client undergoing infertility treatment states that her partner is angry all of the time since beginning treatment and is very negative in comments made about the likelihood of their achieving pregnancy. The client states, "I was angry and depressed, but now I am dedicated to following through with treatment and hoping we get pregnant." What statements best describe the partner's behavior? Select all that apply. 1. Showing that he will not be a good parent 2. In a different stage of grief than the client 3. Feeling guilty about not being able to father a child 4. Having difficulty accepting the reality of their infertility 5. Exhibiting signs of the anger stage of grieving the loss of their dreams of having children

Answer: 2, 5 Explanation: 1. Being in the anger stage of grief is expected and normal and has no bearing on parenting ability. 2. The client is in the acceptance stage of grief, while the partner is in the anger stage. It is common and normal for families to be in different stages of the grieving process. 3. Guilt would manifest as feelings that it is his fault that pregnancy has not yet occurred. The client is describing anger. 4. The partner is in the anger stage of grief. Lack of acceptance would manifest as not believing that the diagnosis is correct. 5. The client's description of her partner correlates with the anger stage of grief. Couples often experience the stages of grief when infertility is diagnosed because childbearing is an expected outcome in marriage; the inability to become pregnant is the loss of the dream of parenthood.

The nurse suspects that a pregnant client is experiencing effects to the gastrointestinal system because of elevated progesterone levels. What did the nurse assess to make this clinical determination? Select all that apply. 1. Nausea 2. Bloating 3. Diarrhea 4. Vomiting 5. Constipation

Answer: 2, 5Explanation: 1. Nausea is common during the first trimester and may result from several factors, including elevated human chorionic gonadotropin (hCG) levels, relaxation of the smooth muscle of the stomach, and changed carbohydrate metabolism. 2. Elevated progesterone levels cause smooth muscle relaxation, resulting in delayed gastric emptying and decreased peristalsis. As a result the pregnant woman may complain of bloating. 3. Diarrhea is not an expected effect of pregnancy on the gastrointestinal system. 4. Vomiting is common during the first trimester and may result from several factors, including elevated human chorionic gonadotropin (hCG) levels, relaxation of the smooth muscle of the stomach, and changed carbohydrate metabolism 5. Elevated progesterone levels cause smooth muscle relaxation, resulting in delayed gastric emptying and decreased peristalsis. As a result the pregnant woman may complain of constipation.

A client asks about the difference between an abdominal hysterectomy and a laparoscopic-assisted vaginal hysterectomy (LAVH). What should the nurse explain to this client? Select all that apply. 1. Recovery will be quicker with an abdominal hysterectomy. 2. The results are nearly the same between the two surgical approaches. 3. An abdominal hysterectomy is indicated for uterine bleeding and small fibroids. 4. Hospitalization will be longer with a laparoscopic-assisted vaginal hysterectomy. 5. A laparoscopic-assisted vaginal hysterectomy does not have an abdominal incision.

Answer: 2, 5Explanation: 1. Recovery is quicker with a LAVH. 2. A benefit is that the surgeon can achieve results similar to those of a total abdominal hysterectomy when using LAVH, but without a large abdominal incision. 3. A vaginal hysterectomy is indicated for uterine bleeding and small fibroids. 4. Hospitalization will be longer with an abdominal hysterectomy. 5. A benefit is that a LAVH does not have a large abdominal incision.

The nurse learns that a client from a different country and culture was married to her first cousin before moving to the United States. The couple is expecting their first child in 4 months. What should the nurse consider when counseling this client? 1. Birth control 2. Carrier testing 3. Pregnancy termination 4. Legal issues in the United States

Answer: 2Explanation: 1. Counseling on birth control would be inappropriate. 2. In many other cultures, marriage of first cousins and others who are related by blood is customary or even preferred. Genetic counseling involves identifying consanguinity and offering risk information, carrier testing, and nondirective counseling. 3. Counseling on pregnancy termination would be inappropriate. 4. The legal issues would not apply since the marriage occurred before coming to the United States.

Which client should the nurse identify as being a multipara? 1. A client at 28 weeks' gestation with no previous pregnancies 2. A client at 32 weeks' gestation who previously delivered one term infant 3. A client at 13 weeks' gestation who previously delivered two term infants 4. A client at 34 weeks' gestation who previously had one spontaneous abortion

Answer: 3 Explanation: 1. A woman who has had no births at more than 20 weeks' gestation is considered a nullipara. 2. A woman who has had one birth at more than 20 weeks' gestation, regardless of whether the infant was born alive or dead, is considered a primipara. 3. A woman who has had two or more births at more than 20 weeks' gestation is considered a multipara. 4. A woman who has had no births at more than 20 weeks' gestation is considered a nullipara.

The nurse is preparing an education session for women on prevention of urinary tract infections (UTIs). Which statement should be included? 1. Lower urinary tract infections rarely occur in women. 2. Back pain often develops with a lower urinary tract infection. 3. The most common causative organism of cystitis is Escherichia coli. 4. Wiping from back to front after a bowel movement will help prevent a UTI.

Answer: 3 Explanation: 1. About 60% of women will experience an episode of cystitis during their lifetime. 2. Low back or flank pain is a sign of pyelonephritis, which is an upper urinary tract infection. Signs of a lower UTI include dysuria, urinary frequency, and urinary urgency. 3. Because E. coli is a common bacterium in the bowel and the female urethra is short and close to the anus, cross-contamination of bowel bacteria into the female urinary tract is common. 4. Wiping from back to front increases the risk of UTIs because the E. coli of the bowel is being drawn toward the urethra. Women should be instructed always to wipe from front to back.

A woman at 28 weeks' gestation reports not feeling the baby move for over 30 minutes. How should the nurse respond first? 1. "When did you eat last?" 2. "Have you been smoking?" 3. "Your baby might be asleep." 4. "You need to go to the emergency room immediately for further evaluation."

Answer: 3 Explanation: 1. After meals, an infant typically is active and moving. 2. Smoking typically will stimulate the infant. 3. Lack of fetal activity for 30 minutes typically is insignificant and means only that the infant is sleeping. The mother should continue to observe for fetal movements over the next 2.5 hours. If a lack of fetal movements continues, she should contact the healthcare provider. 4. The mother would need to come to be examined if there had been no fetal activity for several hours.

What is the best indicator that the client is experiencing menopause? 1. Hot flashes and night sweats 2. No menses for 8 consecutive months 3. High serum follicle-stimulating hormone (FSH) with low serum estrogen 4. Diagnosis of osteoporosis 4 months ago

Answer: 3 Explanation: 1. Although hot flashes and night sweats are common in menopause, laboratory values or 12 months of amenorrhea are better indicators. 2. Menopause is defined as 12 months of amenorrhea. 3. Examining serum levels of the hormones FSH and estrogen is a very accurate indication of menopause. 4. Menopause is not the only cause of osteoporosis; therefore, the diagnosis of osteoporosis 4 months ago is not an indicator of menopause.

Which .client in the gynecology clinic should the nurse assess first? 1. 31-year-old, reports increasing dyspareunia 2. 15-year-old, no menses for past 4 months 3. 22-year-old, using tampons, T = 102°F, P = 122, BP = 70/55 4. 18-year-old seeking information on contraception methods

Answer: 3 Explanation: 1. Although this client might have endometriosis, dyspareunia is not a life-threatening condition. 2. Secondary amenorrhea can be caused by pregnancy. Teen pregnancy is a high risk, but no indication is given that the client is exhibiting a life-threatening condition. 3. A client using tampons who is febrile, tachycardic, and hypotensive might have toxic shock syndrome. Hypotension is life threatening; this client should be seen immediately. 4. Unplanned pregnancy and sexually transmitted infections can be problematic in the future, but this client exhibits no signs or symptoms of a life-threatening condition at this time.

A client in the prenatal clinic believes she is pregnant because she has not menstruated for 3 months, and her breasts are getting bigger. What response by the nurse is best? 1. "Lack of menses and breast enlargement are presumptive signs of pregnancy." 2. "The changes you are describing are definitely indicators that you are pregnant." 3. "Lack of menses can be caused by many things. We need to do a pregnancy test." 4. "Breast and menstrual changes are positive signs of pregnancy. Congratulations."

Answer: 3 Explanation: 1. Although this is true, amenorrhea and breast enlargement also can be caused by weight gain and other conditions. A pregnancy test is needed to determine whether the client is pregnant. 2. This statement is false because amenorrhea and breast enlargement are presumptive signs of pregnancy because they can be caused by other conditions. 3. This is a true statement and addresses that these changes could be caused by things other than pregnancy. 4. This statement is false because amenorrhea and breast enlargement are presumptive signs of pregnancy. It is too early to determine if congratulations are in order.

A client who is at 10 weeks' gestation is concerned about the amount of saliva that is in her mouth since she seems to be spitting when she talks. How should the nurse respond? 1. "You should avoid astringent mouthwashes and chewing gum." 2. "That's called ptyalism, and it's usually caused by increased salt intake during the second trimester." 3. "Excess salivation commonly occurs during the first trimester, although the cause is unknown." 4. "Let's schedule you for a doctor's appointment, because excessive salivation can signal a complication of pregnancy."

Answer: 3 Explanation: 1. Astringent mouthwashes, chewing gum, and sucking hard candy may help relieve the bitter taste that often accompanies ptyalism. 2. Ptyalism, which is excess production of saliva, usually occurs during the first trimester and the cause is unknown. 3. Ptyalism, which is excess production of saliva, commonly occurs during the first trimester and the cause is unknown. 4. Excess salivation, also called ptyalism, is a normal occurrence in women during the first trimester.

A client is identified as having hepatitis B surface antigen (HBsAG) early in her pregnancy. Which client statement about the labor and birth process and having hepatitis B infection indicates the need for additional teaching? 1. "Breastfeeding is a good feeding method for my baby." 2. "My baby will get a bath as soon as its temperature is stable." 3. "An internal fetal monitor will be applied as soon as possible during labor." 4. "Two shots will be given to my baby to prevent transmission of hepatitis B."

Answer: 3 Explanation: 1. Breastfeeding is not contraindicated in a client with HBsAG. 2. The presence of HBsAG indicates that the client is contagious for and capable of transmitting hepatitis B. Perinatal transmission is most likely to occur at the time of birth; thus, measures are taken to prevent exposing the fetus to the mother's blood and body fluids and to clean the baby's skin thoroughly of fluids as soon as possible after birth. 3. An internal fetal monitor will be avoided. 4. A newborn of a mother with HBsAG will receive an injection of hepatitis B immune globulin and a hepatitis B vaccine injection.

A 31-year-old woman with normal ovaries, a normal prolactin level, and an intact pituitary gland is undergoing initial pharmacologic treatment of anovulation. Which medication should the nurse anticipate being prescribed for this client? 1. Bromocriptine (Parlodel) 2. Metformin (Glucophage) 3. Clomiphene citrate (Clomid or Serophene) 4. Human menopausal gonadotropins (hMGs)

Answer: 3 Explanation: 1. Bromocriptine (Parlodel) is used to treat hyperprolactinemia accompanied by anovulation. 2. Oral hypoglycemic agents such as metformin (Glucophage) are used for inducing ovulation in women with polycystic ovary disease (PCOS). 3. Clomiphene citrate (Clomid or Serophene) is a common first-line therapy for inducing ovulation in women with normal ovaries, normal prolactin level, and intact pituitary gland. 4. Human menopausal gonadotropins (hMGs) represent a second line of therapy in women who fail to ovulate or conceive with clomiphene citrate therapy.

The nurse is explaining a client's abnormal Pap smear results. Which statement should the nurse include? 1. "Your cervix needs to be treated with cryotherapy." 2. "The Pap smear is used to diagnose cervical cancer." 3. "Colposcopy to further examine your cervix is the next step." 4. "A loop electrosurgical excision procedure (LEEP) is needed."

Answer: 3 Explanation: 1. Cryotherapy, or freezing of the cervix, is one treatment option for precancerous cervical lesions. However, this client does not yet have a diagnosis; she has only had an abnormal screening test. 2. The Pap smear is a screening tool for cervical abnormalities; it is not diagnostic. 3. Colposcopy is an examination of the cervix through a magnifying device. Solutions are often painted onto the cervix and surrounding tissue and observed for changes secondary to the application of the solution. Biopsy samples are taken of suspected abnormal tissue and sent for pathologic examination and diagnosis. Endocervical canal biopsy is often undertaken with colposcopy. 4. Although LEEP (the removal of the surface tissue of the cervix) might be performed to treat cervical dysplasia or carcinoma in situ, this client has not had a diagnostic examination yet.

4) During the hospital admission process, a child's parent asks for information about family-centered care. What should the nurse explain to this parent? 1. Mother is the principal caregiver in each family. 2. Father is the leader in each home; thus, all communications should include him. 3. Family serves as the constant influence and continuing support in the child's life. 4. Child's physician is the key person in ensuring the health of a child is maintained.

Answer: 3 Explanation: 1. Culturally competent care recognizes that both matriarchal and patriarchal households exist. 2. Culturally competent care recognizes that both matriarchal and patriarchal households exist. 3. The foundation for the development of trusting relationships and partnerships with families is the recognition that the family is the principal caregiver, knows the unique nature of each individual child best, plays the vital role of meeting the child's needs, and is responsible for ensuring each child's health. 4. The physician is not present during the day-to-day routines in a child's life.

The nurse is teaching a class to women who were recently diagnosed with benign breast disease (BBD), commonly known as fibrocystic breast disease. One of the participants reports increased swelling, pain, and pressure in her breasts just before menstruation. What is the best response by the nurse? 1. "It's best to make an appointment with an oncologist." 2. "The pain may be caused by thinning of the normal breast tissue." 3. "Consider asking your healthcare provider about adding a mild diuretic to your regimen." 4. "Breast swelling and pressure are expected symptoms, but pain is abnormal and should be evaluated by your physician."

Answer: 3 Explanation: 1. Cyclic breast pain, swelling, and tenderness are common symptoms associated with BBD. Generally, fibrocystic changes are not a risk factor for breast cancer. 2. The pathology of BBD involves fibrosis, which is a thickening of the normal breast tissue. 3. Treatment of BBD may include taking a mild diuretic during the week prior to the onset of menses to counteract fluid retention, relieve pressure in the breast, and help decrease pain. 4. Common symptoms associated with BBD include cyclic breast pain, tenderness, and swelling.

The nurse is explaining the importance of fetal activity assessment to a pregnant client. What should the nurse do to best reinforce the significance of fetal kick monitoring? 1. Perform daily phone calls to the client at work or home. 2. Ask the client to remember to count the fetal movements. 3. Explain the reason for counting fetal movement to the client. 4. Review the client's written record of fetal movement at each visit.

Answer: 3 Explanation: 1. Daily phone calls would take emphasis away from the importance of the client's counting of fetal movement. 2. Writing down the count is more accurate than the client's simply remembering. When the nurse examines the written record the client has kept, it reinforces the importance of the record and improves the likelihood of continued record keeping. 3. Many healthcare providers encourage pregnant women to monitor their unborn child's well-being by regularly assessing fetal activity beginning at 28 weeks' gestation. Vigorous activity generally provides reassurance of fetal well-being, but a marked decrease in activity or cessation of movement may indicate a problem that needs immediate evaluation. 4. Knowing the reasons for the counting will increase understanding of the process but will not reinforce the significance of the task.

A client at 24 weeks has a history of class II heart disease secondary to rheumatic fever. What should the nurse expect to see in the medical record? 1. Dyspnea and chest pain with mild exertion 2. Elective cesarean birth scheduled for 37 weeks 3. Discussed need for labor epidural and vacuum extraction 4. Respiratory rate 28, pulse 110, 3+ pre-tibial edema bilaterally

Answer: 3 Explanation: 1. Dyspnea and angina with mild exertion are not expected with class II heart disease even during pregnancy, but are symptoms seen in class IV heart disease. 2. Cesarean birth is only undertaken in cardiac clients for fetal or maternal intrapartal indications, not for cardiac reasons alone. 3. Lumbar epidural analgesia decreases the stress response during labor, while vacuum extraction or forceps decreases maternal pushing efforts. Both of these decrease stress on the heart during birth. 4. 3+ pre-tibial edema is never an expected finding during pregnancy. Pulse over 100 and respiratory rate over 24 are indicators of cardiac decompensation.

A child is not enrolled in the Children's Health Insurance Program (CHIP). What should the nurse do to encourage the family to consider enrolling the child in this program? 1. Assessment of the details of the family's income and expenditures 2. Case management to limit costly, unnecessary duplication of services 3. Advocacy for the child by encouraging the family to investigate its CHIP eligibility 4. Education of the family about the need for keeping regular well-child visit appointments.

Answer: 3 Explanation: 1. Financial assessment is more commonly the function of a social worker. 2. The case management activity mentioned will not provide a source of funding. 3. In the role of an advocate, a nurse will advance the interests of another by suggesting the family investigate its CHIP eligibility. 4. The educational effort described will not provide a source of funding.

The nurse is preparing a prenatal class about infant feeding methods to include maternal nutritional requirements for breastfeeding and formula-feeding. What statement should the nurse include? 1. "Formula-feeding mothers need a high protein intake to avoid fatigue." 2. "Producing breast milk requires calories, but any source of food is fine." 3. "Breastfeeding requires a continued high intake of protein and calcium." 4. "Formula-feeding mothers should protect their health with a lot of calcium."

Answer: 3 Explanation: 1. Formula-feeding moms do not need additional nutrients. 2. Although any food source would provide the additional calories, these needs are best met by using increased protein intake as the source for the required additional calories to support milk production. 3. Lactation requires calories, along with increased protein and calcium intake. 4. Formula-feeding mothers do not need additional nutrients.

At her first prenatal visit, the client states, "I'm 5 weeks pregnant now and I would like to hear my baby's heartbeat today." How should the nurse respond? 1. Anticipate that the client will be scheduled for Doppler ultrasound. 2. Prepare to assist with auscultation of the fetal heartbeat using a fetoscope. 3 Explain to the client that the fetal heartbeat is not yet detectable at 5 weeks' gestation. 4. Explain to the client that the fetal heart does not begin to beat until approximately 7 weeks' gestation.

Answer: 3 Explanation: 1. Generally, fetal heart tones cannot be heard until approximately the 8th week of pregnancy by ultrasound Doppler device. 2. Fetal heart tones can be identified through use of a fetoscope at approximately the 20th week of pregnancy. 3. While the tubular heart begins to form during the 3rd week, fetal heart tones generally are not detectable until at least 7 weeks' gestation. 4. The fetal heart begins to beat by the 4th week of gestation.

Which statement by a client pregnant with twins would indicate that teaching was effective? 1. "Identical twins can be the same or different sex." 2. "Identical twins occur more frequently than fraternal twins." 3. "Congenital abnormalities are more prevalent in identical twins." 4. "Because of their birth relationship, fraternal twins are more similar to each other than if they had been born singly."

Answer: 3 Explanation: 1. Identical or monozygotic twins have identical chromosomal structures and therefore are always the same sex. 2. Dizygotic, or fraternal, twins occur more frequently than do monozygotic twins. 3. Due to variations in the timing of the splitting of the embryo, congenital abnormalities are more common in monozygotic twins. 4. Fraternal twins are not more similar to each other than if they had been born singly.

The pregnant client's prenatal record indicates that she is a gravida 4 para 2022. How should the nurse interpret these data about the client? 1. Is pro-abortion 2. Has four living children 3. Delivered two term infants 4. Delivered two infants preterm

Answer: 3 Explanation: 1. In the four-digit number, the third digit indicates the number of abortions the client has experienced. Because abortion may be spontaneous or therapeutic, this number does not necessarily reflect a woman's stance on surgical abortion. 2. In the four-digit number, the fourth number indicates the number of living children, which is 2. 3. In the four-digit number, the first digit indicates the number of term infants born, which is 2. 4. In the four-digit number, the second digit indicates the number of preterm births, so the client has had no preterm births.

What is the major focus of the nurse practitioner (NP)? 1. Leadership 2. Tertiary prevention 3. Physical and psychosocial clinical assessment 4. Independent care of the high-risk, pregnant client

Answer: 3 Explanation: 1. Leadership might be a quality of the NP, but it is not the major focus. 2. The NP cannot do tertiary prevention as a major focus. 3. Physical and psychosocial clinical assessment is the major focus of the NP. 4. NPs cannot provide independent care of the high-risk pregnant client, but must work under a physician's supervision.

The physician has prescribed metronidazole (Flagyl) for a woman diagnosed with trichomoniasis. What should the nurse include when teaching about this medication? 1. "It will turn your urine orange." 2. "This medication could produce drowsiness." 3. "Both partners must be treated with the medication." 4. "Alcohol does not need to be avoided while taking this medication."

Answer: 3 Explanation: 1. Metronidazole does not turn the urine orange. 2. Metronidazole does not cause drowsiness. 3. Both partners should be treated with the medication. 4. Alcohol should be avoided.

The nurse is preparing an antenatal nutrition class for pregnant women. Which material should be included in the teaching? 1. During pregnancy, consumption of oily fish should be avoided. 2. Dietary protein can only be obtained through consuming dairy, meat, and eggs. 3. Nutritional iodine requirements generally can be met through intake of iodized salt. 4. Iron absorption is generally higher for vegetable products than for animal products.

Answer: 3 Explanation: 1. Oily fish provide the best source of docosahexaenoic acid (DHA), which may reduce the risk of preterm birth, preeclampsia, low birth weight, and enhance fetal and infant brain development. 2. Excluding dairy, meat, and eggs, adequate dietary protein can be obtained by consuming a varied diet with adequate caloric intake and plant-based proteins. 3. Intake of iodized salt generally provides the recommended intake of iodine. 4. Iron absorption is generally higher for animal products than for vegetable products.

The prenatal clinic nurse has received four phone calls. Which client should be called back first? 1. Multipara at 11 weeks with untreated hyperthyroidism describing the onset of vaginal bleeding 2. Multipara at 6 weeks with a seizure disorder inquiring what foods are good sources of folic acid 3. Primipara at 28 weeks with a history of asthma reporting difficulty breathing and shortness of breath 4. Primipara at 35 weeks with a positive hepatitis B surface antigen (HBsAG) wondering what treatment her baby will receive after birth

Answer: 3 Explanation: 1. Pregnant women with untreated hyperthyroidism have an increased risk of fetal loss. Vaginal bleeding at 11 weeks could indicate that spontaneous abortion is taking place. But the majority of spontaneous abortions prior to 12 weeks' gestation are complete and without complications. This client is not experiencing a normal pregnancy, but the health of both mother and fetus are not in immediate danger. 2. Women with seizure disorders should be started on folic acid supplements prior to pregnancy, and should continue throughout pregnancy. This client is not the highest priority. 3. Asthma exacerbations are most common between 24 and 36 weeks. Asthma attacks can lead to maternal hypoxia, which can lead to fetal hypoxia. This client is the top priority. 4. A client with a positive HBsAG is contagious for hepatitis B. The risk of transmission to the fetus at birth is reduced by bathing the neonate as soon as possible after birth and giving the infant immunoprophylaxis and the first HBsAG vaccine dose. The client seeking information about what will happen after delivery is a low priority when there are pregnant clients currently experiencing physiologic problems.

The nurse is working in a teen pregnancy clinic. In order to give the pregnant adolescent a role in her prenatal care, what should the nurse encourage the teen to do? 1. Choose the schedule of her prenatal visits 2. Decide if she wants her labor to be induced 3. Measure and record her weight at each visit 4. Choose the type of prenatal vitamin she takes

Answer: 3 Explanation: 1. Prenatal visit schedules are set to detect developing complications of pregnancy. 2. Induction of labor is a medical decision and should not be taken lightly. 3. Having the client weigh herself and record her weight provides her with information that indicates she is growing a healthy fetus. 4. Prenatal vitamins are prescribed by the certified nurse-midwife or the physician. Many formulations exist, and some might not be indicated for this client due to her nutritional practices and lab results. In addition, if the client is a member of a health maintenance organization, only certain medications (including prenatal vitamins) are accepted for coverage.

The client has delivered her first child at 39 weeks. How should the nurse document this type of delivery? 1. Preterm 2. Postterm 3. Full Term 4. Near term

Answer: 3 Explanation: 1. Preterm deliveries are those that occur prior to 36 completed weeks' gestation. 2. Postterm applies to births that occur after 42 weeks' gestation. 3. Full-term births occur between 39 weeks 0 days and 40 weeks 6 days. 4. Near term is not terminology used to describe birth.

A client has decided to use the NuvaRing vaginal contraceptive ring as her method of contraception. Which statement suggests the client needs further instruction? 1. "When I store my replacement rings, I should keep them in my refrigerator." 2. "The contraceptive ring provides a sustained release of low-dose contraceptive." 3. "Every 3 months, I will need to remove the contraceptive ring and replace it with a new one." 4. "I do not need to be examined in order to determine the contraceptive ring size that is correct for me."

Answer: 3 Explanation: 1. Replacement rings should be kept in the refrigerator to maintain integrity. 2. The contraceptive ring provides a low-dose, sustained-release hormonal contraceptive. 3. The ring is left in place for 3 weeks and then removed for 1 week to allow for withdrawal bleeding. 4. One size of the NuvaRing fits virtually all women.

A client who is at 32 weeks' gestation is determined to be at high risk for ABO incompatibility. Which intervention should the nurse anticipate implementing? 1. Intramuscular administration of 300 mcg of Rh immune globulin (RhoGAM). 2. Obtain an antibody screen (indirect Coombs test) to determine whether the client has developed isoimmunity. 3 Note the potential for ABO incompatibility and plan to carefully assess the neonate for the development of hyperbilirubinemia. 4. Notify the primary care provider and document the potential need for treatment of fetal hemolytic anemia in the baby after delivery.

Answer: 3 Explanation: 1. RhoGAM is administered to prevent sensitization after exposure to Rh-positive blood. 2. An antibody screen (indirect Coombs test) is done to determine whether an Rh-negative woman is sensitized (has developed isoimmunity) to the Rh antigen. 3. Unlike the situation with Rh incompatibility, antepartum treatment of ABO incompatibility is not warranted because it does not cause severe anemia. As part of the initial assessment, however, the nurse should note whether the potential for an ABO incompatibility exists in order to alert healthcare providers to the need for carefully assessing the newborn for the development of hyperbilirubinemia. 4. Unlike the situation with Rh incompatibility, antepartum treatment of ABO incompatibility is not warranted because it does not cause severe anemia.

A 32-year-old primipara who is at 8 weeks' gestation asks if she should expect any breathing changes as the pregnancy progresses. How should the nurse respond? 1. "By the third trimester, you will no longer feel as though you're short of breath." 2. "You may experience shortness of breath due to stretching of the round ligament." 3. "If you develop shortness of breath, it should improve in the last few weeks of your pregnancy, as lightening occurs." 4. "Shortness of breath is an abnormal finding during any stage of pregnancy, and it is considered a serious complication."

Answer: 3 Explanation: 1. Shortness of breath occurs as the uterus rises into the abdomen and causes pressure on the diaphragm. This problem worsens in the last trimester. 2. Round ligament stretching causes a "grabbing" sensation in the lower abdomen and inguinal area. 3. The primigravida experiences considerable relief from shortness of breath in the last few weeks of pregnancy, when lightening occurs and the fetus and uterus move down in the pelvis. 4. Because of decreased vital capacity from pressure of the enlarging uterus on the diaphragm, shortness of breath is a common problem of pregnancy.

The nurse manager is interviewing nurses for a position in an infertility clinic. Which statement best indicates that the interviewee understands the role of the nurse when working with infertile clients? 1. "This position is an assistant to the physician during diagnostic testing for infertility." 2. "My job will be teaching clients how to take their medications and scheduling tests." 3. "I will both teach and support families struggling with emotions as they attempt to become pregnant." 4. "Many of my duties will involve forming therapeutic relationships with clients struggling with infertility."

Answer: 3 Explanation: 1. Some assisting might be a part of this position; the role of the RN in an infertility clinic involves much teaching and providing emotional support to infertile clients. 2. Although teaching and facilitating scheduling are important, the emotions that families deal with during treatment for infertility must also be addressed. 3. This answer addresses the two main aspects of the RN working with infertile clients: emotional support and education. 4. Although this response addresses the emotional aspects of infertility, it does not mention providing support or teaching, which are also major components of the job.

A client in the third trimester of pregnancy reports working 8 hours a day as a cashier and stands when at work. What response by the nurse is the most appropriate? 1. "No problem. Your baby will be fine." 2. "Do you get regular breaks for eating?" 3. "Your risk of preterm labor is higher." 4. "Standing might increase ankle swelling."

Answer: 3 Explanation: 1. Standing more than 5 hours a day increases the risk of preterm labor. To be therapeutic in communication, avoid false reassurance. 2. Although breaks for eating, drinking, and toileting are important for pregnant employees, it is more important to tell the client about the increased risk of preterm labor. 3. Pregnant women who stand for more than 3 hours a day have an increased risk of preterm labor. Because preterm labor can put the infant's life at risk, this statement would be the highest priority. 4. Although this is true, it is less important than teaching the client about the risks of preterm labor when standing more than 3 hours a day.

While auscultating fetal heart tones, a client who is at 37 weeks' gestation and is in the supine position is experiencing dizziness, lightheadedness, and clammy skin. Which nursing action is the most appropriate? 1. Administer supplemental oxygen. 2. Help the client turn onto her right side. 3. Place a wedge beneath the client's right hip. 4. Prepare for administration of packed red blood cells (PRBCs).

Answer: 3 Explanation: 1. Supplemental oxygen is not required for supine hypotension syndrome. 2. Positioning the client on her right side would likely exacerbate the reduction in right atrial blood flow. 3. The client is verbalizing symptoms consistent with supine hypotension syndrome, in which compression of the vena cava by the uterus reduces right atrial blood flow. Signs and symptoms include decreased blood pressure, dizziness, pallor, and clamminess. Appropriate interventions include having the woman lie on her left side, or placing a pillow or wedge under her right hip as she lies in a supine position. 4. Packed red blood cell infusion is not required to treat supine hypotension syndrome.

A 20-year-old woman at 28 weeks' gestation has a history of past drug abuse and her urine screening indicates recent heroin use. What should the nurse recognize this client is at risk for developing? 1. Diabetes mellitus 2. Abruptio placentae 3. Erythroblastosis fetalis 4. Pregnancy-induced hypertension

Answer: 4 Explanation: 1. Diabetes is an endocrine disorder that is unrelated to drug use/abuse. 2. Abruptio placentae is seen more commonly with cocaine/crack use. 3. Erythroblastosis fetalis is secondary to physiologic blood disorders such as Rh incompatibility. 4. Women who use heroin are at risk for poor nutrition, anemia, and pregnancy-induced hypertension (or preeclampsia).

When preparing nutritional instruction, which pregnant client should the nurse consider the highest priority? 1. 40-year-old gravida 2 2. 35-year-old gravida 4 3. 15-year-old nulligravida 4. 22-year-old primigravida

Answer: 3 Explanation: 1. The 40-year-old client has completed her growth cycle, and her body can focus on diverting the nutritional needs to the fetus. 2. The 35-year-old client has completed her growth cycle, and her body can focus on diverting the nutritional needs to the fetus. 3. Adolescent clients typically are still in their own growth cycle. Suddenly, they have to supply nutrition for themselves and the fetus. This places them at greatest risk for malnutrition. 4. The 22-year-old client has completed her growth cycle, and her body can focus on diverting the nutritional needs to the fetus.

If only a small volume of sperm is discharged into the vagina, an insufficient amount of enzymes might be released when sperm encounters the ovum. What would be the result for pregnancy? 1. The block to polyspermy (cortical reaction) would not occur. 2. The fertilized ovum would be unable to implant in the uterus. 3. Sperm would be unable to penetrate the zona pellucida of the ovum. 4. Peristalsis of the fallopian tube would decrease, making it difficult for the ovum to enter the uterus.

Answer: 3 Explanation: 1. The answer choice of the block to polyspermy (cortical reaction) not occurring is incorrect because it is mediated by release of materials from cortical granules below the ovum's surface and not the result of low sperm count. 2. With a low sperm count it is unlikely that the ovum would be fertilized. 3. Sperm would be unable to penetrate the zona pellucida of the ovum because it takes hundreds of acrosomes (the result of the acrosomal reaction) to rupture and release enough hyaluronic acid to clear the way for a single sperm to penetrate the ovum's zona pellucida successfully. 4. "Peristalsis of the fallopian tube would decrease, making it difficult for the ovum to enter the uterus" is an incorrect statement.

The nurse receives the following report on a client who delivered 36 hours ago: para 1, rubella immune, A-negative, antibody screen negative, newborn B-positive, Coombs negative, discharge orders are written for both mother and newborn. What should be the priority action by the nurse? 1. Administer rubella vaccine. 2. Ask if she is breast- or bottlefeeding. 3. Determine if RhoGAM has been given. 4. Discuss the discharge education with the client.

Answer: 3 Explanation: 1. The client is rubella immune and does not need the rubella vaccine. 2. This is important but is not the top priority. 3. The client is A-negative and the newborn B-positive. The client needs RhoGAM prior to discharge. Without RhoGAM, the client will make antibodies against Rh-positive blood, and future pregnancies would be in jeopardy. 4. Discharge education is always important, but in this case it is not the most important action.

3) The nurse is comparing several different families' developmental stages. What serves as a marker for a family's developmental stage according to Duvall? 1. The father's age 2. The mother's age 3. The oldest child's age 4. The youngest child's age

Answer: 3 Explanation: 1. The father's age is not a marker, according to Duvall. 2. The mother's age is not a marker, according to Duvall. 2. The oldest child's age serves as a marker for the family's developmental stage, except in the last two stages, when children are no longer present. 4. The youngest child's age is not a marker, according to Duvall.

A 20-weeks'-gestation adolescent states that it is important not to have a baby that weighs too much and has been limiting her calories so that her current weight has dropped from 110 pounds to 106 pounds. How should the nurse respond? 1. "You are causing harm to your baby." 2. "You shouldn't be worrying about your figure." 3. "Your baby needs adequate nutrition to develop." 4. "It's okay to want a small baby when you're a teen."

Answer: 3 Explanation: 1. The first role of the nurse is to explain why food is important to the growing fetus, specifying how each food group will help the fetus develop. Next, the nurse must assist the pregnant adolescent to plan foods that she likes to eat from each food group. 2. Pregnant adolescents are just adapting to a new body image created by the changes of puberty when the pregnancy produces rapid and substantial body changes. The desire to maintain a socially desirable figure can lead to nutritional deficits. 3. Teens might not understand the physiology behind the profound body changes of pregnancy. 4. Anticipatory guidance in the body changes of pregnancy will assist the adolescent's adjustment to them. Although many teens are anxious, this teen is expressing a direct nutritional deficit.

A prenatal client states, "The doctor said he might have to cut my cervix so the baby can get out during delivery." Based upon this statement, what structure should the nurse define when teaching about an episiotomy? 1. Mons pubis 2. Labia majora 3. Perineal body 4. Vaginal vestibule

Answer: 3 Explanation: 1. The mons pubis is a softly rounded mound of subcutaneous fatty tissue that covers the front portion of the symphysis pubis. 2. The labia majora are longitudinal, raised folds of pigmented skin located on either side of the vulvar cleft. 3. The perineal body, which is located between the lower part of the vagina and the anus, is often the site of an episiotomy or lacerations during childbirth. 4. The vaginal vestibule contains the vaginal opening, which is the border between the external and internal genitals.

At 32 weeks' gestation, a woman is scheduled for a second non-stress test. Which client response indicates an adequate understanding of this procedure? 1. "I'll have an IV started before the test." 2. "I need to have a full bladder for this test." 3. "I cannot get up and walk around during the test." 4. "I must avoid drinks containing caffeine for 24 hours before the test."

Answer: 3 Explanation: 1. There is no IV needed to administer medications. 2. Clients usually are asked to have their bladders full only for ultrasounds. 3. The purpose of the non-stress test is to determine the results of movement on fetal heart rate. The client will have to lie still on her side during the procedure. 4. Caffeine might cause the infant to be more active and cause the test to go more quickly.

Which client is at greatest risk for developing Chlamydia trachomatis infection? 1. 35-year-old woman on oral contraceptives 2. 22-year-old mother of two, developed dyspareunia 3. 16-year-old, sexually active, using no contraceptive 4. 48-year-old woman with hot flashes and night sweats

Answer: 3 Explanation: 1. There is no correlation between oral contraceptive use and an increased rate of chlamydial infection. Additionally, chlamydial infection is more commonly seen in young women. 2. Dyspareunia sometimes develops with chlamydial infection, but dyspareunia is not a symptom specific to chlamydial infection. 3. Teens have the highest incidence of sexually transmitted infections, especially chlamydial infection. A client not using contraceptives is not using condoms, which decrease the risk of contracting a sexually transmitted infection (STI). 4. This client is experiencing signs of menopause, not chlamydial infection.

A client at 9 weeks' gestation learns about being HIV positive. Which client statement indicates teaching about the effects on the baby has been effective? 1. "The pregnancy will increase the progression of my disease and will reduce my CD4 counts." 2. "The HIV will not affect my baby, and I will have a low-risk pregnancy without additional testing." 3. "My baby will probably be born with anti-HIV antibodies, but that does not mean the baby is infected." 4. "I cannot take the medications that control HIV during my pregnancy because they will harm the baby."

Answer: 3 Explanation: 1. There is no evidence to indicate that pregnancy increases the progression of HIV/AIDS. 2. Pregnancy affected by HIV/AIDS is considered complicated, and the fetus is monitored closely. Fetal assessments include weekly non-stress tests beginning at 32 weeks. 3. Babies of HIV-positive women or women with AIDS are born with maternal anti-HIV antibodies. HIV infection in infants should be diagnosed using HIV virologic assays as soon as possible, with initiation of infant antiretroviral prophylaxis immediately if the test is positive. 4. Most of the medications that control HIV progression are safe to take during pregnancy. Antiretroviral medications are recommended during pregnancy to prevent perinatal transmission.

Which maternal-child client should the nurse see first? 1. Blood type B, Rh-positive 2. Blood type O, Rh-negative 3. Direct Coombs test positive 4. Indirect Coombs test negative

Answer: 3 Explanation: 1. This client's blood type creates no problems. 2. This client is Rh-negative, but there is no indication that the alloimmunization has occurred. 3. A direct Coombs test looks for Rh antibodies in the fetal blood circulation. A positive result indicates that that there is an Rh incompatibility between mother and infant, and the baby is making anti-Rh antibodies, which in turn leads to hemolysis. This infant is at risk for anemia and hyperbilirubinemia. 4. An indirect Coombs test looks for Rh antibodies in the maternal serum; a negative result indicates the client has not been alloimmunized.

The nurse is preparing materials for couples beginning prenatal care. What information is most important for the nurse to include? 1. The birthing unit was remodeled and redecorated last year. 2. Some of the healthcare providers recommend circumcision for baby boys. 3. There are various types of healthcare providers to support the client through the process. 4. There are different types of rooms for giving birth, each with different equipment.

Answer: 3 Explanation: 1. This information does not help clients understand their options or make decisions. 2. Because not all clients will be having boys, this statement is only helpful to those clients who give birth to males and see a healthcare provider who recommends circumcision. This statement is too narrowly focused to be helpful to all clients. 3. This statement is the most important. The nurse should inform clients what their options are, including the types of healthcare providers available. 4. This statement is too vague to facilitate decision making by the couple.

Which client should the nurse document as exhibiting signs and symptoms of primary dysmenorrhea? 1. 17-year-old, has never had a menstrual cycle 2. 19-year-old, regular menses for 5 years that have suddenly become painful 3. 14-year-old, irregular menses for 1 year, experiences cramping every cycle 4. 16-year-old, had regular menses for 4 years, but has had no menses in 4 months

Answer: 3 Explanation: 1. This is primary amenorrhea, or the lack of menses. 2. Secondary dysmenorrhea is the sudden onset of pain and discomfort with menses. 3. Primary dysmenorrhea is when menstruation has been painful from the first menstrual cycle and consistently continues to be painful each month. 4. Secondary amenorrhea is the term used when a client has had regular cycles that cease.

A pregnant client at 23 weeks' gestation has a hemoglobin of 9.5. Which diet choice indicates that teaching has been effective? 1. Broiled fish, lettuce salad, grapefruit half, carrot sticks 2. Pork chop, mashed potatoes and gravy, cauliflower, tea 3. Roast beef, steamed spinach, tomato soup, orange juice 4. Tofu with mixed vegetables in curry, milk, whole-wheat bun

Answer: 3 Explanation: 1. This meal is high in fiber, low in fat, and moderately high in protein, but low in iron. This client is anemic and needs iron. 2. This meal has a moderate amount of protein, but no vitamin C. The meal containing beef is better. 3. This client is anemic and needs iron. This meal contains iron in the beef, folic acid in the spinach, and vitamin C in the tomato soup and orange juice. Vitamin C helps absorption of the iron; folic acid is needed for production of red cells. 4. This diet is high in calcium. The client has iron-deficiency anemia and requires a high-iron diet.

Which statement is best to include when teaching a pregnant adolescent about nutritional needs? 1. "You just need to pay attention to what you eat now." 2. "Folic acid intake is the key to having a healthy baby." 3. "It is important eat iron-rich foods like meat every day." 4. "Calcium and milk aren't needed until the third trimester."

Answer: 3 Explanation: 1. This response is too vague to be helpful. Adolescents will need specific information to improve nutrition during pregnancy. 2. Although folic acid is important during pregnancy to prevent neural tube defects, and for lactation, there is not one single nutritional element responsible for having a healthy baby. 3. Adolescents often have an iron intake that is inadequate for pregnancy. Giving specific examples is most helpful when providing nutritional information. 4. Calcium is needed throughout pregnancy and should be consumed daily.

The nurse is reviewing nursing documentation related to the care of a client who had an amniocentesis. Which nursing note reflects appropriate client care? 1. Prior to discharge, the client demonstrated vaginal spotting. 2. An Rh-positive client received RhoGAM after the amniocentesis. 3. The client was monitored for 30 minutes after completion of the test. 4. The client reported that she takes insulin before each meal and at bedtime.

Answer: 3 Explanation: 1. Vaginal spotting after the amniocentesis is not an expected finding. A client experiencing vaginal bleeding of any amount after amniocentesis requires additional assessment and should not be sent home. 2. Only Rh-negative clients receive RhoGAM after amniocentesis. The Rh-positive client should not ever receive RhoGAM. 3. Twenty to 30 minutes of fetal monitoring is performed to assess for fetal well-being and to rule out injury of the fetus or placenta during the examination. 4. Whether or not a client takes insulin has nothing to do with amniocentesis. This answer does not relate to the question asked.

Which client is not a good candidate for Depo-Provera (DMPA)? 1. One with a vaginal prolapse 2. One who weighs 200 pounds 3. One who wishes to breastfeed 4. One who wishes to get pregnant within 3 months

Answer: 4 Explanation: 1. There is no correlation between a vaginal prolapse and use of Depo-Provera. 2. There is no correlation between one's weight and use of Depo-Provera. 3. Studies have proven there is no harm to a breastfed baby when a woman uses Depo-Provera. 4. Return of fertility after the use of Depo-Provera takes an average of 10 months.

A pregnant client at 14 weeks' gestation is diagnosed with hyperemesis gravidarum. The most recent vital signs are: blood pressure 95/48, pulse 114, respirations 24. Which order should the nurse implement first? 1. Weigh the client. 2. Encourage clear liquids orally. 3. Give 1 L of lactated Ringer solution IV. 4. Administer 30 mL Maalox (magnesium hydroxide) orally.

Answer: 3 Explanation: 1. Weighing the client provides information on weight gain or loss, but it is not the top priority in a client with excessive vomiting during pregnancy. The vital signs indicate hypovolemia. The client needs IV fluids. 2. The client needs IV fluids because of the vital signs indicating hypovolemia. Oral fluids are not likely to be tolerated well by a client with hyperemesis. Lack of tolerance of oral fluids through excessive vomiting is what has led to the hypovolemia. 3. The vital signs indicate hypovolemia. Giving this client a liter of lactated Ringer solution intravenously will reestablish vascular volume and bring the blood pressure up, and the pulse and respiratory rate down. 4. The vital signs indicate hypovolemia. There is no indication that the client has dyspepsia. The client needs IV fluids.

A 23-year-old client who is at 10 weeks' gestation with a first pregnancy expresses worry over feeling no sexual desire for her spouse and asks if this is normal. How should the nurse respond? 1. "That's unusual. Throughout a healthy pregnancy, sexual desire usually increases with each trimester." 2. "That's unusual. Usually, there are minimal changes in sexual desire throughout a healthy pregnancy." 3. "That sounds normal. In many cases, sexual desire decreases in the first trimester, but it increases again during the second trimester." 4. "That sounds normal. During the first trimester, sexual desire often decreases; however, by the third trimester, sexual desire is usually greater than before pregnancy."

Answer: 3 Explanation: 1. While each woman may be different, sexual desire often decreases during the first trimester, increases during the second trimester, and then decreases again during the third trimester. 2. While each woman may be different, sexual desire often decreases during the first trimester, increases during the second trimester, and then decreases again during the third trimester. 3. During the first trimester, factors such as fatigue, nausea, vomiting, and breast tenderness may decrease desire for sexual activity. During the second trimester, as these discomforts lessen and pelvic vascular congestion increases, the woman may experience greater sexual satisfaction than before pregnancy. 4. While each woman may be different, sexual desire often decreases during the first trimester, increases during the second trimester, and then decreases again during the third trimester.

The nurse is preparing a community education session on the stages of the rape trauma syndrome. In which order should the nurse explain the phases of this syndrome? 1. Buys a weapon 2. Advocates for others 3. Desires revenge and feels guilty 4. Changes email address and phone number

Answer: 3, 1, 4, 2Explanation: 1. Buying a weapon occurs during the outward adjustment phase. 2. Advocating for others occurs during the integration and recovery phase. 3. Desiring revenge and feeling guilty occurs during the acute phase. 4. Changing communication methods occurs during the reorganization phase.

The nurse is discharging a client after hospitalization for pelvic inflammatory disease (PID). Which statements indicate that teaching was effective? Select all that apply. 1. "Tubal pregnancy could occur after PID." 2. "My PID was caused by a yeast infection." 3. "It is important for me to finish my antibiotics." 4. "I might have infertility because of this infection." 5. "I am going to have an IUD placed for contraception."

Answer: 3, 4 Explanation: 1. There is no evidence to support that PID increases the risk of tubal pregnancies. 2. PID is caused by bacteria, most commonly Chlamydia trachomatis or Neisseria gonorrhoeae. Yeast infections do not ascend and become upper reproductive tract infections. 3. Antibiotic therapy should always be completed when a client is diagnosed with any infection. 4. Women sometimes become infertile because of scarring in the fallopian tubes as a result of the inflammation of PID. 5. An intrauterine device (IUD) in place increases the risk of developing PID; a client who has a history of PID is not a good candidate for an IUD.

During a group session of rape trauma survivors the nurse notes that one participant is in the reorganization phase of recovery. What did this participant demonstrate to cause the nurse to make this decision? Select all that apply. 1. Blames the assailant for the rape 2. Asks what she did to deserve the attack 3. Explains about constantly reliving the rape 4. Asks when the nightmares are going to stop 5. Talks about having one-night stands after bar hopping

Answer: 3, 4, 5Explanation: 1. Blaming the assailant is a behavior consistent with the integration and recovery phase. 2. Asking what she did to deserve the attack is a characteristic of the acute phase. 3. Constantly reliving the rape or having flashbacks is a characteristic of the reorganization phase. 4. Experiencing nightmares is a characteristic of the reorganization phase. 5. Risky sexual behavior is a characteristic of the reorganization phase.

19) A pregnant client wants natural childbirth and asks what approaches can be used to keep the mind and body relaxed during labor. Which mind-based therapies should the nurse review with this client? Select all that apply. 1. Qigong 2. Massage 3. Hypnosis 4. Visualization 5. Guided imagery

Answer: 3, 4, 5Explanation: 1. Qigong is a self-discipline that involves the use of breathing, meditation, self-massage, and movement. It is not considered a mind-based therapy. 2. Massage therapy involves manipulation of the soft tissues of the body to reduce stress and tension, increase circulation, diminish pain, and promote a sense of well-being. It is not considered a mind-based therapy. 3. Hypnosis is a state of great mental and physical relaxation during which a person is very open to suggestions. Pregnant women who receive hypnosis before childbirth have reported shorter, less painful labors and births. 4. Visualization is a complementary therapy in which a person goes into a relaxed state and focuses on, or "visualizes," soothing or positive scenes such as a beach or a mountain glade. Visualization helps reduce stress and encourage relaxation. 5. Guided imagery is a state of intense, focused concentration used to create compelling mental images. It is sometimes considered a form of hypnosis.

he nurse is reviewing the embryonic primary germ layers with a group of students. Which structures should the nurse identify as being formed from the mesoderm? Select all that apply. 1. Nails 2. Liver 3. Spleen 4. Muscles 5. Skeleton

Answer: 3, 4, 5Explanation: 1. The nails originate from the ectoderm. 2. The liver originates from the endoderm. 3. The spleen originates from the mesoderm. 4. The muscles originate from the mesoderm. 5. The skeleton originates from the mesoderm.

After completing a physical assessment the nurse determines that a laboring client is experiencing a panic attack. What findings did the nurse use to make this clinical determination? Select all that apply. 1. Flat affect 2. Monotone replies 3. Heart rate 120 bpm 4. Respiratory rate 28/minute 5. Disoriented to place and time

Answer: 3, 4Explanation: 1. A flat affect would be associated with depression. 2. Monotone replies are associated with depression. 3. A heart rate of 120 bpm indicates tachycardia, a manifestation of a panic attack. 4. A respiratory rate of 28/minute indicates hyperventilation, a manifestation of a panic attack. 5. Disorientation to place and time would be associated with schizophrenia.

The nurse is preparing an educational program on the different types of pregnancy tests. What should the nurse include about the β-subunit radioimmunoassay (RIA) test? Select all that apply. 1. It causes a color change. 2. It takes 2 to 3 hours to perform. 3. It is able to detect trophoblastic disease. 4. It is able to detect an ectopic pregnancy. 5. It is the same as an over-the-counter test.

Answer: 3, 4Explanation: 1. Enzyme-linked immunosorbent assay (ELISA) uses a substance that results in a color change after binding. 2. Fluoroimmunoassay (FIA) takes about 2 to 3 hours to perform. 3. β-Subunit radioimmunoassay (RIA) uses an antiserum with specificity for the β-subunit of hCG in blood plasma. This test may not only detect pregnancy but also detect trophoblastic disease. 4. β-Subunit radioimmunoassay (RIA) uses an antiserum with specificity for the β-subunit of hCG in blood plasma. This test may not only detect pregnancy but also detect an ectopic pregnancy. 5. Over-the-counter pregnancy tests are ELISA tests. This assay, which may be done on urine or blood, is sensitive and quick. It can detect hCG levels as early as 7 to 9 days after ovulation and conception, which is 5 days before the first missed period.

During a wellness visit a young adult client asks for information regarding weight management and exercise prior to pregnancy. What should the nurse explain to this client? Select all that apply. 1. Reduce the intake of complex carbohydrates. 2. Achieve normal weight for height before conceiving. 3. Begin an exercise program a month before conceiving. 4. Select an exercise program to be followed throughout the pregnancy. 5. Reduce weight to below normal for height to compensate for pregnancy weight gain.

Answer: 3, 4Explanation: 1. No particular food group should be avoided for adequate nutritional status. 2. Before conception, it is advisable for the woman to be at an average weight for her body build and height. 3. A woman is advised to establish a regular exercise plan beginning at least 3 months before she plans to attempt to become pregnant. 4. The exercise should be one she enjoys and will continue. 5. Weight should not be reduced to an unhealthy level before conceiving.

The nurse notes that a client is admitted for treatment of a disease caused by a single-nucleotide genetic disorder. For which health problem should the nurse plan care for this client? 1. Cancer 2. Heart disease 3. Sickle cell anemia 4. Huntington disease

Answer: 3Explanation: 1. Cancer can be caused by either a somatic mutation or a multifactorial disorder. 2. Heart disease is considered a multifactorial disorder. 3. Alterations as small as a single-nucleotide change are known to cause disease. Sickle cell disease is such a disorder. 4. Huntington disease is caused by trinucleotide repeat expansion disorder.

A pregnant client is receiving the results of perinatal testing. Which statement indicates that the client understands the test result? 1. "Because my contraction stress test was positive, we know that my baby will tolerate labor well." 2. "The reactive non-stress test means that my baby is not growing because of a lack of oxygen." 3. "My biophysical profile score of 6 points indicates everything being normal and healthy for my baby." 4. "The normal Doppler velocimetry wave result indicates my placenta is getting enough blood to the baby."

Answer: 4 Explanation: 1. A contraction stress test creates mild contractions. The presence of decelerations is termed a positive result and indicates a lack of adequate placental functioning. 2. The non-stress test utilizes external fetal monitoring to assess the fetal heart rate in relationship to fetal movement. When accelerations in the fetal heart rate are associated with fetal movement (a reactive result), the fetus is well oxygenated, and the placenta is functioning well. 3. The biophysical profile score should be 8 (with adequate amniotic fluid) or 10. A score of 6 is abnormal and indicates that further assessment is needed. 4. The Doppler velocimetry test looks at blood flow through the umbilical artery. A normal result indicates there is no vasospasm decreasing blood flow to the placenta; therefore, the baby is getting an adequate blood supply.

During a non-stress test, the nurse notes that the fetal heart rate decelerates about 15 beats during a period of fetal movement. The decelerations occur twice during the test and last 20 seconds each. What should the nurse expect the outcome of this test will be? 1. A reactive test 2. A negative test 3. An equivocal test 4. A nonreactive test

Answer: 4 Explanation: 1. A reactive stress test has the expected results of an increase in heart rate of 15 beats per minute for 15 seconds or more. 2. Non-stress tests are scored as either reactive or nonreactive. 3. Non-stress tests are scored as either reactive or nonreactive. 4. In a nonreactive stress test, the reactivity criteria are not met. Since this client experienced a deceleration during the test, this is considered nonreactive.

A client diagnosed with polycystic ovarian disease (PCOS) asks why spironolactone (Aldactone) has been prescribed. How should the nurse respond? 1. "Spironolactone is often used to reduce complications associated with PCOS, including rectocele." 2. "Menstrual irregularities related to polycystic ovarian disease are treated using spironolactone." 3. "Condylomata acuminata, which are sometimes caused by polycystic ovarian disease, are treated with spironolactone." 4. "Spironolactone may be used to decrease symptoms associated with PCOS, such as excessive hair growth and acne."

Answer: 4 Explanation: 1. A rectocele, which may develop when the posterior vaginal wall is weakened, is associated with pelvic relaxation. 2. Combined oral contraceptive (COC) or cyclic progesterone is used to treat menstrual irregularities associated with PCOS. 3. Condylomata acuminata, also called genital or venereal warts, is a sexually transmitted condition unrelated to PCOS. 4. Spironolactone may be used to treat symptoms of hyperandrogenism that are secondary to PCOS, including excessive hair growth and acne.

A nurse is examining different nursing roles. Which statement best illustrates an advanced practice nursing role? 1. A registered nurse who is the manager of a large obstetric unit 2. A clinical nurse specialist working as a staff nurse on a mother-baby unit 3. A registered nurse who is the circulating nurse at surgical deliveries (cesarean sections) 4. A clinical nurse specialist with whom other nurses consult for this nurse's expertise in caring for high-risk infants

Answer: 4 Explanation: 1. A registered nurse who is the manager of a large obstetric unit or one who is a circulating nurse at surgical deliveries (cesarean sections) is defined as a professional nurse, and has graduated from an accredited program in nursing and completed the licensure examination. 2. A clinical nurse specialist working as a staff nurse on a mother-baby unit might have the qualifications for an advanced practice nursing staff but is not working in that capacity. 3. A registered nurse who is the manager of a large obstetric unit or one who is a circulating nurse at surgical deliveries (cesarean sections) is defined as a professional nurse, and has graduated from an accredited program in nursing and completed the licensure examination. 4. A clinical nurse specialist with whom other nurses consult for expertise in caring for high-risk infants would illustrate an advanced practice nursing role. This nurse has specialized knowledge and competence in a specific clinical area, and is master's-prepared.

The school sexual health clinic nurse has female adolescent students waiting to be seen. Which student should be seen first? 1. 17-year-old adolescent with a history of child abuse 2. 14-year-old adolescent whose 17-year-old sister is pregnant 3. 15-year-old adolescent who reports using condoms regularly 4. 16-year-old adolescent who had a chlamydial infection treated 2 weeks ago

Answer: 4 Explanation: 1. Although adolescents with a history of abuse are more likely to become pregnant than are their peers who have not experienced abuse, too little information is given about this client to determine risk for pregnancy. This client is not the top priority. 2. Although this student is at risk for becoming pregnant because her sister is experiencing an adolescent pregnancy, it is not known whether this client is sexually active. This student is not the top priority. 3. Condom use will decrease the risk of becoming pregnant. This client is a low priority. 4. This client is the top priority. Having had a chlamydial infection, a sexually transmitted infection, indicates that the client is sexually active and not using a barrier method of birth control. This client is at risk for pregnancy and another STI.

The prenatal clinic nurse is designing a new prenatal intake information form for pregnant clients. Which question is the most important to include on this form? 1. What is the name of the baby's father? 2. Where was the father of the baby born? 3. Are you married to the father of the baby? 4. Do genetic diseases run in the family of the baby's father?

Answer: 4 Explanation: 1. Although it is helpful for the nurse to know the name of the father's baby to include him in the prenatal care, this is psychosocial information and much less important than possible genetic diseases that the baby might have inherited. 2. This is not important information for pregnancy. 3. Although the marital status of the client might have cultural significance, this is psychosocial information and much less important than possible genetic diseases that the baby might have inherited. 4. This question has the highest priority because it gets at the physiologic issue of inheritable genetic diseases that might directly impact the baby.

The nurse learns that a pregnant client's religion is Judaism. Why is this information important? 1. Religious and cultural background can impact what a client eats during pregnancy. 2. Knowing the client's beliefs and behaviors regarding pregnancy is important. 3. Clients sometimes encounter problems in their pregnancies based on what religion they practice. 4. It provides a baseline from which to ask questions about the client's religious and cultural background.

Answer: 4 Explanation: 1. Although this is true, much more than diet is impacted by religious and cultural background; values, beliefs, expectations for the birth, and acceptance or refusal of medical treatment are also influenced by religious or cultural background. 2. Not all people interpret or live out their religious or cultural backgrounds the same way. It is imperative to avoid stereotyping clients based on their background. The nurse must use the information on the client's background as an educated starting point on which to base further questions about how this specific client enacts her religious or cultural background. 3. How a client enacts her religion occasionally will cause problems with pregnancy. But the most important reason for asking a client for her religious or cultural background is to have a starting point on which to base further questions on the specifics of how this client is impacted by or enacts her cultural or religious background as a unique individual. 4. This is the best explanation because not all people interpret or live out their religious or cultural backgrounds the same way. It is imperative to avoid stereotyping clients. Thus, the nurse should use the information on the client's background as an educated starting point on which to base further questions about how this specific client enacts her religious or cultural background.

An adolescent client reports that her period is late but that her home pregnancy test is negative. Which response is most appropriate? 1. "This means you are not pregnant." 2. "We do not trust home tests. Come to the clinic for a blood test." 3. "Most people do not use the tests correctly. Did you read the instructions?" 4. "You might be pregnant, but it might be too early for your home test to be accurate."

Answer: 4 Explanation: 1. Although this might be true, this is not the best response because the pregnancy may not yet be detectable through use of a urine pregnancy test. 2. This statement is not therapeutically worded. Additionally, this statement is not true because home pregnancy tests are quite simple to use and quite accurate. A clinic pregnancy test is usually a urine test. Blood tests are more invasive and more expensive. 3. This response does not address the issues presented in the client's statement. 4. This is an accurate and appropriate response. Most home pregnancy tests have low false- positive rates, but the false-negative rate is slightly higher. Repeating the test in 1 week is recommended.

The nurse is caring for a client at 35 weeks' gestation who has been critically injured in a shooting. Which statement by the paramedics bringing the woman to the hospital should cause the greatest concern? 1. "Blood pressure 110/68, pulse 90." 2. "Clear fluid is leaking from the vagina." 3. "Client is positioned in a left lateral tilt." 4. "Entrance wound present below the umbilicus."

Answer: 4 Explanation: 1. These are normal vital signs, indicating a hemodynamically stable client. 2. Clear fluid from the vagina could be amniotic fluid from spontaneous rupture of the membranes. Although this is not a normal finding at 35 weeks, this fetus is near term and would likely survive birth at this time. 3. Positioning the client in a lateral tilt position prevents vena cava syndrome. 4. Penetrating abdominal trauma has a 59% to 80% fetal injury rate. This fetus is at great risk for injury.

The nurse is working with male teens whose partners are pregnant. Which situation with a father-to-be requires nursing intervention? 1. The pregnancy does not seem real to him, and he is not sure what he should do to plan for the future. 2. Because his father was not involved in his life, he wants to be actively involved in the life of his child. 3. He is not convinced that he is the father of the baby and does not want his name on the birth certificate. 4. He is the only other person who will be present, although his girlfriend wants her mother to be with her during the birth.

Answer: 4 Explanation: 1. An early pregnancy is often an abstract concept to fathers. It is normal for a teen to be unsure about the future. 2. This might be the case when a young father was raised without his own father, or a lack of involvement with the child might be seen as the norm and desired. 3. Many young fathers are not sure if they are the father of the baby and do not want to be listed on the birth certificate if they are not sure. Often these young men will request paternity testing to either verify or discount their parentage. 4. It is common for pregnant adolescents to want their mothers to accompany them for the labor and birth. Overriding his girlfriend's expressed desire could be an indication that their relationship is abusive.

2) The nurse is performing a family assessment. What type of family should the nurse identify when both parents work? 1. An extended family 2. An extended kin family 3. A traditional nuclear family 4. A dual-career/dual-earner family

Answer: 4 Explanation: 1. An extended family is defined as couples who share household and childrearing responsibilities with parents, siblings, or other relatives. 2. An extended kin family is a specific form of an extended family. 3. The traditional nuclear family is defined as a husband/provider, a wife who stays home, and children. 4. A dual-career/dual-earner family is characterized by both parents working, by either choice or necessity.

10) The nurse tells family members the sex of a newborn baby without first consulting the parents. Which act did this nurse commit? 1. Negligence 2. Malpractice 3. A breach of ethics 4. Breach of privacy

Answer: 4 Explanation: 1. Any nurse who fails to meet appropriate standards of care invites allegations of negligence. 2. Any nurse who fails to meet appropriate standards of care invites allegations of malpractice. 3. A breach of ethics would not apply to this situation. 4. A breach of privacy would have been committed in this situation, because it violates the right to privacy of this family. The right to privacy is the right of a person to keep his or her person and property free from public scrutiny (of even other family members).

5) The home health nurse visits the home of a Korean couple to follow up on their jaundiced 4-day-old baby. Considering family structure, what family members might the nurse expect to see in the home? 1. The godparents 2. Just the parents 3. The grandmother 4. The grandfather and parents

Answer: 4 Explanation: 1. Asians traditionally revere their elders and their wisdom. The godparents would not have the last word in decision making for this family. 2. Asians traditionally revere their elders and their wisdom. The parents would not have the last word in decision making for this family. 3. Asians traditionally revere their elders and their wisdom. The grandmother would not have the last word in decision making for this family. 4. The grandfather is the family member who plays a key role in decision making and who is likely to be present in this situation. Asians traditionally revere their elders and their wisdom.

The spouse accompanies a pregnant client to a prenatal visit. Which question should the nurse use to determine the amount of anticipatory guidance the spouse will need? 1. "What kind of work do you do?" 2. "How moody has your wife been lately?" 3. "What furniture have you gotten for the baby?" 4. "How are you feeling about becoming a father?"

Answer: 4 Explanation: 1. Asking about vocation does not help determine the amount of anticipatory guidance the spouse will need. 2. Asking about the client's mood does not help determine the amount of anticipatory guidance the spouse will need. 3. Buying furniture does not help determine the amount of anticipatory guidance the spouse will need. 4. Anticipatory guidance of the expectant father is a necessary part of any plan of care. He may need information on a variety of topics about the pregnancy, and the best question to learn the spouse's needs is to ask about his feelings about becoming a father.

The nurse explains to a pregnant woman that her antepartum assessment will include assessment of clinical pelvimetry. Which client response reflects understanding of the reason for this test? 1. "It will help me understand how big a baby I can have." 2. "It will be used to screen for gestational diabetes." 3. "It will be used to find out whether my baby has a chromosomal abnormality." 4. "It will help tell whether my pelvis is big enough to deliver my baby vaginally."

Answer: 4 Explanation: 1. Clinical pelvimetry is performed to estimate the adequacy of pelvic size for the purpose of vaginal delivery; delivery of larger infants may be accommodated via cesarean section. 2. Screening for maternal gestational diabetes requires some form of glucose screening. 3. Clinical pelvimetry involves estimating the adequacy of pelvic size for facilitating vaginal birth. 4. Clinical pelvimetry is performed to estimate the ease or difficulty associated with vaginal delivery of an infant.

The nurse is caring for a client diagnosed with endometriosis. Which statement by the client requires immediate follow-up? 1. "I've noticed my voice is lower since I started taking danazol." 2. "I am having many hot flashes since I had the Lupron injection." 3. "The pain I experience with intercourse is becoming more severe." 4. "My leg has become painful and swollen since I started taking birth control pills."

Answer: 4 Explanation: 1. Danocrine (danazol) is a testosterone derivative that suppresses gonadotrophin-releasing hormone (GnRH) and has high-androgen and low-estrogen effects. A lowered voice is one side effect of danazol. This client is not experiencing a complication. 2. Leuprolide acetate (Lupron) is a GnRH agonist and causes symptoms of a hypoestrogenic state (hot flashes, vaginal dryness, decreased libido, and bone density loss). Hot flashes are expected and not a complication. 3. Dyspareunia is a common symptom of endometriosis and therefore is not a complication. 4. Combination oral contraceptive pills contain estrogen. A painful, swollen lower extremity can be a sign of deep vein thrombosis, which can cause thromboembolus, which is potentially life threatening. This is a complication and must be addressed immediately.

A pregnant 14-year-old client confides having problems with bulimia nervosa. Which nursing observation best supports the client's statement? 1. The client reports dietary cravings for soil and clay. 2. In terms of food variety and quantity, the client's diet is extremely restrictive. 3. Despite being extremely underweight, the client describes herself as being fat. 4. The client is of normal weight for her height and reports binge eating followed by purging.

Answer: 4 Explanation: 1. Dietary cravings for and consumption of nonnutritive substances is consistent with pica. 2. The dietary intake of individuals with anorexia nervosa is very restrictive in both variety and quantity. 3. Anorexia nervosa is an eating disorder characterized by an extreme fear of weight gain and becoming fat, and it incorporates a self-perception of being overweight even when the individual is extremely underweight. 4. Bulimia is characterized by binge eating and purging, and individuals with bulimia nervosa often maintain normal or near-normal weight for their height.

A pregnant client asks about the differences between monozygotic and dizygotic twins. Which should the nurse include during this teaching? 1. Dizygotic twins share one placenta and one chorion. 2. Monozygotic twins are also referred to as "fraternal" twins. 3. Dizygotic twinning occurs less frequently than does monozygotic twinning. 4. Monozygotic twins originate from division of the fertilized ovum at different stages.

Answer: 4 Explanation: 1. Dizygotic twins each have a separate chorion and amnion. 2. Monozygotic twins are also referred to as "identical" twins. 3. Dizygotic twinning occurs more frequently than does monozygotic twinning. 4. Monozygotic twins originate from division of the fertilized ovum at different stages of early development.

A client desiring to become pregnant is concerned because the opening of her spouse's penis is below the tip. How should the nurse respond to this client? 1. "This is epispadias and it will cause infertility." 2. "This is hypospadias and it will cause infertility." 3. "This is epispadias and is not likely to impact fertility." 4. "This is hypospadias and is not likely to impact fertility."

Answer: 4 Explanation: 1. Epispadias is the condition where the urethral opening is on the upper aspect of the penis. The client is describing hypospadias, when the urethral opening is on the lower side of the penis. Mild hypospadias, when the urethral opening is on the glans of the penis, does not impact fertility. 2. The client is describing hypospadias, where the urethral opening is on the lower side of the penis. Mild hypospadias, where the urethral opening is on the glans of the penis, does not impact fertility. 3. Epispadias is the condition where the urethral opening is on the upper aspect of the penis. The client is describing hypospadias, when the urethral opening is on the lower side of the penis. Mild hypospadias, when the urethral opening is on the glans of the penis, does not impact fertility. 4. The client is describing hypospadias, which is the urethral opening on the lower aspect of the penis. Mild hypospadias, when the urethral opening is on the glans of the penis, does not impact fertility.

The nurse is teaching a client who recently learned that she is pregnant. Which hormone should the nurse instruct as being secreted by the fertilized egg? 1. Estrogen 2. Progesterone 3. Luteinizing hormone 4. Human chorionic gonadotropin (hCG)

Answer: 4 Explanation: 1. Estrogen is an ovarian hormone. 2. Progesterone is an ovarian hormone. 3. Luteinizing hormone is excreted by the anterior pituitary. 4. When the ovum is fertilized and implants in the endometrium, the fertilized egg begins to secrete hCG hormone to maintain the corpus luteum.

7) A woman of Korean descent has just given birth to a son. Her partner wishes to give her sips of hot broth from a thermos they brought with them. They have refused your offer of ice chips or other cold drinks for the client. What should the nurse do? 1. Explain to the client that she can have the broth if she will also drink cold water or juice. 2. Encourage the client to have the broth, after the nurse takes it to the kitchen and boils it first. 3. Explain to the couple that food is not allowed to be brought from home, but that the nurse will make hot broth for them. 4. Encourage the partner to feed the client sips of their broth. Ask if the client would like you to bring her some warm water to drink as well.

Answer: 4 Explanation: 1. Explaining to the client that she can have broth if she will drink cold water or juice first does not show cultural sensitivity and does not respect the client's beliefs. 2. Encouraging the client to have broth after you take it to the kitchen and boil it first does not demonstrate cultural sensitivity. 3. Explaining to the couple that food is not allowed to be brought from home but that you will make hot broth for them does not demonstrate cultural sensitivity. 4. Encouraging the partner to feed the client sips of their broth and asking if the client would like you to bring her some warm water to drink are approaches that show cultural sensitivity. The equilibrium model of health, based on the concept of balance between light and dark, heat and cold, is the foundation for this belief and practice.

7) What is the maternity nurse's best defense against an accusation of malpractice or negligence? 1. Follows the physician's written orders 2. Acts on the advice of the nurse manager 3. Becomes certified as a nurse-midwife or nurse practitioner 4. Meets the Association of Women's Health, Obstetric and Neonatal Nurses (AWHONN) standards of practice

Answer: 4 Explanation: 1. Following the physician's written orders is not enough to defend the nurse from accusations because the orders and/or advice might be wrong or unethical. 2. Acting on the advice of the nurse manager is not enough to defend the nurse from accusations because the orders and/or advice might be wrong or unethical. 3. Being a certified nurse-midwife or nurse practitioner does not defend the nurse against these accusations if she does not follow the AWHONN standards of practice. 4. Meeting the AWHONN standards of practice would cover the maternity nurse against an accusation of malpractice or negligence because the standards are rigorous and cover all bases of excellent nursing practice.

A pregnant client who uses cocaine and ecstasy on a regular basis asks why ecstasy should not be used during pregnancy. What should the nurse explain about this drug? 1. "It produces intrauterine growth restriction and meconium aspiration." 2. "It leads to deficiencies of thiamine and folic acid, which help the baby develop." 3. "It produces babies with small heads and short bodies with brain function alterations." 4. "It can cause a high fever in you if high doses are taken and therefore cause the baby harm

Answer: 4 Explanation: 1. Heroin causes these fetal effects, not ecstasy. 2. Alcohol, not ecstasy, causes deficiencies of thiamine and folic acid. Folic acid helps prevent neural tube defects. 3. Cocaine causes these fetal effects, not ecstasy. 4. High body temperature is a side effect of MDMA (methylenedioxymethamphetamine: ecstasy). Increased body temperature increases fetal oxygen needs, which can lead to hypoxia and subsequent brain and major organ damage.

A client who is at 18 weeks' gestation has been newly diagnosed with megaloblastic anemia. Which client statement indicates teaching has been effective? 1. "My body makes red blood cells that are smaller than they should be." 2. "Megaloblastic anemia is not known to cause any serious risks to my baby." 3. "Whenever possible, I should boil my vegetables in at least 2 quarts of water." 4. "I should include fresh leafy green vegetables, red meat, fish, poultry, and legumes in my diet."

Answer: 4 Explanation: 1. In megaloblastic anemia, red blood cells become enlarged and are fewer in number. 2. Maternal folic acid deficiency has been associated with an increased risk of neural tube defects (NTDs) such as spina bifida, meningomyelocele, and anencephaly in the newborn. 3. Folic acid, which is crucial for inclusion in the diet of clients with megaloblastic anemia, is easily destroyed by overcooking or cooking with large quantities of water. 4. Folic acid, which is used to treat megaloblastic anemia, is readily available in foods such as fresh leafy green vegetables, red meat, fish, poultry, and legumes.

A 15-year-old primipara at 8 weeks' gestation who is 64 inches tall and weighs 115 pounds asks why she is supposed to gain so much weight. What is the best response by the nurse? 1. "Inadequate weight gain delays lactation after delivery." 2. "It's what your certified nurse-midwife recommended for you." 3. "Weight gain is important to assure that you get enough vitamins." 4. "Gaining 25 to 35 pounds is recommended for healthy fetal growth."

Answer: 4 Explanation: 1. Inadequate weight gain can lead to decreased fetal growth and development. 2. Although this statement might be true, the client has asked a "why" question that should be directly answered. 3. Vitamin intake is related to the types of food consumed, not to caloric intake. Because this client is 15, her diet is probably not optimal, and her intake of empty calories or junk food might make up the majority of her caloric intake. 4. Adolescents who become pregnant less than 4 years after menarche are at risk because of their physiologic and anatomic immaturity. They are more likely than older adolescents to still be growing, which can affect the fetus's development.

The nurse is doing preconception counseling with a 28-year-old woman with no prior pregnancies. Which client statement indicates that teaching has been effective? 1. "A beer once a week will not damage the fetus." 2. "I can continue to drink alcohol until I am diagnosed as being pregnant." 3. "I can drink alcohol while breastfeeding since it does not pass into breast milk." 4. "I need to stop drinking alcohol completely when I start trying to get pregnant."

Answer: 4 Explanation: 1. It is not known how much alcohol will cause fetal damage; therefore, alcohol during pregnancy is contraindicated. 2. Women should discontinue drinking alcohol when they start to attempt pregnancy. 3. Breastfeeding generally is not contraindicated, although alcohol is excreted in breast milk. Excessive alcohol consumption may intoxicate the infant and inhibit the maternal letdown reflex. 4. Because birth defects that are related to fetal alcohol exposure can occur in the first 3 to 8 weeks' gestation, often before the woman even knows she is pregnant, women should discontinue drinking alcohol when they start to attempt pregnancy.

The nurse is caring for a pregnant teen. What should the nurse do to accurately assess the teen's nutritional intake? 1. Assess laboratory values. 2. Ask about cooking facilities. 3. Observe for clinical signs of malnutrition. 4. Ask to complete a dietary recall to identify eating patterns.

Answer: 4 Explanation: 1. Laboratory values only provide information about the nutritional status of the client. 2. Cooking facilities might not be related to food intake. 3. Clinical signs of malnutrition only provide information about the nutritional status of the client. 4. In assessing the diet of the pregnant adolescent, it is important to consider the eating pattern over time, not simply a single day's intake. Once the pattern is identified, counseling can be directed toward correcting deficiencies.

A client comes to the clinic complaining of severe menstrual cramps. She has never been pregnant, has been diagnosed with ovarian cysts, and has had an intrauterine device (IUD) for 2 years. What is the most likely reason for this client's complaint? 1. Menorrhagia 2. Hypermenorrhea 3. Primary dysmenorrhea 4. Secondary dysmenorrhea

Answer: 4 Explanation: 1. Menorrhagia is excessive, profuse flow. 2. Hypermenorrhea is an abnormally long menstrual flow. 3. Primary dysmenorrhea is defined as cramps without underlying disease. 4. Secondary dysmenorrhea is associated with pathology of the reproductive tract and usually appears after menstruation has been established. Conditions that most frequently cause secondary dysmenorrhea include ovarian cysts and the presence of an intrauterine device.

Being aware that several pregnant students have been diagnosed with iron-deficiency anemia, the school nurse plans a class about nutrition for pregnant teens. What should the nurse encourage the teens to consume to increase iron absorption? 1. Milk 2. Green tea 3. Gatorade 4. Orange juice

Answer: 4 Explanation: 1. Milk does not contain vitamin C. 2. Green tea does not contain vitamin C. 3. Gatorade does not contain vitamin C. 4. Vitamin C is found in citrus fruits and juices and is known to enhance the absorption of iron from meat and nonmeat sources.

A 28-year-old client who is pregnant with her first child reports increased dental caries (cavities) since becoming pregnant. How should the nurse explain the likely cause for this change? 1. "Each woman experiences changes that affect her teeth while she's pregnant." 2. "When a woman is pregnant, her teeth lose calcium and she is more susceptible to getting cavities." 3. "During pregnancy, tooth enamel softens and the woman is more susceptible to getting cavities." 4. "It may be necessary to pay extra attention to dental care while you're pregnant, especially if you're vomiting frequently."

Answer: 4 Explanation: 1. No demonstrable changes occur in the teeth of pregnant women. 2. Calcium is not lost from the teeth during pregnancy. 3. Tooth enamel does not soften during pregnancy. 4. The dental caries that sometimes accompany pregnancy are probably caused by inadequate oral hygiene and dental care, especially if the woman has problems with bleeding gums or nausea and vomiting.

Upon arriving at the prenatal clinic in the morning, the nurse receives messages from four clients. Which client complaint should be addressed first? 1. Primipara at 24 weeks' gestation with ankle edema 2. Multipara at 35 weeks' gestation with new onset of hemorrhoids 3. Primipara at 9 weeks' gestation with increased fatigue and nocturia 4. Multipara at 30 weeks' gestation with vaginal bleeding after performing yoga

Answer: 4 Explanation: 1. Particularly during the second and third trimesters, ankle edema is a common pregnancy-related discomfort. 2. Hemorrhoids are a common pregnancy-related discomfort, especially during the second and third trimesters. 3. Fatigue and increased urination at night is a common pregnancy-related discomfort during the first trimester. 4. Vaginal bleeding after yoga is a warning sign that should be immediately reported to the healthcare provider; this client is the highest priority for care.

A pregnant client who is a lacto-ovo vegetarian asks for help planning a diet that includes adequate protein intake. What instruction should the nurse give? 1. "To improve protein absorption, avoid simultaneous intake of animal protein and plant protein." 2. "Following a lacto-ovo vegetarian diet will require you to take a daily supplement of vitamin B12." 3. "Because you don't eat meat, eggs, or dairy products, it's important to eat adequate plant-based proteins." 4. "In addition to eggs and dairy products, beans, peanut butter, and soy milk can be effective sources of plant-based proteins."

Answer: 4 Explanation: 1. Plant protein quality can be improved if it is consumed with certain animal proteins. 2. Vegan diets, in which no animal products are consumed, often require daily supplementation of vitamin B12. 3. While lacto-ovo vegetarians do not eat meat, they do eat eggs, milk, and dairy products. 4. A diet that includes plant proteins, such as beans and rice, peanut butter on whole-grain bread, and whole-grain cereal with soy milk, helps ensure the expectant mother obtains all the essential amino acids.

The nurse is working with a group of pregnant teens. Which statement indicates that teaching has been successful? 1. "My baby could come late because I am a teenager." 2. "Because I am young, I have a low risk for preeclampsia." 3. "I am more likely to use birth control after I have this baby." 4. "Pregnant teens are more likely to quit school prior to graduation."

Answer: 4 Explanation: 1. Postdates are not a risk for adolescent pregnancy; preterm birth and small-for-gestational-age infants are risks for pregnant teens. 2. Preeclampsia is more likely to occur in teen mothers than in mothers older than age 19. 3. Clients who give birth the first time as adolescents are more likely to have their next child during adolescence as well. Birth control use is not higher among teen parents. 4. Clients who give birth the first time as adolescents are more likely to have lower educational levels, including a higher rate of dropping out of high school and not attending college or vocational training.

Which statement, if made by a pregnant adolescent, indicates that she understands her increased risk of physiologic complications during pregnancy? 1. "Smoking and using crack cocaine won't harm my baby." 2. "My anemia and eating mostly fast food are not important." 3. "It's no big deal that I started prenatal care in my seventh month." 4. "I need to take good care of myself so my baby doesn't come early."

Answer: 4 Explanation: 1. Pregnant adolescents are at great risk for complications such as pregnancy-induced hypertension, anemia, preterm birth, low-birth-weight infants, fetal harm from cigarette smoking, alcohol consumption, or the use of street drugs. 2. Pregnant adolescents are at great risk for complications such as anemia. 3. Early and regular prenatal care is the best intervention to prevent complications or to detect them early, to minimize the harm to both the teen and her fetus. 4. Early and regular prenatal care is the best intervention to prevent complications or to detect them early, to minimize the harm to both the teen and her fetus.

A pregnant couple would like their 5-year-old to attend the birth. Which should the nurse say in response to this couple's plan? 1. "Bring some toys to keep your child occupied." 2. "Children under 12 are not allowed to be present at the birth." 3. "You should let your child stay home because you will be focusing on the birth." 4. "You should bring someone who will only tend to any specific needs of your child."

Answer: 4 Explanation: 1. Preparing the child on what to expect is beneficial. Toys will not sustain a 5-year-old's attention for an extended time period. 2. Children are allowed to be present at births. 3. A sibling should have his own support person whose primary responsibility is to take care of the child's needs so that the child will have support if anxiety develops over the birth process, and the mother can concentrate on the labor and birth. 4. A sibling should have his own support person whose primary responsibility is to take care of the child's needs so that the child will have support if anxiety develops over the birth process, and the mother can concentrate on the labor and birth.

The nurse instructs a client on the importance of reducing exposure to infections while pregnant. Which client statement indicates that teaching has been effective? 1. "My genital herpes infection will have no effect on my baby." 2. "Because I have toxoplasmosis, my baby might be born with an abnormally long body." 3. "The rubella infection I experienced in my second trimester may lead me to become deaf." 4. "My baby may develop a serious blood infection because I have group B strep in my vagina."

Answer: 4 Explanation: 1. Primary herpes simplex infection poses the greatest risk to both the mother and her infant. Primary infection has been associated with spontaneous abortion, low birth weight, and preterm birth. Transmission to the fetus almost always occurs after the membranes rupture and the virus ascends or during birth through an infected birth canal. 2. Toxoplasmosis during pregnancy can cause fetal microcephaly, hydrocephalus, coma, convulsions, or retinochoroiditis. 3. Rubella infection during pregnancy can lead to fetal deafness, congenital heart defects, and developmental delays in the fetus. Maternal deafness is not a risk for perinatal rubella. 4. Group B streptococcus can cause neonatal septicemia or pneumonia unless IV antibiotics are given during labor.

A postpartum client with blood type A, Rh-negative delivered a newborn with blood type AB, Rh-positive. Which statement indicates that teaching about this blood type inconsistency has been effective? 1. "Because my baby is Rh-positive, I do not need RhoGAM." 2. "Before my next pregnancy, I will need to have a RhoGAM shot." 3. "If my baby had the same blood type I do, it might cause complications." 4. "I need to get RhoGAM so I do not have problems with my next pregnancy."

Answer: 4 Explanation: 1. Rh-negative mothers who give birth to Rh-positive infants should receive Rh immune globulin (RhoGAM) to prevent alloimmunization. 2. Rh-negative mothers who give birth to Rh-positive infants should receive Rh immune globulin (RhoGAM). The injection must be given within 72 hours after delivery to prevent alloimmunization. 3. It is specifically the Rh factor that causes complications; ABO grouping does not cause alloimmunization. 4. Rh-negative mothers who give birth to Rh-positive infants should receive Rh immune globulin (RhoGAM) to prevent alloimmunization, which could cause fetal anemia and other complications during the next pregnancy.

The nurse is teaching a group of female teenagers about prevention of sexually transmitted infection (STI). Which student's statement suggests the need for additional teaching? a. "I can get venereal warts by having anal sex." b. "The same STI that causes genital warts can cause cancer of the cervix." c. "If I test positive for the human papillomavirus (HPV), my sexual partner and I both have to be treated with medication." d. "Human papillomavirus (HPV) can be spread from one person to another during oral sex."

Answer: c. "If I test positive for the human papillomavirus (HPV), my sexual partner and I both have to be treated with medication." Feedback: Sex partners do not require treatment unless large lesions are present.

A client learns that Huntington disease has occurred in some family members. Which type of genetic anomaly should the nurse explain as causing this disorder? 1. Somatic mutation 2. Germline mutation 3. Single gene mutation 4. Trinucleotide repeat expansion

Answer: 4 Explanation: 1. Somatic mutations are DNA alterations that occur in an individual at any time throughout a lifetime after fertilization. They result from errors during cell division (mitosis) or from environmental influences such as radiation, toxins, or viral infections and are not passed from one generation to another. 2. Germline mutations exist in the reproductive cells, causing the DNA in every cell of any offspring to have the mutation, which can then be transmitted to following generations. 3. Single gene mutations are known to cause diseases such as cystic fibrosis. 4. Trinucleotide repeat expansion occurs at sites within a gene where the DNA sequence consists of adjacent three-nucleotide repeats. These repeat sequences tend to expand during meiosis, a feature known as anticipation, resulting in a larger number of repeats in subsequent generations. A larger number of repeats may be associated with disease; typically, the larger the number of repeats, the more severe the condition. Huntington disease is caused by this type of genetic anomaly

A client in labor is a Lebanese immigrant, and she explains that in the Muslim faith, the baby's name is selected after delivery, as it is God's will whether or not the baby will be born. Which nursing response is appropriate? 1. "Are you afraid your baby will not live?" 2. "We have a very low rate of complications at this facility." 3. "In the United States, you can feel free to choose your baby's name prior to the delivery." 4. "Thank you for explaining that to me. By sharing your cultural beliefs with me, you are helping me to provide you with the best possible care."

Answer: 4 Explanation: 1. The client is describing the application of a culturally based belief, not a fear-based concern. 2. The client is describing the application of a culturally based belief, not a concern about complications. 3. It is neither realistic nor appropriate to assume that people of another culture will automatically abandon their ways and adopt the practices of a dominant culture. 4. The identification of cultural values is useful in planning and providing culturally sensitive care.

After teaching a class about the female reproductive system, the nurse asks the attendees to describe the process of meiosis. Which student response suggests successful comprehension of the material? 1. "At the time of ovulation, the first meiotic division begins." 2. "At the time of puberty, the second meiotic division begins." 3. "Completion of the first meiotic division produces three polar bodies and one primary oocyte." 4. "Completion of the second meiotic division results in formation of three polar bodies and one ovum."

Answer: 4 Explanation: 1. The first meiotic division of oocytes begins before the female fetus is born. 2. The second meiotic division begins at the time of ovulation. 3. Completion of the first meiotic division produces one polar body and one secondary oocyte. 4. Completion of the second meiotic division produces three polar bodies and one ovum.

Which statement best indicates that the client understands the differences in the follicular and luteal phases of the ovarian cycle? 1. "My period will be every 28 days." 2. "The follicular phase is when the egg is fertilized." 3. "The follicular phase is the second half of my cycle." 4. "The first part of my period might vary in length, but not the second."

Answer: 4 Explanation: 1. The follicular phase can vary, resulting in cycle length other than 28 days. 2. The follicular phase comprises days 1 to 14 of the menstrual cycle, not when the egg is fertilized. 3. The luteal phase is the second half of the cycle. 4. For a female with a 28-day cycle, the follicular phase comprises days 1 to 14 of the menstrual cycle, and the luteal phase comprises days 15 to 28. The luteal phase does not vary.

In preparation for teaching a women's community center class about physiologic changes during menopause, the nurse is preparing a handout for students. Which information should the nurse include in this teaching? 1. Due to changes in estrogen levels, the labia minora increase in size after menopause. 2. The ovaries remain small after puberty, but they increase in size following menopause. 3. After menopause, the endometrium continues to undergo monthly degeneration and renewal. 4. Ovarian secretion of estrogen decreases between the ages of 45 and 55, after which point ovulatory activity ceases.

Answer: 4 Explanation: 1. The labia minora decrease in size after menopause because of changes in estrogen levels. 2. The ovaries of girls are small, but they become larger after puberty and then decrease in size following menopause. 3. From menarche to menopause, the endometrium undergoes monthly degeneration and renewal in the absence of pregnancy. 4. Between the ages of 45 and 55, a woman's ovaries secrete decreasing amounts of estrogen. Eventually, ovulatory activity ceases and menopause occurs.

Which statement by a parent of a pregnant, unmarried 15-year-old is expected? 1. "I'm not going to get involved. She understands how her health insurance works." 2. "An abortion is the best choice for her. She can deal with our Catholic priest later." 3. "We're very happy for her. It will be easier to focus on education with a new baby." 4. "Her father told her to stop dating that boy. Now look at the trouble she's gotten into."

Answer: 4 Explanation: 1. The parent of a pregnant teen is usually the support person and helps the teen understand how to access prenatal care. It is unlikely that a 15-year-old would understand health insurance. 2. This statement indicates that the parent is not going to discuss the pregnancy with the teen but might be forcing the teen into abortion. Because religious tradition impacts views on abortion, and Catholicism disapproves of abortion, the teen might not accept abortion as an option. 3. Most parents accept the pregnancy but are not excited when their 15-year-old is pregnant. And education is harder when child care is involved. Teens that give birth are less likely to complete their education. 4. This statement indicates anger and dismay, which are expected when a parent finds out about a teen daughter's pregnancy.

A client with preeclampsia is assessed with the following: blood pressure 158/100; urinary output 50 mL/hour; lungs clear to auscultation; urine protein 1+ on dipstick; and edema of the hands, ankles, and feet. Which new assessment finding indicates the client's condition is getting worse? 1. Reflexes 2+ 2. Platelet count 150,000 3. Blood pressure 158/104 4. Urinary output 20 mL/hour

Answer: 4 Explanation: 1. The reflexes are normal at 2+. 2. The platelet count is normal, though it is at the lower end. 3. The blood pressure has not had a significant rise. 4. The decrease in urine output is an indication of decrease in glomerular filtration, which indicates a loss of renal perfusion. The assessment finding most abnormal and life threatening is the urine output change.

A multigravida gave birth to an 18-week fetus last week. During her follow-up she sees that it is documented in her medical record that she had one abortion and becomes upset over the use of this word. How can the nurse best explain this terminology to the client? 1. "Abortion is what we call all babies who are stillborn." 2. "Abortion is the word we use when someone has miscarried." 3. "Abortion is how we label pregnancies that end in the second trimester." 4. "Abortion is the medical term for all pregnancies that end before 28 weeks."

Answer: 4 Explanation: 1. Third-trimester losses are considered fetal death in utero, and the term abortion is not used. 2. Abortions are fetal losses prior to the onset of the third trimester and include elective induced (medical or surgical) abortions, ectopic pregnancies, and spontaneous abortions or miscarriages. 3. Abortions are fetal losses prior to the onset of the third trimester and include elective induced (medical or surgical) abortions, ectopic pregnancies, and spontaneous abortions or miscarriages. 4. Abortions are fetal losses prior to the onset of the third trimester and include elective induced (medical or surgical) abortions, ectopic pregnancies, and spontaneous abortions or miscarriages.

A pregnant client who swims 3 to 5 times per week asks the nurse if she should stop this activity. What is the appropriate nursing response? 1. "You should discontinue your exercise program immediately." 2. "You should increase the number of times you swim per week." 3. "You should decrease the number of times you swim per week." 4. "You should continue your exercise program because it would be beneficial."

Answer: 4 Explanation: 1. Thirty minutes of moderate-intensity exercise daily is recommended for pregnant women, but even mild exercise is helpful. There is no reason for the exercise to be discontinued. 2. Thirty minutes of moderate-intensity exercise daily is recommended for pregnant women. There is no reason for the exercise to be increased. 3. Thirty minutes of moderate-intensity exercise daily is recommended for pregnant women. There is no reason for the exercise to be decreased. 4. Thirty minutes of moderate-intensity exercise daily is recommended for pregnant women, but even mild exercise is helpful. Women who exercise regularly have better muscle tone, self-image, bowel function, energy levels, sleep, and postpartum recovery than do those who are sedentary.

The nurse is seeing prenatal clients in the clinic. Which client is exhibiting expected findings? 1. Primipara at 26 weeks with fundal height of 30 cm 2. Multipara at 12 weeks who reports bright red vaginal bleeding 3. Multipara at 22 weeks who reports no fetal movement felt yet 4. Primipara at 12 weeks with fetal heart tones heard by Doppler fetoscope

Answer: 4 Explanation: 1. This is an abnormal finding. Beginning in the second trimester, the fundal height should correlate with weeks of gestation; thus, at 26 weeks' gestation, the fundal height should be about 26 cm. 2. This is an abnormal finding. Bright red bleeding during pregnancy is never expected. 3. This is an abnormal finding. Fetal movement should be felt by 20 weeks. 4. This is an expected finding because fetal heart tones should be heard by 12 weeks using an ultrasonic Doppler fetoscope.

9) The charge nurse is reviewing the healthcare plans written by the unit's staff nurses. Which NANDA nursing diagnosis is most likely to be construed as culturally biased and possibly offensive? 1. Fear related to separation from support system during hospitalization 2. Spiritual Distress related to discrepancy between beliefs and prescribed treatment 3. Interrupted Family Processes related to a shift in family roles secondary to demands of illness 4. Noncompliance related to impaired verbal communication secondary to recent immigration from non-English-speaking area

Answer: 4 Explanation: 1. This option seeks to explain how the culturally sensitive nurse can partner with the families more effectively. 2. This option seeks to explain how the culturally sensitive nurse can partner with the families more effectively. 3. This option seeks to explain how the culturally sensitive nurse can partner with the families more effectively. 4. The phrase "impaired verbal communication" might be offensive because speaking a different language is not equivalent to being impaired, and noncompliance does not stem from misunderstanding.

A client who has been unable to conceive asks the nurse if it is her fault or her husband's fault that they have not been able to become pregnant. What is the nurse's best response? 1. "The male infertility factors are more common than female." 2. "Female infertility issues are more common than male issues." 3. "The testing the doctor will order will determine who is at fault." 4. "We will know more about what is causing your infertility after some tests are done."

Answer: 4 Explanation: 1. This statement is not true. Because of the complexity of ovulation and maintaining a pregnancy, it is more likely that a female issue is causing the infertility. Regardless, using the term "at fault" is blaming and should be avoided. 2. Although this statement is true because of the complexity of ovulation and maintaining a pregnancy, using the term "at fault" is blaming and should be avoided. 3. Testing will determine what the infertility issue is, but using the term "at fault" is blaming and should be avoided. 4. This is a factual answer that avoids using the term "at fault." This statement is therapeutically worded and therefore is the best answer.

The client with insulin-dependent type 2 diabetes and an HbA1c of 5% is planning to become pregnant soon. What anticipatory guidance should the nurse provide this client? 1. Vascular disease that accompanies diabetes slows progression. 2. The risk of ketoacidosis decreases during the length of the pregnancy. 3. The baby is likely to have a congenital abnormality because of the diabetes. 4. Insulin needs decrease in the first trimester and increase during the third trimester.

Answer: 4 Explanation: 1. Vascular disease progresses more rapidly during pregnancy, especially if blood sugar control is not good. Problems such as nephropathy and retinopathy can result. 2. The risk of ketoacidosis increases during pregnancy. 3. Infants of diabetic mothers have a 5% to 10% greater risk of having a congenital abnormality. This risk increases to 20% to 25% if the HbA1c is over 10%. 4. In addition, insulin requirements drop suddenly after delivery of the placenta.

The nurse is planning an educational session for pregnant clients who are vegans. What information should the nurse include? 1. Vegan diets are high in iron. 2. Rice contains a high level of vitamin B12. 3. Soy is not a good source of protein for vegans. 4. Eating beans and rice provides complete protein needs.

Answer: 4 Explanation: 1. Vegan diets are low in iron, and pregnant vegans often experience anemia. 2. Rice does not contain vitamin B12. 3. Soy is a very good source of protein and calcium and is safe during pregnancy. 4. Complete proteins can be obtained by eating different types of plant-based proteins such as beans and rice.

A 28-year-old woman at 16 weeks' gestation being screened for ABO incompatibility learns that her blood contains anti-A antibodies. What should the nurse explain about this finding? 1. "You may have contracted anti-A antibodies as a result of a viral infection." 2. "It's most likely that you contracted anti-A antibodies through sexual activity." 3. "Anti-A antibodies are inherited; usually, they are genetically passed down from father to daughter." 4. "Anti-A antibodies occur naturally, as a result of exposure to foods and different infections."

Answer: 4 Explanation: 1. Women develop anti-A and anti-B antibodies as a result of exposure to the A and B antigens through infection by gram-negative bacteria and not viruses. 2. Anti-A and anti-B antibodies are naturally occurring; that is, women are naturally exposed to the A and B antigens through the foods they eat and through exposure to infection by gram-negative bacteria. These antibodies are not contracted through sexual activity. 3. Women develop anti-A and anti-B antibodies naturally as a result of exposure to the A and B antigens through the foods they eat and through exposure to infection by gram-negative bacteria. These antibodies are not inherited. 4. Anti-A and anti-B antibodies are naturally occurring; that is, women are naturally exposed to the A and B antigens through the foods they eat and through exposure to infection by gram-negative bacteria.

It has been identified that a pregnant client's diet is low in zinc. Which food should the nurse suggest to increase intake of this mineral? 1. Yogurt 2. Bananas 3. Cabbage 4. Shellfish

Answer: 4 Explanation: 1. Yogurt is high in other nutrients but does not have significant levels of zinc. 2. Bananas are high in other nutrients but do not have significant levels of zinc. 3. Cabbage is high in other nutrients but does not have significant levels of zinc. 4. Zinc is found in greatest concentration in meats and meat by-products. Enriched grains also tend to be high in zinc.

A pregnant woman married to an intravenous drug user had a negative HIV screening test just after missing her first menstrual period. What would indicate that the client needs to be retested for HIV? 1. Elevated blood pressure and ankle edema 2. Shortness of breath and frequent urination 3. Hemoglobin of 11 g/dL and a rapid weight gain 4. Unusual fatigue and recurring Candida vaginitis

Answer: 4 Explanation: 2. The client would have a decrease in blood pressure and no ankle edema. 3. Shortness of breath and frequent urination do not indicate a need to retest for HIV. 1. The client would be anemic and anorexic. 4. The client who is HIV-positive would have a suppressed immune system and would experience symptoms of fatigue and opportunistic infections such as Candida vaginitis.

he nurse reviews the consequences of not treating a chlamydial infection with a young couple. Which statements indicate that teaching has been effective? Select all that apply. 1. "She could become pregnant." 2. "It could cause us to develop rashes." 3. "She could have severe vaginal itching." 4. "She could develop a worse infection of the uterus and tubes." 5. "He could get an infection in the tube that carries the urine out."

Answer: 4, 5 Explanation: 1. Chlamydial infection does not cause a woman to become pregnant. 2. Chlamydial infection does not cause a rash. 3. Chlamydial infection does not cause vaginal itching. 4. Chlamydial cervicitis can ascend and become pelvic inflammatory disease, or infection of the uterus, fallopian tubes, and sometimes ovaries. 5. Chlamydia trachomatis is a major cause of nongonococcal urethritis (NGU) in men.

A client scheduled for genetic testing is concerned that an employer will learn about the testing and terminate the client's employment. How should the nurse respond to this client? 1. "There really is not anything that can be done." 2. "The insurance company has to report the testing to the employer." 3. "Your employer will have this information from the healthcare claim." 4. "There is a federal law prohibiting employers from using genetic information for employment purposes."

Answer: 4Explanation: 1. The client should not be concerned because of a federal law prohibiting employers from discriminating based upon genetic testing. 2. The insurance company is prohibited from disclosing information to the employer. 3. The healthcare claim is not sent to the employer. 4. Federal legislation to prohibit discrimination based on genetic information in health insurance and employment (the Genetic Information Nondiscrimination Act [GINA]) was implemented in November 2009. As a federal law, GINA offers protection to Americans in all states.

A client describes breast swelling and tenderness. Gathering what piece of data would be most important? a. Timing of the symptoms b. Birth control method c. Method of breast self-examination d. Diet history

Answer: a. Timing of the symptoms Feedback: The breast undergoes regular cyclical changes in response to hormonal stimulation. The nurse will want to determine when the swelling and tenderness occurs within the menstrual cycle. Birth control method, method of BSE, and diet history may contribute to the database, but do not have priority.

A married woman presents to the clinic with complaints of grayish vaginal discharge with a "fishy" odor. Which of the following statements demonstrates that your teaching regarding BV has been understood? a. "It is not necessary to treat my partner." b. "I can stop taking this medicine when the discharge looks better." c. "I should avoid Metronidazole gel during pregnancy." d. "BV is a sexually transmitted disease."

Answer: a. "It is not necessary to treat my partner." Feedback: Bacterial vaginosis is not a sexually transmitted disease. Studies have not found a lower risk of reinfection with treating male partners. It is caused by a disruption of normal vaginal flora. Metrogel (Flagyl) is considered safe in pregnancy. Any prescribed course of antibiotics should be completed regardless of symptoms.

A 24-year-old female client is being treated with tinidazole for bacterial vaginosis. Which information should the nurse include in this client's discharge instructions? a. Alcohol should be avoided when taking tinidazole. b. Tinidazole may be administered orally in tablet form or vaginally as a cream. c. Side effects of tinidazole include white or gray vaginal discharge with a foul odor described as "fishy." d. Tinidazole is contraindicated in pregnant clients.

Answer: a. Alcohol should be avoided when taking tinidazole. Feedback: Alcohol should be avoided when taking tinidazole. This medication is administered orally. These are signs and symptoms of bacterial vaginosis infection. Tinidazole is not contraindicated in pregnant clients.

The community health nurse is creating an educational brochure about sexually transmitted infections (STIs). Which information should the nurse include when discussing the two types of herpes infections? a. Both HSV-1 and HSV-2 can cause genital herpes. b. HSV-1 may cause cold sores but does not cause genital herpes. c. If administered early in the course of treatment, acyclovir, valacyclovir, or famciclovir is an effective cure for either type of HSV. d. Emotional stress does not impact the recurrence of HSV.

Answer: a. Both HSV-1 and HSV-2 can cause genital herpes. Feedback: Both HSV-1 and HSV-2 can cause genital herpes. HSV-1 can lead to genital herpes through oral-genital contact. Antiviral medications can suppress or shorten HSV outbreaks, but there is no cure. Stress may precipitate outbreaks.

In order to assess the origin of nipple discharge, the nurse must gather data about what? a. Color and consistency of discharge b. Location of masses c. Whether position dimpling is noticeable d. Density of rash

Answer: a. Color and consistency of discharge Feedback: Determining color and consistency of that discharge assists the nurse in distinguishing between physiologic discharge and pathologic discharge. Masses, dimpling, and presence of rash will not give the nurse data concerning nipple discharge

A 32-year-old woman who is at 29 weeks' gestation presents to the clinic for a routine prenatal visit. Her physician orders a urinalysis, which reveals asymptomatic bacteriuria. Which of the following plans of care should the nurse anticipate? a. Instructing the client about the risks of developing an ascending urinary tract infection b. Explaining that pregnancy is protective against UTIs c. Preparing for administration of a broad-spectrum antibiotic, as infections are usually caused by two separate organisms d. Facilitating additional diagnostic testing, including a blood culture, and encouraging strict bedrest until antibiotic therapy is completed

Answer: a. Explaining the risks of developing an ascending urinary tract infection Feedback: Urinary tract infection in pregnancy increases the risk of pyelonephritis. Urinary tract infection is almost always caused by a single organism, typically Escherichia coli, and treatment should target the causative organism.

The nurse is teaching a class on infertility. Which statement correctly describes infertility? a. Lack of conception after more than 12 months despite unprotected sexual intercourse b. An absolute factor preventing reproduction c. Difficulty conceiving because both partners have decreased fertility d. Women over 35 who are having difficulty conceiving

Answer: a. Lack of conception after more than 12 months despite unprotected sexual intercourse Feedback: Infertility is defined as lack of conception despite unprotected sexual intercourse for over 12 months. When there is an absolute factor preventing reproduction, it is described as sterility. A couple having difficulty conceiving because both partners have decreased fertility describes subfertility (reduced ability to conceive). Being a woman over 35 and having difficulty conceiving is a possible risk factor for infertility but does not describe infertility.

A nurse is educating a group of clients about the risk factors for developing toxic shock syndrome (TSS). Which statement by a client would indicate that she would benefit from more teaching on the risk factors of TSS? a. "I should alternate using tampons and pads during my period." b. "We should use a condom in place of my diaphragm during my period." c. "I should change my tampon every 3-6 hours during my period." d. "I can use tampons as soon as I want to after I have a baby."

Answer: d. "I can use tampons as soon as I want to after I have a baby." Feedback: Tampons should be avoided for 6-8 weeks following childbirth. The risk of TSS is reduced when alternating between pads and tampons. The risk of TSS is reduced when a diaphragm or cervical cap is not used during the menses. The risk of TSS is reduced when tampons are changed every 3-6 hours during the menses

A nurse is teaching a group of college women about prevention of toxic shock syndrome (TSS). Which of the following statements should be included in the seminar? a. Usually, the causative organism of TSS is a toxin released by a strain of Pseudomonas aeruginosa. b. Hypertension is often seen in clients with TSS. c. To avoid TSS, tampons should be changed every 6-9 hours. d. If you get a rash or a fever while using tampons, go to the doctor immediately.

Answer: d. If you get a rash or a fever while using tampons, go to the doctor immediately. Feedback: Early diagnosis and treatment of TSS are important for preventing fatality. The usual causative organism is a toxin released by a strain of Staphylococcus aureus. TSS is associated with hypotension. Tampons should be changed every 3-6 hours to avoid TSS.


Set pelajaran terkait

palmaris longus action, origin, insertion, innervation

View Set

The Federal Reserve and Monetary Policy

View Set

MAN3240 - RESOURCE LIST - MGT 291 Exam 2 Question Bank

View Set

Chapter 2 - The Business Vision and Mission

View Set

Cardiovascular Saunders Study Guide

View Set

Challenges of the World Economy: Sessions 5 and 6

View Set